Sunteți pe pagina 1din 119

cuadernos DIGITALES

Recursos 1 bachillerato
- edit. Oxford -




2011
L
E
N
G
U
A




LENGUA CASTELLANA Y LITERATURA - 1 BACHILLERATO
OXFORD EDUCACIN - TESELA

En la etapa de Bachillerato, la materia de Lengua castellana y Literatura tiene como
objetivo fundamental el desarrollo de los conocimientos necesarios para intervenir de forma
adecuada y satisfactoria en la comprensin y expresin en los diferentes mbitos sociales. El
alumnado deber atender a potenciar el desarrollo de su capacidad comunicativa en todo tipo
de discursos, y estos recursos que os presentamos van en la lnea de adquisicin del mismo.
El presente cuaderno RECURSOS OXFORD EDUCACION est organizado de
acuerdo con el orden de los temas del libro de texto de 1 de Bachillerato. De esta manera,
las actividades se distribuyen en dos grandes bloques:
1) La variedad de los discursos, el tratamiento de la informacin y el conocimiento de la
lengua. Se corresponde con las actividades pertenecientes a las lecciones 1 a 10.
2) El discurso literario. Se corresponde con el resto de actividades y que pertenecen a las
lecciones 11 a 20.
Cada unidad se completa con una pgina denominada Ideas claras, que se dedica
al resumen de las cuestiones tratadas; una pgina de Actividades de repaso; otra llamada
Norma y uso, centrada en cuestiones normativas y una ltima destinada a Evaluacin.
Resultar muy conveniente que lo trabajes ya que suponen una tarea de sntesis muy
aprovechable de los contenidos bsicos de cada leccin.
Dependiendo de los objetivos de la asignatura de Lengua castellana y Literatura que
no hayas alcanzado, debers trabajar unas actividades u otras. Esperamos que os sean de
provecho.

EL DEPARTAMENTO DE LENGUA Y LITERATURA
I.E.S. JUAN DE LA CIERVA - VLEZ MLAGA ( MLAGA )
.
REFUERZO
1. La comunicacin.
Funciones del lenguaje
Los marcos de referencia permiten establecer distintas situaciones de comunicacin y el papel
que desempean en ella los participantes. En el caso de los textos literarios, es posible recono-
cer dos procesos comunicativos: uno externo, en el que participamos como lectores, y otro
interno, que se establece en la ficcin creada en ellos. Adems, en estos textos pueden manifes-
tarse las distintas funciones que cumple el lenguaje.
Arxemiro
En ese instante, despert Alguien me hablaba. Me pareci que despertaba de otro
mundo. Delante de m haba un fantasma, un fantasma que haba visto muchas veces.
Era Arxemiro, que tena el candil encendido en la mano. Le pregunt:
Qu te pasa? Te ha venido a visitar el enemigo? Has visto a la Santa Compa-
a? Por qu ests tan plido?
Peor que eso me contest. Me vi muerto. No volvera a mi cuarto por nada
del mundo.
La voz de Arxemiro pareca que vena de muy lejos. Yo no saba qu hacer. Me esta-
llaba la cabeza. Por fin, le dije:
Era el vino, hombre, era el vino. Vulvete a acostar.
l me contest:
No, Bieito. Era la muerte! Escucha y despus, juzga. Cuando llegu a mi cuarto,
encend el candil para acostarme. Me fui desnudando poco a poco, dejando la ropa en
el suelo. Ya me haba quitado la guerrera y la camisa. Todava la tena en la mano. No s
cmo mir para la cama. All haba un hombre durmiendo con la cabeza tapada por la
manta. Qu hara yo? Le quit la ropa
A Arxemiro le castaeaban los dientes como a un hombre desnudo una maana de
helada. Yo me sent en la cama. Su voz ya no era que viniese de lejos: era la voz
de otro. Hablaba tartamudeando y me dijo:
Era yo, que estaba muerto!
Tena que quitarle aquel miedo, fuera como fuera.
Ahora mismo le dije vamos all.
nxel FOLE
en De cmo me encontr con el demonio en Vigo, Trama
A qu tipo de texto pertenece el fragmento
segn su intencin comunicativa?
Reconoce los elementos de la comunicacin
que se observan en el texto.
Razona por qu hay intencin comunicativa
en el dilogo.
En la segunda y en la tercera pregunta que
formula el primer interlocutor, se menciona
al enemigo y a la Santa Compaa. Qu tipo
de contexto aclara las interrogaciones?
Arxemiro menciona la guerrera. Explica por
qu el significado de esta palabra lo aclara
el contexto lingstico y brinda un ejemplo
en el que dicho trmino adquiera otro
significado.
Qu intencin tiene el interlocutor
de Arxemiro al decirle Era el vino, hombre,
era el vino?
Has analizado el proceso comunicativo interno,
el que se establece en el texto. Explica ahora
los elementos del proceso comunicativo
externo.
Indica, con ejemplos, las funciones
del lenguaje que se pueden observar
en el fragmento.
Seala algunos rasgos lingsticos
caractersticos presentes en cada una de ellas.
Si no has encontrado alguna de las funciones
del lenguaje, escribe enunciados que las
ejemplifiquen.
10
9
8
7
6
5
4
3
2
1
23 1. La comunicacin
ACTI VI DADES
1
M
A
T
E
R
I
A
L

F
O
T
O
C
O
P
I
A
B
L
E

/


O
x
f
o
r
d

U
n
i
v
e
r
s
i
t
y

P
r
e
s
s

E
s
p
a

a
,
S
.
A
.
0B1LLLP(2008).01+s 3/6/08 19:07 Pgina 23
REFUERZO
2. Tipos de signos
Recuerda que signo es aquello que est en lugar de otra cosa y que los signos pueden clasifi-
carse en funcin de diferentes criterios: fuente de emisin, intencionalidad y conciencia del
emisor, aparato receptor y vnculo con el referente (clasificacin de Peirce).
Indica de qu tipo son los signos
representados en las imgenes en funcin
de los criterios que figuran en la pgina 13
de tu libro de texto.
Seala, en todos los casos, la realidad a la que
sustituyen dichos signos.
Considerando que esos signos se han
representado por medio de dibujos, qu
relacin guardan las imgenes con la realidad
que pretenden representar?
A qu tipo de signo corresponde esa relacin?
Recuerda los criterios de clasificacin
de la pgina 13 de tu libro.
Cita otros signos que correspondan a cada
una de las tipologas representadas
en las imgenes.
Qu tipos de signos de los que figuran
en el cuadro de la pgina 13 no se incluyen
en los dibujos? Cita ejemplos de cada uno
de ellos.
6
5
4
3
2
1
ACTI VI DADES
1
24 1. La comunicacin
M
A
T
E
R
I
A
L

F
O
T
O
C
O
P
I
A
B
L
E

/


O
x
f
o
r
d

U
n
i
v
e
r
s
i
t
y

P
r
e
s
s

E
s
p
a

a
,
S
.
A
.
0B1LLLP(2008).01+s 3/6/08 19:07 Pgina 24
REFUERZO
3. Los cdigos no verbales
Las siguientes imgenes y el fragmento de Adelaida Garca Morales registran diversos signos
pertenecientes a los cdigos paralingstico, cinsico y proxmico, que constituyen sistemas
de comunicacin que funcionan junto con el lenguaje verbal.
A qu cdigos no verbales corresponden
los rostros, los grupos de personas
y las referencias del texto?
Explica qu sentimientos expresa cada una
de las caras dibujadas en esta pgina.
Cmo se refleja la relacin existente entre
las personas en su disposicin en el espacio?
Seala las referencias paralingsticas del
texto de Adelaida Garca Morales e indica
qu tipo de informacin comunican.
Escribe de nuevo ese texto aadiendo otras
referencias paralingsticas e incorporando
elementos relativos a la cinsica y a la
proxmica.
5
4
3
2
1
25 1. La comunicacin
ACTI VI DADES
1
M
A
T
E
R
I
A
L

F
O
T
O
C
O
P
I
A
B
L
E

/


O
x
f
o
r
d

U
n
i
v
e
r
s
i
t
y

P
r
e
s
s

E
s
p
a

a
,
S
.
A
.
La ausencia de Bene
Qu pasa? dijo con sobresalto al escuchar mi voz.
Tengo miedo le respond, deseando que recordara un tiempo ya pasado en el
que yo le despertaba por las noches con esa misma frase. Pero esta vez me respondi
fastidiado:
Todava tienes miedo? Con lo mayor que eres!
Estoy asustada por Bene. Me parece que le est pasando algo en este momento
dije, tratando de justificarme y segura de que aquellas palabras le despertaran de
una vez.
Qu dices?! me contest irritado, pero mostrando al mismo tiempo una gran
preocupacin.
Bene no est en su habitacin dije lentamente, como si le notificara algo muy
grave.
Qu tontera! me respondi. Estar en el cuarto de bao.
No, no est all, ni tampoco en el jardn. La he buscado por todas partes, tambin
en la torre. No est en ningn sitio.
Y a ti qu te importa dnde est Bene? me dijo malhumorado, y despus
aadi: Vete ya a dormir y deja de espiarla o te llevars un susto.
Por qu?
Por nada, nia. Pareces tonta.
Adelaida GARCA MORALES
El sur & Bene, Anagrama
0B1LLLP(2008).01+s 3/6/08 19:07 Pgina 25
REFUERZO
4. Variedades de la lengua
Cada lengua presenta diferencias o variedades internas: diatpicas, que constituyen sus dialec-
tos, diastrticas, que se manifiestan en sociolectos, y diafsicas, que se plasman en distintos
registros de lengua. En los siguientes textos puedes observar algunas de estas variedades.
Qu variedad interna del castellano presenta
el texto 1? Dnde se registra? Seala los
rasgos lingsticos presentes en el texto
caractersticos de esta variedad.
Explica la diferencia en el habla de cada uno
de los interlocutores del texto 2. Cmo se
denominan estos usos? Indica los rasgos
especficos de las palabras de Anastasia;
cmo denominaras a su manera de hablar?
Qu tipo de variedad se observa en el texto 3?
Indica sus rasgos caractersticos. Transforma el
texto pensando en que la primera persona que
habla est en una entrevista de trabajo.
Caracteriza el texto 4 segn su variedad. Seala
vocablos que permitan tal caracterizacin. Qu
tipo de lector exige este fragmento?
Redacta un texto sobre las variedades del
castellano.
5
4
3
2
1
ACTI VI DADES
1
26 1. La comunicacin
M
A
T
E
R
I
A
L

F
O
T
O
C
O
P
I
A
B
L
E

/


O
x
f
o
r
d

U
n
i
v
e
r
s
i
t
y

P
r
e
s
s

E
s
p
a

a
,
S
.
A
.
Texto 1
PADRE.tonces, hijo? O quers decir
algo ms?
NIO.S.
PADRE.Qu quers decir?
NIO.No s.
PADRE.A ver, pens.
NIO.No s.
PADRE.Y, dec cualquier cosa, lo primero
que se te ocurra
NIO.Qu?
PADRE.Una palabra, por ejemplo.
NIO.rbol!
Andrs NEUMAN
Una vez Argentina, Anagrama
Texto 3
Pues yo me curro un ritmo gua-
po y luego le meto la meloda que me va
saliendo, medio inventada. Yo no tengo
ni guarra de solfeo, sabes, yo saco tres
acordes, un ritmo y ya est; luego mete-
mos la batera y el bajo, y as las cancio-
nes salen como churros. Cuando no ten-
go muchas ganas de currarme una letra,
le digo al cantante que se la curre l,
sabes
Qu vas a hacer este verano?
Currar para ver si me saco unas
pelas para agosto. Voy a ver si puedo
sacarme un curro como socorrista y lue-
go puede que vaya a ver a Santi, el bate-
ra, a la Manga.
Jos ngel MAAS
Historias del Kronen, Destino
Texto 2
PEPE.Ahora lo que yo deploro vivsimamente es haber venido a produ-
cir a ustedes esta molestia suntuaria, este trasiego ornamental
ANASTASIA.No, seor; no faltara otra cosa. Muchsimo gusto. Lo que usts
se merecen y naa ms.
PEPE.Oh! No diga usted eso; tanto agasajo nosotros, dos personas
tan
ANASTASIA.Y una lo que siente es no haber sabo antes lo que eran usts.
PEPE.Oh, eso no, por Dios! Pero qu es lo que somos nosotros, diga
usted? Haga el favor de decrmelo! Qu somos nosotros?
ANASTASIA.Toma, pues meno! Digo Nada! Una friolera! Y por
qu no han quero usts decirlo al llegar?
PEPE.Pues no lo hemos querido decir porque francamente porque no
lo sabamos que aqu se nos estimase de una manera tan halagea.
ANASTASIA.Aqu, crea el seor, que, aunque esto es un humilde pueblo, se
sabe tratar a las presonas de categora, como son los excelentsimos
seores. (Aparte.) Voy a ver si son melitares. (Alto.) Y usts de qu
son?
PEPE.(Palpndose con asombro.) Cmo que de qu somos?
Carlos ARNICHES
El santo de la Isidra. El amigo Melquades. Los caciques, Alianza
Texto 4
La teora de la relevancia est basada en la tesis de que la atencin
y el pensamiento humano obedecen a una constante bsqueda de infor-
macin relevante, o sea, informacin que produzca efectos cognoscitivos
adecuados a cambio de un esfuerzo de procesamiento mnimo. A este
principio general sobre la cognicin humana, Sperber y Wilson, que pre-
sentaron la teora de la relevancia en 1986, y, en versin revisada, en
1995, aaden un segundo principio general acerca de la comunicacin,
que puede enunciarse as: Cada acto de comunicacin ostensiva comu-
nica la presuncin de su propia relevancia ptima. Este segundo princi-
pio es el ms importante en pragmtica, y en lo que sigue lo llamaremos
simplemente: principio de relevancia.
Graciela REYES, Elisa BAENA y Eduardo URIOS
Ejercicios de pragmtica (I), Arco Libros
0B1LLLP(2008).01+s 3/6/08 19:07 Pgina 26
ORTOGRAFA
5. Uso de la tilde
Para el correcto uso de la tilde debes recordar:
Las reglas generales de acentuacin. Las palabras agudas llevan tilde cuando terminan
en -n, -s o vocal (redaccin, ciprs, llev; laurel, radar, cantad); las llanas, cuando no acaban
en esas letras (cncer, husped, rquiem; cantan, respetas, escriba); las esdrjulas y sobrees-
drjulas siempre llevan tilde (nica, dgaselo). En las llanas, un caso especial lo constituyen
aquellas palabras que acaban en -s precedida de otra consonante, las cuales llevan tilde:
bceps, frceps.
Los hiatos formados por una vocal abierta (a, e, o) tona y una cerrada (i, u) tnica, o vicever-
sa, siempre llevan tilde: raz, bal, redo, odo, tena, huamos.
En los adverbios en -mente, si el adjetivo base de derivacin lleva tilde, la palabra derivada
la mantiene: corts > cortsmente.
Por regla general, los monoslabos no llevan tilde: sol, di, dio, fue, fui
La tilde diacrtica permite diferenciar palabras con los mismos sonidos pero con valores
gramaticales y significados diferentes: an (Adv, todava) / aun (Conj, incluso); d (Verbo
dar) / de (Prep); l (Pron) / el (Art); ms (Adv) / mas (Conj); m (Pron) / mi (Adj Pos); s (Verbo ser
o saber) / se (Pron); s (Adv de afirmacin) / si (Conj); t (Sust) / te (Pron); t (Pron) / tu (Adj Pos).
Los pronombres, adjetivos y adverbios interrogativos y exclamativos son palabras tnicas y
llevan tilde: qu, quin, cul, cmo, dnde, cundo, cunto.
El uso de la tilde es potestativo en el caso de solo (Adv, solamente) y este / a / s (Pron
Dem), salvo cuando pueda haber ambigedad, en cuyo caso deben llevarla: Trabaja solo por
las noches (Adj) / Trabaja slo por las noches (Adv); Esta maana los traer (Adj Dem) / sta
maana los traer (Pron Dem).
Las maysculas siempre llevan tilde: ngel, vila, CRDOBA
Coloca la tilde cuando corresponda.
El proceso de indiferencia
La inferencia es el proceso por el que se reconstruyen los vinculos que permiten ligar la
seal indicial y el contenido al que esta se refiere. Tipicamente, la recuperacion de este vinculo
esta basada en la utilizacion de los conocimientos extralingisticos, que permiten salvar la
distancia que media entre el estimulo utilizado y su contenido. []
La inferencia es, basicamente, un proceso por el que se integran los contenidos de dife-
rentes representaciones internas, que pueden proceder, a su vez, de diferentes fuentes.
Cuando procesamos datos lingisticos, combinamos la informacion obtenida a partir de la
descodificacion del mensaje linguistico con otra informacion contextual.
Son, por tanto, inferenciales los procesos que nos permiten especificar los contenidos
codificados lingisticamente, como, por ejemplo, los que llevamos a cabo para identificar
los referentes a los que aluden las expresiones linguisticas (a que se refiere la palabra casco
en la frase Tendras que limpiar el casco?), o los que especifican contenidos vagos (cuando
decimos El mio es mas rapido queremos decir que es mas rapido con respecto a que?). Y lo
son tambien los que nos conducen a determinar que es exactamente lo que el interlocutor
quiso comunicarnos, o en que sentido hay que tomar sus palabras (lo que dijo, era una
sugerencia, o un consejo, o una orden, o una amenaza?).
Sin embargo, no hay que perder de vista que los procesos inferenciales no son exclusivos
del procesamiento de enunciados lingisticos: se dan tambien en la interpretacion de datos
de otras fuentes, tanto si han sido producidos intencionalmente como si no (como cuando
inferimos datos sobre la procedencia geografica de alguien a partir de su acento). Esto indica,
por tanto, que los procesos inferenciales tienen lugar con independencia de que haya comu-
nicacin; como vimos, tendremos que deducir informacion a partir de cualquier clase de
datos disponible; la principal diferencia entre las seales producidas intencionalmente y la
observacion de datos fortuitos radica en que solo las primeras se interpretan desde el supues-
to de que quien las produce las respalda.
M. Victoria ESCANDELL VIDAL
La comunicacin, Gredos
1
27 1. La comunicacin
ACTI VI DADES
1
M
A
T
E
R
I
A
L

F
O
T
O
C
O
P
I
A
B
L
E

/


O
x
f
o
r
d

U
n
i
v
e
r
s
i
t
y

P
r
e
s
s

E
s
p
a

a
,
S
.
A
.
0B1LLLP(2008).01+s 3/6/08 19:07 Pgina 27
Evaluacin 1
23 1. La comunicacin
1. Resume el contenido del texto y reconoce los elemen-
tos de la comunicacin que se establecen en l. Diferen-
cia estos elementos de los del proceso externo de la
comunicacin que se constituye al leer el texto.
2. De qu tipo de texto se trata segn su objetivo comu-
nicativo? Seala otras clases posibles, atendiendo al
mismo criterio de clasificacin.
3. Qu intencin comunicativa manifiesta el protagonista
al proferir el largo parlamento que comienza con Veo,
caballero ?
4. Localiza en el fragmento de Eduardo Mendoza algunas
menciones de signos visuales.
5. En el texto se hace referencia a signos no verbales.
Copia las expresiones correspondientes y clasifcalas en
un cuadro como este.
6. Caracteriza los conceptos de lengua histrica y lengua
funcional y pon ejemplos tomados del texto. Seala,
adems, los tipos de variedades que presenta cualquier
lengua histrica.
Lengua histrica Lengua funcional
7. Escoge una palabra del fragmento y explica, a partir de
ella, la nocin de signo lingstico.
Signo lingstico
8. Qu tipo de signo es, segn la clasificacin de Pierce, la
palabra que has escogido en la actividad 7? Elige una de
las opciones, justifica tu respuesta y explica la diferencia
entre este tipo de signo y los otros dos.
a) Smbolo.
b) Icono.
c) ndice.
9. Reconoce las funciones del lenguaje presentes en el texto
y brinda ejemplos para las que no encuentres.
10. Redacta un texto en el que expliques las caractersticas
de la facultad del lenguaje y de las lenguas humanas.
No olvides cuidar la ortografa y la presentacin de tu
escrito.
El sueco
En qu puedo servirle?
Quiero pasar asever el sueco con voz temblorosa.
Vacil unos instantes, pero acab franquendole el paso, ya
que se trataba de un cliente de mi hermana, autodenominado
novio, por ms seas, y no me convena de modo alguno ene-
mistarme con ella. Pens que quiz quera discutir algn asunto
de familia y que, siendo yo el varn, me consideraba el interlo-
cutor idneo para ello. Esta fineza, ya anacrnica, y algo en el
aspecto del sueco me decan que estaba en presencia de un
hombre de bien, y no menoscab mi estima el hecho de que
sacara un pistoln de la faltriquera y me encaonara con l al
tiempo que se sentaba en la cama. Pero me dan miedo las
armas, o no habra tomado mi carrera delictiva tan corto vuelo,
y as se lo hice saber.
Veo, caballero dije lentamente, con profusin de adema-
nes y procurando vocalizar bien para que la barrera del idioma no
fuera bice a nuestro entendimiento mutuo , que algo le impul-
s a desconfiar de m: quiz el natural recelo que inspira mi facha,
quiz un rumor de esos a cuya divulgacin son dadas las malas
lenguas. Sin embargo, puedo asegurarle por mi honor, el de mi
hermana, sister, y el de nuestra santa madre, que Dios haya en su
gloria, que no tiene usted nada que temer de m. Soy perspicaz y,
aunque no tenga el placer de conocerle salvo superficialmente,
no he dejado de advertir que es usted hombre de principios, ins-
truido, cabal y de buena cuna, a quien acaso reveses de fortuna
han lanzado a una vida desasosegada en pos de ms amplios
horizontes, del olvido, incluso.
Mi llaneza no pareca hacer mella en su obstinacin. Segua
sentado en la cama, con los ojos clavados en m y el rostro
inexpresivo, perdidos sin duda sus pensamientos en quin sabe
qu recuerdos dolorosos, qu visiones indescriptibles, qu
melancola.
Eduardo MENDOZA
El misterio de la cripta embrujada, Seix Barral
CDIGOS NO VERBALES
Paralingstica
Cinsica
Proxmica
Discusin en la noche, por Edward Hopper.
0B1LLLA.01 31/3/08 21:26 Pgina 23
REFUERZO
1. La conversacin
La conversacin en la que se emplea la lengua coloquial es el discurso prototpico de la
comunicacin oral. A veces, los textos escritos imitan o reproducen la variedad coloquial con
distintos objetivos.
Qu tipo de informacin aportan las
acotaciones en el primer texto? A qu
elementos del acto comunicativo se refieren?
Hay informacin del mismo tipo en el
segundo texto? Qu diferencias encuentras
en el registro de habla de los interlocutores?
Lee el primer texto aportando los aspectos
prosdicos y los elementos no verbales
propios de la comunicacin oral.
Distingue qu tipo de conversacin
(oral coloquial o escrita que imita la variedad
coloquial) se representa en cada uno
de los fragmentos.
Reconstruye una versin escrita del segundo
texto similar al anterior.
Reconoce las caractersticas de la lengua
coloquial presentes en los textos.
6
5
4
3
2
1
37 2. La comunicacin oral y escrita
M
A
T
E
R
I
A
L

F
O
T
O
C
O
P
I
A
B
L
E

/


O
x
f
o
r
d

U
n
i
v
e
r
s
i
t
y

P
r
e
s
s

E
s
p
a

a
,
S
.
A
.
ACTI VI DADES
2
Texto 1
CHUSA.Dnde estn estos?
JAIMITO.Se han largado.
CHUSA.Adnde?
JAIMITO.(Sale del lavabo y se acerca.) Se han largado del todo; se han abierto, ta. Se han llevado sus cosas
Quedan esas cajas de ah; van a venir luego a por ellas. En eso han quedado.
(De pronto ella toma contacto con la realidad. Ve las cajas. Luego las cosas que faltan y el cambio en la
habitacin.)
CHUSA.(Deja de guardar la ropa y se sienta muy afectada.) Pero, cmo? Qu ha pasado?
JAIMITO.Se han largado, juntos, los dos. Los dos y sus madres. Los cuatro. Bueno, y el padre. Se van a casar. Han
cogido un piso en Mstoles. El da que yo sal del hospital, y te cogieron a ti, fue todo un lo.
CHUSA.Qu tal sigue tu brazo?
JAIMITO.(Sacndole y metindole del pauelo con que se le sujeta al cuello.) Bueno, mira. Le puedo mover ya.
Maana o pasado me quito esto. Pues nada, que se han ido.
CHUSA.Alberto tambin?
JAIMITO.No te digo que se han ido los dos juntos?
Jos Luis ALONSO DE SANTOS
Bajarse al moro, Ctedra
Texto 2
S1: t qu Csar? que como nunca has fumao ni nada no tienes problemas/// t no has
fumao nunca
C1: si/ fumaba
S2: mm?
C2: fumaba yy- y lo dej/ tam(b)in/ me senta mal del pecho// yy lo dej/ yy- lo que pasa es
que noo// yohasta los dieciocho aos no fumaba
J: yo hasta los diecinueve
C3: yy luego he estao fumando hastaa hacee dos aos o as / peroo no- no me gustaba
nadaa/ as paraa/ yo de pequeo haba tenido bronquitis/ y cosas de este tipo/ y ahora
to(d)avaa/ mm-cada poco tengo de la gargantao dee/ y entonces lo del fumar era/ era
una/ era una pesadez
Antonio BRIZ GMEZ
El espaol coloquial en la conversacin, Ariel
0B1LLLP(2008).02+s 3/6/08 19:09 Pgina 37
REFUERZO
2. La entrevista
La entrevista es una conversacin que informa sobre las opiniones de una persona. Con fre-
cuencia, las noticias o los reportajes de la prensa incluyen declaraciones, dilogos con el perio-
dista, que no tienen la forma de una entrevista larga y formal. En este caso, podemos distinguir
tres tipos de representacin escrita del dilogo: una que comienza con la presentacin del
entrevistado y contina con la modalidad pregunta-respuesta; otra que contiene la reproduc-
cin de las respuestas del entrevistado, con la intercalacin de informacin sobre su vida y obra
y comentarios o descripciones del entrevistador; y finalmente la que mezcla ambas formas.
38 2. La comunicacin oral y escrita
M
A
T
E
R
I
A
L

F
O
T
O
C
O
P
I
A
B
L
E

/


O
x
f
o
r
d

U
n
i
v
e
r
s
i
t
y

P
r
e
s
s

E
s
p
a

a
,
S
.
A
.
ACTI VI DADES
2
Texto 1
Empieza la conversacin hablando de cmo encuentran
los temas que llevan a los escenarios.
No somos especialmente originales, reconoce Fernn-
dez. Tomamos los temas de aquello que nos choca de la pro-
pia realidad, cosas que vemos en la prensa o a nuestro alrede-
dor. Luego est la historia. Y, no puedo evitarlo, soy fillogo,
la revisin de la literatura.
En mi caso, a veces surge de la imaginacin de forma aza-
rosa y en otras ocasiones del deseo de tratar un tema que me
lleva a algo concreto para lo que tengo que buscar la forma,
explica Mayorga. La tortuga de Darwin procede directamen-
te de esta foto (muestra un recorte de peridico []) que me
dispar la imaginacin por un personaje que ha sido testigo
de la Revolucin de Octubre y la perestroika, ha conocido
a doce papas y a treinta y cinco presidentes de Estados Unidos.
Me pareci interesante y busqu la forma teatral. []
P.Y qu echan de menos en la escena espaola?
J. MAYORGA.Una escritura original que consiga un teatro
completamente actual a la vez que intemporal. Espacios,
como en Inglaterra y Francia, que desafen a los autores
para que sus obras no sean la rplica de lo que sale en los
peridicos.
J. R. FERNNDEZ.Que las obras puedan reposar y madurar
en los escenarios. La mirada de los jvenes autores y la de los
mayores.
El Cultural, 7 de febrero de 2008
Texto 2
S, es guapsima. Ojos de mar, pmulos redon-
dos, labios llenos, piel de melocotn, una preciosi-
dad. Cierto que llega maquillada de un acto de las
marcas a las que vende cara su imagen. Pero la luz
que emana su rostro supera las prestaciones del
mejor iluminador cosmtico. Parece que se acaba
de tragar, encendida, una bombilla de cien vatios.
No puedes dejar de mirarla.
P.Cundo fue consciente del efecto que cau-
saba en los dems?
R.De nia. Tendra doce aos. Los chicos
empezaron a dejarme notitas diciendo que estaban
por m. Y me lo cre, me puse chulita, me crec. Pero
mis amigas me pusieron en mi sitio. Me dejaron de
lado, completamente sola. Fue muy duro, me di
cuenta de que gustar es agradable, pero hay otras
cosas mucho ms importantes. Aprend la leccin.
El Pas Semanal
Texto 3
El Libro Guinness de los rcords tiene registrada como la
mujer ms anciana del mundo a Edna Parker, de Illinois (EE. UU.),
con 115 aos. Obviamente, no tienen constancia de la existencia
de la abuela de El Vacie (Mara Daz Corts, 116 aos, de una
barriada de Sevilla). A Mara esas historias no la interesan. Est
cansada de las televisiones y los peridicos. Yo qu soy, un
divertimento na ms?, protesta.
Son las dos. Aunque es la hora de comer, la abuela Mara deci-
de volverse a la cama. Estoy cansada y se me va la memoria, se
disculpa. Su hija y su nieta la cogen en volandas. Es tan menuda
que apenas las cuesta levantarla. Antes de desaparecer, mira des-
confiada a la visita: Qu dice esa paya? Qu me van a dar la
casita o qu?, pregunta ceuda. Manuel se acerca a su bisabue-
la, con un chupete en la boca, y ella, desde arriba, le mira fijamen-
te. Pero qu guapo est mi nio, presume a gritos. Entonces la
preguntan que cmo se siente ella. Y Mara sentencia, resuelta:
Pues igual de joda que siempre, hija. Cmo voy a estar?.
El Pas, 3 de febrero de 2008
Cmo se estructuran los dos primeros textos?
Indica los temas principales del primer texto.
Cmo se sealan las declaraciones de los
dramaturgos?
En qu tipo de entrevistas incluiras el
segundo texto? Por qu?
La intervencin del periodista del texto 2,
aporta informacin sobre el personaje?
El tercer texto constituye una entrevista?
En qu gnero periodstico incluiras el texto?
Identifica rasgos de la lengua coloquial
en estos textos.
Realiza una entrevista a un compaero
o compaera que desarrolle una actividad
literaria, artstica o deportiva. Utiliza la
modalidad pregunta-respuesta y reelabora la
informacin siguiendo el modelo del texto 2.
7
6
5
4
3
2
1
0B1LLLP(2008).02+s 3/6/08 19:09 Pgina 38
ORTOGRAFA
3. Signos de exclamacin,
interrogacin y comillas
Recuerda que los signos de interrogacin y exclamacin se utilizan para delimitar enunciados interro-
gativos y exclamativos en estilo directo.
Son dos (apertura y cierre) y se colocan al principio y al final del enunciado. Despus de los signos
no se escribe nunca punto.
El signo de principio de pregunta o exclamacin se coloca donde esta empieza, aunque no
coincida con el comienzo del enunciado: Pero entonces, por qu no se lo dijiste?; Lo esper en vano,
se haba ido sin m!
Si se escriben varias preguntas o exclamaciones seguidas y breves, se pueden considerar oracio-
nes independientes, con mayscula cada una y sus signos respectivos: Eso te dijo? Por qu no le
contestaste?; Silencio! Empieza la funcin! Si forman parte de un mismo enunciado, hay que
separar las oraciones por comas o por puntos y comas y solo la primera se escribir con mayscu-
las: Qu ha pasado?, quin ha sido?, por qu?; Qu calor!, abre la ventana!, no puedo respirar!
Las comillas, dobles () o simples ( ) se utilizan en los siguientes casos:
Para reproducir citas textuales. Los chicos opinan: No es tan guapo como dicen.
En los textos narrativos en ocasiones se utilizan las comillas para reproducir los pensamientos
de los personajes, a diferencia de las rayas que transcriben el discurso directo.
Para indicar que una palabra o expresin es impropia, vulgar o de otra lengua, se utiliza de forma
irnica o con un significado especial (Hacerlo? Pa qu?; Hoy tambin ha faltado, est
enferma), o cuando se comenta o se trata una palabra en particular. En estos dos ltimos
casos, en textos impresos, se utiliza la letra cursiva o se cambia el tipo de letra.
Transforma el texto 1 en un dilogo en estilo
directo.
Coloca las comillas que faltan en el segundo
texto.
Pon los signos de interrogacin y los puntos
que faltan en el texto 3.
Escribe una narracin (sin dilogos)
del dilogo citado.
4
3
2
1
39 2. La comunicacin oral y escrita
M
A
T
E
R
I
A
L

F
O
T
O
C
O
P
I
A
B
L
E

/


O
x
f
o
r
d

U
n
i
v
e
r
s
i
t
y

P
r
e
s
s

E
s
p
a

a
,
S
.
A
.
ACTI VI DADES
2
Texto 1
Era la segunda vez en mi vida que alguien me llama-
ba escritor, y me abrum una mezcla inextricable de ver-
genza y de orgullo, y tambin una oleada de afecto por
Rodney. No dije nada, pero, como el hombre no pareca
dispuesto a invitarme a entrar, ni a deshacer el silencio,
por decir algo le pregunt si era el padre de Rodney.
Me dijo que s. Luego volv a preguntarle por Rodney y me
respondi que no saba dnde estaba.
Texto 3
Qu es lo que queras preguntarme []
Quin es Tommy Birban
La cara de Rodney no se alter, y yo no supe
leer la mirada de su ojo nico, o quiz es que
no haba nada que leer en ella. Cuando habl
a continuacin consigui que su voz sonara
natural
De dnde has sacado ese nombre
Lo mencion tu padre Dijo que antes de
que te marchases de Urbana t y l hablasteis
por telfono, y que por eso te marchaste
No te dijo nada ms
Qu ms debera haberme dicho []
En aquel momento anunciaron por mega-
fona la llegada inminente del tren de Atocha
Tommy era un compaero dijo Rod-
ney Lleg a Luang Nai cuando yo era un
veterano, y nos hicimos muy amigos Nos mar-
chamos de all casi al mismo tiempo, y desde
entonces no he vuelto a verle
Javier CERCAS
La velocidad de la luz, Tusquets
Texto 2
A Rodney no le interesaba discutir el argumento de
mi libro, que era en cambio el punto que ms me preocu-
paba a m, sino quin desarrollaba el argumento. Las his-
torias no existen, me dijo una vez. Lo que s existe es
quien las cuenta. Si sabes quin es, hay historia; si no
sabes quin es no hay historia. Entonces yo ya tengo la
ma, le dije. Le expliqu que lo nico que tena claro en mi
novela era precisamente la identidad del narrador: un
tipo exactamente igual que yo que se hallaba exacta-
mente en las mismas circunstancias que yo. Entonces el
narrador eres t mismo?, conjetur Rodney. Ni hablar,
dije, contento de ser ahora yo quien consegua confun-
dirle. Se parece en todo a m pero no soy yo.
0B1LLLP(2008).02+s 3/6/08 19:09 Pgina 39
Evaluacin 2
37 2. La comunicacin oral y escrita
1. Redacta un resumen del contenido del texto de Carmen
Martn Gaite.
2. Identifica las modalidades textuales que aparecen en
ese fragmento y reconoce las funciones del lenguaje
predominantes.
Modalidades textuales
Funciones del lenguaje
3. En este texto se imita un gnero oral. Indica qu carac-
tersticas propias de una conversacin cotidiana observas
en las palabras de las dos mujeres.
4. Indica el registro y el nivel lingstico utilizados por los
personajes. Cul de ellos emplea ms expresiones colo-
quiales? Menciona algunas de ellas.
5. Cita los principales gneros orales dialogados y enumera
las caractersticas de cada uno.
6. En qu se distinguen los gneros orales monologados
de los dialogados? Expn las diferencias entre unos y
otros brevemente.
7. Existe una oposicin clara entre el habla y la escritura?
Cita ejemplos de textos escritos con rasgos orales.
8. Enuncia tres criterios de clasificacin de los textos escritos.
9. Confecciona una tabla con las principales diferencias
entre el habla y la escritura.
10. Escribe una narracin (sin incluir ningn dilogo) acerca
de lo que ocurre y se dice en el fragmento Compromiso
con mayo.
!)# u $ ' !)# u !. -% /
Compromiso con mayo
Al seor me lo he encontrado en el portal cuando yo
entraba dijo Consuelo. Iba de mala leche. O, bueno, no s
si es que la tiene tomada conmigo.
No mujer. Es que trabaja mucho.
Jolines, pero tambin ganar pasta. El que quiere la col,
quiere las hojitas de alrededor, no? Ahora, eso s, lo que est
es muy moderno. Se da un flash a Mario Conde.
Luego me pregunt que si anoche habamos estado de
fiesta.
S, pero no aqu, en casa de otra gente. Una casa a todo
tren. Te acuerdas de aquel seor alto que nos arregl el cuarto
de bao? Pues all.
El del Escorial? Vaya que si me acuerdo. Estaba como
para hacerle padre, no cree usted?
Yo no. Pero gustos son gustos.
La nevera estaba pelada. Me tom el caf y le dej dinero a
Consuelo para que hiciera una buena compra y les preparara
algn guiso rico a las chicas, que seguramente se quedaran
a comer.
Qu pasa? Que usted se larga?
P ues s, hija, me largo. Es uno de mayo y me voy por ah a
celebrarlo a mi manera.
Es el aniversario de su boda?
De mis bodas con mayo. Has visto qu da hace? Aqu
estoy de ms y mayo me echa de menos.
Consuelo se qued mirndome con los ojos muy abiertos.
Qu fuerte!, mayo me echa de menos. Lo ha inventado
usted?
Carmen MARTN GAITE
Nubosidad variable, Crculo de Lectores La mujer de la casa, por Marcel Gromaire.
0B1LLLA.02 31/3/08 21:30 Pgina 37
DOCUMENTACIN
1. Lenguas y legislacin
El carcter plurilinge del Estado espaol est reconocido en la legislacin. La Constitucin
espaola de 1978, en su artculo 3. (reproducido en la pgina 39 del Libro del alumno) establece
la oficialidad de las lenguas. Adems, cada una de las comunidades autnomas bilinges ha
legislado al respecto en sus estatutos de autonoma.
49 3. La realidad plurilinge de Espaa
ACTI VI DADES
3
Estatuto de Autonoma de Catalua
ARTCULO 3.
1) La lengua propia de Catalua es el cataln.
2) El idioma cataln es el oficial de Catalua, as
como tambin lo es el castellano, oficial en todo el
Estado espaol.
3) La Generalidad garantizar el uso normal y oficial
de los dos idiomas, adoptar las medidas necesarias
para asegurar su conocimiento y crear las condi-
ciones que permitan alcanzar su plena igualdad en
lo que se refiere a los derechos y deberes de los ciu-
dadanos de Catalua.
Estatuto de Autonoma del Pas Vasco
ARTCULO 6.
1) El euskera, lengua propia del pueblo vasco, ten-
dr, como el castellano, carcter de lengua oficial en
Euskadi; todos sus habitantes tienen el derecho de
conocer y usar ambas lenguas.
2) Las instituciones comunes de la Comunidad Au-
tnoma, teniendo en cuenta la diversidad sociolin-
gstica del Pas Vasco, garantizarn el uso de ambas
lenguas, regulando su carcter oficial y arbitrarn y
regularn las medidas y medios necesarios para ase-
gurar su conocimiento.
3) Nadie podr ser discriminado por razn de la
lengua.
4) La Academia de la Lengua Vasca, Euskaltzaindia,
es la institucin consultiva oficial en lo que respecta
al euskera.
Estatuto de Autonoma de las Illes Balears
ARTCULO 3.
La lengua catalana, propia de las Illes Balears, tendr,
junto con la castellana, el carcter de idioma oficial,
y todos tienen el derecho de conocerla y utilizarla.
Estatuto de Autonoma de Galicia
ARTCULO 5.
1) La lengua propia de Galicia es el gallego. 2) Los idiomas gallego y castellano son oficiales en
Galicia y todos tienen el derecho a conocerlos y
usarlos.
3) Los poderes pblicos de Galicia garantizarn el
uso normal y oficial de los dos idiomas y potencia-
rn la utilizacin del gallego en todos los rdenes
de la vida pblica, cultural e informativa, y dispon-
drn los medios necesarios para su conocimiento.
4) Nadie podr ser discriminado por razn de la
lengua.
Ley orgnica del rgimen Foral de Navarra
ARTCULO 9.
1) El castellano es la lengua oficial de Navarra.
2) El vascuence tendr, tambin, carcter de lengua
oficial en las zonas vascoparlantes de Navarra.
Estatuto de Autonoma de la Comunitat
Valenciana
ARTCULO 7.
1) Los dos idiomas oficiales de la Comunidad Aut-
noma son el valenciano y el castellano. Todos tienen
derecho a conocerlos y usarlos.
2) La Generalitat Valenciana garantizar el uso
normal y oficial de las dos lenguas y adoptar las me-
didas necesarias para asegurar su conocimiento.
3) Nadie podr ser discriminado por razn de su
lengua.
4) Se otorgar especial proteccin y respeto a la recu -
peracin del valenciano.
M
A
T
E
R
I
A
L

F
O
T
O
C
O
P
I
A
B
L
E

/


O
x
f
o
r
d

U
n
i
v
e
r
s
i
t
y

P
r
e
s
s

E
s
p
a

a
,
S
.
A
.
0B1LLLP(2008).03+s 3/6/08 19:11 Pgina 49
DOCUMENTACIN
2. Leyes de normalizacin
lingstica
Todas las comunidades autnomas con lengua propia han promulgado leyes que desa rrollan
los artculos de los estatutos referidos a las lenguas. A continuacin, citamos algunos artcu los
de esas leyes.
50 3. La realidad plurilinge de Espaa
M
A
T
E
R
I
A
L

F
O
T
O
C
O
P
I
A
B
L
E

/


O
x
f
o
r
d

U
n
i
v
e
r
s
i
t
y

P
r
e
s
s

E
s
p
a

a
,
S
.
A
.
ACTI VI DADES
3
Ley de normalizacin lingstica de Galicia (1983)
TTULO I
Artculo 1.
El gallego es la lengua propia de Galicia. Todos los gallegos tienen el deber de
conocerlo y el derecho de usarlo.
Artculo 2.
Los poderes de Galicia garantizarn el uso normal del gallego y del castellano,
lenguas oficiales de la Comunidad Autnoma.
Artculo 3.
Los poderes pblicos de Galicia adoptarn las medidas oportunas para que
nadie sea discriminado por razn de lengua.
Los ciudadanos podrn dirigirse a los jueces y tribunales para obtener la pro-
teccin judicial del derecho a emplear la lengua.
TTULO II
Artculo 4.
1) El gallego, como lengua propia de Galicia, es lengua oficial de las instituciones
de la Comunidad Autnoma, de su Administracin, de la Administracin local
y de las entidades pblicas dependientes de la Comunidad Autnoma.
TTULO III
Artculo 12.
1) El gallego, como lengua propia de Galicia, es tambin lengua oficial en la
enseanza en todos los niveles educativos.
Artculo 13.
1) Los nios tienen derecho a recibir la primera enseanza en su lengua materna.
2) Las autoridades educativas de la Comunidad Autnoma arbitrarn las medidas
encaminadas a promover el uso progresivo del gallego en la enseanza.
Artculo 14.
1) La lengua gallega es materia de estudio obligatorio en todos los niveles edu-
cativos no universitarios.
TTULO IV
Artculo 18.
El gallego ser la lengua usual en las emisoras de radio y televisin y en los
dems medios de comunicacin social sometidos a gestin o competencia de
las instituciones de la Comunidad Autnoma.
Ley de normalizacin lingstica
de las Illes Balears (1986)
TTULO PRELIMINAR
Artculo 1.
La presente ley tiene por objeto desarrollar el artculo 3.
o
del Estatuto de Autonoma en lo que respecta a la normalizacin de la lengua catalana como propia de las
Illes Balears en todos los mbitos y garantizar el uso del cataln y del castellano como idiomas oficiales de la Comunidad Autnoma.
Ley de normalizacin lingstica
de Catalua (1983)
TTULO PRELIMINAR
Artculo 1.
1) La presente ley tiene por objeto el desarrollo del artculo 3.
o
del
Estatuto de Autonoma de Catalua para llevar a cabo la normali -
zacin del uso de la lengua catalana en todos los mbitos y garantizar
el uso normal y oficial del cataln y el castellano.
2) Dada la situacin lingstica de Catalua, son, pues, objetivos de
esta ley:
a) Amparar y fomentar el uso del cataln por parte de todos los
ciuda danos.
b) Dar efectividad al uso oficial del cataln.
c) Normalizar el uso del cataln en todos los medios de comuni-
cacin social.
d) Asegurar la extensin del conocimiento del cataln.
TTULO I
Artculo 5.
1) El cataln, como lengua propia de Catalua, lo es tambin de la
Generalidad y de la Administracin territorial catalana, de la Admi-
nistracin local y de las dems corporaciones pblicas dependientes
de la Generalidad.
TTULO II
Los nios tienen derecho a recibir la primera enseanza en su lengua
habitual, ya sea esta el cataln o el castellano. La Administracin
debe garantizar este derecho y poner los medios necesarios para
hacerlo efectivo. []
3) La lengua catalana y la lengua castellana deben ser enseadas
obligatoriamente en todos los niveles y grados de la enseanza no
universitaria.
4) Todos los nios de Catalua, cualquiera que sea su lengua habitual
al iniciar la enseanza, deben poder utilizar normal y correctamente
el cataln y el castellano al final de sus estudios bsicos.
TTULO III
Artculo 21.
2) El Consejo Ejecutivo de la Generalidad debe reglamentar la nor-
malizacin del uso de la lengua en los medios de comunicacin
social some tidos a la competencia o gestin de la Generalidad, con el
objetivo de asegurar la comprensin y mejorar el conocimiento de la
lengua catalana teniendo en cuenta la situacin lingstica de cada
medio en concreto.
0B1LLLP(2008).03+s 3/6/08 19:11 Pgina 50
51 3. La realidad plurilinge de Espaa
D
O
C
U
M
E
N
T
A
C
I

N
2
.

L
e
y
e
s

d
e

n
o
r
m
a
l
i
z
a
c
i

n

l
i
n
g

s
t
i
c
a
M
A
T
E
R
I
A
L

F
O
T
O
C
O
P
I
A
B
L
E

/


O
x
f
o
r
d

U
n
i
v
e
r
s
i
t
y

P
r
e
s
s

E
s
p
a

a
,
S
.
A
.
A
C
T
I
V
I
D
A
D
E
S
3
Ley sobre el uso y la enseanza del valenciano (1983)
TTULO PRELIMINAR
Artculo 1.
1) La presente ley tiene por objeto genrico dar cumplimiento y desarrollar lo dispuesto en el artculo 7.

del Estatuto de
Autonoma, regulando el uso normal y oficial del valenciano en todos los mbitos de la convivencia social, as como su
enseanza.
2) En base a ello son objetivos especficos de la presente ley los siguientes:
a) Hacer efectivo el derecho de todos los ciudadanos a conocer y usar el valenciano.
b) Proteger su recuperacin y garantizar su uso normal y oficial.
c) Regular los criterios de aplicacin del valenciano en la Administracin, los medios de comunicacin social y la ense-
anza.
d) Delimitar los territorios en los que predomine el uso del valenciano y del castellano.
e) Garantizar, con arreglo a principios de graduabilidad y voluntariedad, el conocimiento y uso del valenciano en todo
el mbito de la Comunidad.
TTULO I
Artculo 7.
1) El valenciano, como lengua propia de la Comunidad Valenciana, lo es tambin de la Generalidad y de su Administracin
pblica, de la Administracin local y de cuantas corporaciones e instituciones pblicas dependan de aquellas.
TTULO II
Artculo 18.
1) La incorporacin del valenciano a la enseanza en todos los niveles educativos es obligatoria. []
3) El valenciano y el castellano son lenguas obligatorias en los planes de enseanza de los niveles no universitarios. []
Artculo 19.
2) [] Al final de los ciclos en que se declara obligatoria la incorporacin del valenciano a la enseanza y cualquiera que
hubiera sido su lengua habitual al iniciar los mismos, los alumnos han de estar capacitados para utilizar, oralmente y por
escrito, el valenciano en igualdad con el castellano.
TTULO III
Artculo 25.
1) El Consejo de la Generalidad Valenciana velar para que el valenciano tenga una adecuada presencia en aquellas emi-
soras de radio y televisin y dems medios de comunicacin gestionados por la Generalidad Valenciana, o sobre los que la
misma ley tenga competencia, de acuerdo con lo dispuesto en la presente ley.
Ley bsica de normalizacin del uso del euskera (1982)
TTULO PRELIMINAR
Artculo 2.
La lengua propia del Pas Vasco es el euskera.
Artculo 3.
Las lenguas oficiales en la Comunidad Autnoma del Pas Vasco son el euskera y el castellano.
TTULO II
Artculo 6.
1) Se reconoce a todos los ciudadanos el derecho a usar tanto el euskera como el castellano en sus
relaciones con la Administracin pblica en el mbito territorial de la Comunidad Autnoma, y a ser
atendidos en la lengua oficial que elijan.
Artculo 15.
Se reconoce a todo alumno el derecho a recibir la enseanza tanto en euskera como en castellano en
los diversos niveles educativos.
A tal efecto, el Parlamento y el Gobierno adoptarn las medidas oportunas tendentes a la generaliza-
cin progresiva del bilingismo en el sistema educativo de la Comunidad Autnoma del Pas Vasco.
Artculo 17.
El Gobierno adoptar aquellas medidas encaminadas a garantizar al alumnado la posibilidad real, en
igualdad de condiciones, de poseer un conocimiento prctico suficiente de ambas lenguas oficiales al
finalizar los estudios de enseanza obligatoria y asegurar el uso ambiental del euskera, haciendo
del mismo vehculo de expresin normal, tanto en las actividades internas como externas y en las
actuaciones y documentos administrativos.
Ley foral del vascuence
en Navarra (1986)
TTULO PRELIMINAR
Artculo 1.
2) Son objetivos esenciales:
a) Amparar el derecho de los ciudadanos
a conservar y usar el vascuence y de -
finir los instrumentos para hacerlo
efectivo.
b) Proteger la recuperacin y el desarrollo
del vascuence en Navarra, sealando
las medidas para el fomento de su
uso.
c) Garantizar el uso y la enseanza del
vascuence con arreglo a principios de
voluntariedad, gradualidad y respeto,
de acuerdo con la realidad sociolin-
gstica de Navarra.
3) Las variedades dialectales del vascuence
en Navarra sern objeto de especial respeto
y proteccin.
FUENTE: Maitena ETXEBARRA ARSTEGUI, El bilingismo
en el Estado espaol, FBV.
0B1LLLP(2008).03+s 3/6/08 19:11 Pgina 51
AMPLIACIN
3. Lenguas en contacto
52 3. La realidad plurilinge de Espaa
M
A
T
E
R
I
A
L

F
O
T
O
C
O
P
I
A
B
L
E

/


O
x
f
o
r
d

U
n
i
v
e
r
s
i
t
y

P
r
e
s
s

E
s
p
a

a
,
S
.
A
.
ACTI VI DADES
3
Los siguientes textos tratan consecuencias del contacto de lenguas en las comunidades vasca
y catalana.
Texto 1
En los ltimos aos se detecta un aumento
del inters por la calidad lingstica de la pro-
duccin en euskera. Euskaltzaindia (1994) ha
mostrado su preocupacin porque se est pro-
duciendo en los medios de comunicacin y
entre los hablantes ms jvenes una ruptura
con la tradicin de proporciones notables.
Tambin en el mbito de la institucin esco-
lar existen voces que sealan que la lengua est
sufriendo un desgaste considerable, sobre todo
debido a la introduccin masiva del euskera en
mbitos castellano-hablantes, a travs de pro-
gramas de inmersin. En opinin de algunos, se
ha centrado ms la atencin en la produccin en
euskera que en la calidad de la misma. [] Los
usos incorrectos son debidos en gran parte, sin
duda, a las interferencias y calcos provocados
por la lengua dominante respectiva (castellano
o francs), que afectan a todos los niveles de la
lengua (fontico-fonolgico, morfo sintctico y
lxico-semntico).
En cuanto a la forma que est adquiriendo
la lengua estndar en los ltimos aos, el aca-
dmico Ibon Sarasola ha sealado que se est
produciendo un divorcio preocupante entre las
variedades del Sur de los Pirineos y las del Nor-
te. Sarasola, que trabaja sobre todo en el mbi-
to del lxico, apunta que existe el riesgo de que
el vasco quede reducido a la condicin de len-
gua criolla del castellano, y propone tender al
equilibrio entre las variedades peninsulares y
las continentales.
En el campo de la sintaxis tambin hay
varias propuestas para el desarrollo de la
variedad estndar, observndose en todas
ellas una preocupacin comn: la de que el
euskera no se convierta en una traduccin del
castellano ms torpe (debate que afecta de
lleno a los medios de comunicacin).
En resumen, cabe sealar que en los ltimos
aos est fortalecindose, desde diferentes sec-
tores, el inters por aumentar no solo la canti-
dad de produccin en euskera, sino tambin la
calidad de la misma; ello significa un intento de
mantener una relacin ms estrecha con los
usos tradicionales de la lengua, y la bsqueda
de autonoma en su de sarrollo con respecto,
sobre todo, al castellano.
Julin MAIA LARRETXEA
La lengua euskera en la enseanza
Textos de didctica de la lengua y de la literatura, n. 18
Texto 2
En el interior de la sociedad, las interferencias, los prstamos y la
alternancia en el uso del cataln y el castellano constituyen un hecho.
Se puede constatar en el cataln hablado, condicionado secularmen-
te por las estructuras del castellano a niveles cultos, ms moderna-
mente por el peso de muchos aos de escolarizacin exclusiva en cas-
tellano, y ms recientemente por la influencia del castellano coloquial
de importantes sectores de poblacin procedentes de la inmigracin.
Lo prueba la abundancia, entre otras, de obras de divulgacin sobre
el uso de un cataln correcto []; todo ello con una tradicin
(obsesin) que se re monta ya a las primeras pocas de la moderna
normativizacin.
La competencia oral en castellano de los catalanohablantes es sufi-
ciente para permitirles, por ejemplo, cuando narran en cataln, repro-
ducir textualmente intervenciones de terceros en lengua castellana.
El cataln hablado en todo el territorio delata a la otra lengua en
contacto, ms o menos segn cada hablante: pues (doncs), bue-
no (b), bandeja (safata), relleno (farcit), tiburn (taur), oja-
l (tant de bo) Pero hay que decir que los prstamos del cataln al
castellano estn vivos y arraigados incluso entre monolinges caste-
llanohablantes: paleta (albail), tornavs (destornillador), enche-
gar (poner en marcha), rachola (baldosa), enracholar (alicatar),
enracholador (alicatador), collar (apretar), no cal (no es necesa-
rio), plegar (dar de mano), baldufa (peonza), pencar (currar),
charrar (charlar), colla (grupo, pandilla), no me veo (no veo),
adeu (adis) Se trata de catalanismos a menudo difciles de susti-
tuir en los registros coloquiales del castellano hablado en los territo-
rios de lengua catalana.
Estos fenmenos, producidos de manera absolutamente indiscri-
minada y espontnea, tienen ahora un regulador en el sistema edu-
cativo de las tres comunidades, al cual, por primera vez en amplias
zonas, o de forma mucho ms generalizada en otras, le compete la
informacin y la formacin en ambas lenguas.
Luis LPEZ DEL CASTILLO
La lengua catalana en la enseanza
Textos de didctica de la lengua y de la literatura, n. 18
Cul es la mayor preocupacin respecto al uso
del euskera? A qu factor social se debe esta situacin?
Indica las causas de los usos incorrectos.
Menciona las ideas principales que se desarrollan
en la conclusin del texto de Julin Maia Larretxea.
Qu fenomenos de contacto de lenguas son habituales
en el uso del cataln y el castellano? Explica en qu
consisten.
Indica a qu registro de lengua del cataln afectan
principalmente y cules son los factores histricos
y sociales que han favorecido su desarrollo.
Cita ejemplos de interferencias lxicas en ambas lenguas.
Explica la conclusin a la que llega el autor del texto 2. 7
6
5
4
3
2
1
0B1LLLP(2008).03+s 3/6/08 19:11 Pgina 52
ORTOGRAFA
4. Uso de ll e y
El dgrafo ll representa el fonema lateral palatal /l

/.
Se escriben con ll:
Las palabras de uso general terminadas en -illo, -illa: chiquillo, bombilla, campanilla.
La mayor parte de los verbos terminados en -illar, -ullar y -ullir: acuchillar, apabullar, bullir.
La letra y puede representar dos fonemas distintos: uno equivalente al representado por la
letra i en palabras como estoy, voy; y otro consonntico, el fonema lateral palatal sonoro, en
voces como bueyes, cuyo.
Se escriben con y:
Las palabras que terminan con el sonido correspondiente a /i / precedido de una vocal con
la que forma diptongo, o dos con las que forma triptongo: soy, rey, Paraguay.
Las palabras con sufijos ad-, dis- y sub-: adyacente, disyuntiva, subyacente.
Formas verbales de caer, recaer, leer, creer, poseer, proveer, sobreseer; y de los verbos acabados
en -or y -uir: cay, ley, creyeron, oyendo, huy, contribuyeron.
Las palabras con la slaba -yec-: proyectaron, inyectar.
Los plurales de los sustantivos terminados en -y: ley, leyes.
El gerundio del verbo ir: yendo
En algunas zonas de habla castellana, se produce el fenmeno del yesmo, es decir, la pronun-
ciacin como /y/ de palabras con ll y con y: vaya y valla se pronuncian igual, lo cual conduce a
numerosas incorrecciones grficas.
Completa con ll o y las palabras siguientes:
apo

aron foeto
jeringui

a pro__ecto
ampo

a panta__a
re__enar o__ente
subra__ar arro__uelo
atribu__ enro__arse
cn__uge ro__o
in__eccin desfa__ecer
desma__o ca__eron
desarro__aron arro__o
pi__aste ensa__
ha__arn influ__ente
a__udaron pe__orativo
fo__etn ve__o
be__o exclu__
disminu__a ta__aron
fo__n o__a
Escribe halla o haya, segn corresponda:
a) Cuando alguna novedad,
te llamaremos.
b) la solucin a estos problemas.
c) Quien te lo dicho, te ha mentido.
d) No se a gusto en esta ciudad.
e) No creo que llegado nadie an.
Escribe los gerundios de los siguientes verbos.
ir or huir
influir destruir construir
incluir caer leer
atraer extraer retraer
contraer proveer atribuir
retribuir concluir excluir
Conjuga los siguientes verbos en tercera
persona del singular y del plural del pretrito
perfecto simple de indicativo y del presente
del subjuntivo: disminuir, influir, constituir,
obstruir, diluir.
Completa las siguientes oraciones con las
formas verbales correspondientes:
a) Ten cuidado, se est el lquido!
(Caer)
b) No estoy seguro de que maana;
estamos muy cansados. (Ir)
c) No se sabe nada del joven que ayer
de su casa. (Huir)
d) Silencio! No estis lo que os dice.
(Or)
e) El mes pasado los accidentes de
trfico. (Disminuir)
f) Ayer todos las actividades
programadas. (Concluir)
5
4
3
2
1
53 3. La realidad plurilinge de Espaa
M
A
T
E
R
I
A
L

F
O
T
O
C
O
P
I
A
B
L
E

/


O
x
f
o
r
d

U
n
i
v
e
r
s
i
t
y

P
r
e
s
s

E
s
p
a

a
,
S
.
A
.
ACTI VI DADES
3
0B1LLLP(2008).03+s 3/6/08 19:11 Pgina 53
Evaluacin 3
55 3. La realidad plurilinge de Espaa
1. Contesta las siguientes cuestiones:
Qu informacin da el autor sobre el origen y for-
macin de la lengua espaola?
Con qu lengua actual se identifica la antigua que
se usava en Espaa antes que los romanos viniessen?
Qu datos aporta sobre ella?
2. Resume los hechos histricos que han originado la
diversidad lingstica de la Espaa actual.
3. El texto de Juan de Valds, escrito en el siglo XVI, repro-
duce de modo aproximado caractersticas fnicas del
castellano de la poca. Cita algunos ejemplos.
4. Qu establece la Constitucin de 1978 respecto de la
situacin lingstica de Espaa?
5. Nombra las lenguas oficiales de Espaa, en qu comuni-
dades tienen ese carcter y sus variedades dialectales.
6. Cules son los llamados dialectos histricos? Indica
sus caractersticas ms destacadas.
7. Explica los objetivos de las leyes de normalizacin lin-
gstica y seala los mbitos a los que se refieren.
8. Qu fenmenos de las lenguas en contacto pueden
darse en las comunidades bilinges? En qu consisten?
9. Realiza un esquema de las variedades geogrficas del
espaol con sus caractersticas y las relaciones que
existen entre algunas de ellas.
10. Redacta un texto expositivo sobre la importancia del
respeto a la pluralidad lingstica y cultural de Espaa.
La lengua que oy se habla en Castilla, de la qual vosotros queris
ser informados tiene parte de la lengua que se usava en Espaa
antes que los romanos la enseoreassen, y tiene tambin alguna
parte de la de los godos, que sucedieron a los romanos, y mucha
de la de los moros, que reinaron muchos aos, aunque la principal
parte es de la lengua que introduxeron los romanos, que es la lengua
latina, ser bien que primero esaminemos qu lengua era aquella
antigua que se usava en Espaa antes que los romanos viniessen
a ella. Lo que por la mayor parte los que son curiosos destas cosas
tienen y creen es que la lengua que oy usan los vizcanos es aquella
antigua espaola. Esta opinin confirman con dos razones harto
aparentes. La una es que, ass como las armas de los romanos
quando conquistaron la Espaa no pudieron passar en aquella
parte que llamamos Vizcaya, ass tampoco pudo pasar la lengua
al tiempo que, despus de averse hecho seores de Spaa, quisieron
que en toda ella se hablasse la lengua romana. La otra es la dis-
conformidad que tiene la lengua vizcana con cualquiera de todas
las otras lenguas que el da de oy en Espaa se usan.
Juan DE VALDS
Dilogo de la lengua, Castalia
La lengua de los vizcanos
Danzas en Egaray, por Martnez Ortiz.
0B1LLLA.03 31/3/08 21:47 Pgina 55
REFUERZO
1. El anlisis fnico
El nivel fnico de estudio de la lengua se ocupa de las unidades relacionadas con el aspecto
articulatorio y auditivo, distinguindose dos disciplinas, la fontica y la fonologa. Lee el
siguiente texto y responde las cuestiones que se te plantean a continuacin.
El seor prisionero
SR. TEPN.No vaya usted ahora a andar con vergenzas con nosotros. Si quiere que le
soltemos las ligaduras, dganoslo.
SRA. TEPN.Usted pngase lo ms cmodo que pueda.
ZEPO.Bueno, si se ponen as, sultenme las ligaduras. Pero solo se lo digo por darles el
gusto.
SR. TEPN.Hijo, qutaselas. (ZAPO le quita las ligaduras de los pies.)
SRA. TEPN.Qu, se encuentra usted mejor?
ZEPO.S, sin duda. A lo mejor les estoy molestando mucho.
SR. TEPN.Nada de molestarnos. Usted, considrese como en su casa. Y si quiere que le
soltemos las manos, no tiene ms que pedrnoslo.
ZEPO.No. Las manos, no. Es pedir demasiado
SR. TEPN.Que no, hombre, que no. Ya le digo que no nos molesta en absoluto.
ZEPO.Bueno entonces, destenme las manos. Pero solo para comer, eh?, que no
quiero yo que me digan luego que me ofrecen el dedo y me tomo la mano entera.
SR. TEPN.Nio, qutale las ligaduras de las manos.
SRA. TEPN.Qu bien, con lo simptico que es el seor prisionero, vamos a pasar un
buen da de campo.
ZEPO.No tiene usted que decirme seor prisionero, diga prisionero a secas.
SRA. TEPN.No le va a molestar?
ZEPO.No, en absoluto.
SR. TEPN.Desde luego hay que reconocer que es usted modesto.
Fernando ARRABAL
Pic-Nic. El triciclo. El laberinto, Ctedra
Caracteriza la situacin planteada en este
fragmento de la obra teatral de Fernando
Arrabal. Argumenta tu respuesta.
La seora Tepn formula dos preguntas a Zepo.
Cmo es su entonacin? Compralas con
otros enunciados del fragmento.
Realiza la transcripcin fonolgica de las dos
interrogaciones que la seora Tepn plantea
al prisionero.
Caracteriza los fonemas que intervienen
en ambas teniendo en cuenta sus rasgos
mnimos.
Seala el o los rasgos que diferencian los
siguientes pares de fonemas.
/i/, /u/ /i/, /e/ /i/, /a/ /p/, /b/ /p/, /t/
/p/, /f/ /r/, /l/ /b/, /m/ /m/, /n/ /s/, /x/
Explica qu rasgos se tienen en cuenta para
definir los fonemas voclicos. Y para definir
los fonemas consonnticos?
Qu diferencia los fonemas de los sonidos
lingsticos? Qu disciplina lingstica se
ocupa de unos y de otros?
Los fonemas voclicos funcionan como centro
silbico. Busca en el texto ejemplos de slabas
abiertas y cerradas.
Extrae del texto distintos tipos de diptongos
y de hiatos.
Explica las relaciones posibles entre fonemas
y grafas atendiendo a las siguientes palabras
del texto. Incorpora otros ejemplos para
completar tu explicacin.
ahora vergenzas buen
cmodo quiere ofrecen
Localiza en el texto ejemplos del texto
de distintos tipos de palabras acentuadas
e inacentuadas.
Extrae ahora ejemplos de palabras oxtonas,
paroxtonas y proparoxtonas.
12
11
10
9
8
7
6
5
4
3
2
1
61 4. Fontica y fonologa
M
A
T
E
R
I
A
L

F
O
T
O
C
O
P
I
A
B
L
E

/


O
x
f
o
r
d

U
n
i
v
e
r
s
i
t
y

P
r
e
s
s

E
s
p
a

a
,
S
.
A
.
ACTI VI DADES
4
0B1LLLP(2008).04+s 3/6/08 19:14 Pgina 61
ORTOGRAFA
2. Uso de la h
En la lengua castellana existe una letra que no representa ningn fonema, la h. Esta grafa puede
aparecer al principio de palabra (hora), en su interior (ahora) y, solo en algunas interjecciones, al final
(ah!, oh!, eh!, uh!, bah!).
Debes recordar que se escriben con h:
Las formas de los verbos haber, hacer, hallar, hablar y habitar: hemos, habis, hacan, he hecho,
hallamos, hallarn, habl, hablaba, habitaste, habitaremos. Del mismo modo, tambin llevan h las
palabras derivadas de algunos de dichos verbos: deshacer, hallazgo, hablador, habitacin.
Las palabras que comienzan por los diptongos ia, ie, ue y ui: hiato, hierba, huelga, huida. Si el dip-
tongo ue est precedido de vocal, se escribe h intercalada: cacahuete.
Las palabras que comienzan con las races clsicas hecto-, helio-, hema- / hemato- / hemo-, hemi,
hepta, hetero-, hidra- / hidro-, higro-, hiper-, hipo-, holo-, homeo-, homo-: hectogramo, heliotera-
pia, hemate, hematfago, hemofilia, hemiplejia, heptaedro, heterogneo, hidratacin, hidroavin,
higrometra, hiperglucemia, hiponimia, holocausto, homeotermia, homogneo.
Por regla general, las palabras que empiezan por histo-, hosp-, hum-, horm-, herm-, hern-, holg-
y hog-: historia, hospitalario, hmedo, hormign, hermoso, hernia, holgazn, hoguera.
Es necesario, adems, que tengas en cuenta palabras homfonas con h y sin h: ha / a / ah; a ver /
haber; habra / abra; asta / hasta; aya / haya / halla; desecho / deshecho; echo / hecho; errar / herrar;
ojear / hojear; ola / hola; onda / honda; ora / hora; reusar / rehusar, as como palabras parnimas
como hay / ah / ay.
Explica el significado de las races clsicas
citadas en la informacin superior, as como
el de los ejemplos dados.
hecto-: cien veces.
Construye oraciones con las palabras
homfonas y parnimas mencionadas
en la informacin superior.
Indica si estn correctamente escritas
las siguientes oraciones. Si no son correctas,
seala cul es el error en cada una de ellas.
a) Ah ay un error: lo descubr al ojear la
revista.
b) Juan se ha hechado novia; haber cundo
haces lo mismo.
c) Hallad la superficie de esta figura; cuando
ayis acabado, levantad la mano.
d) Ojal haya fresas en el mercado! Nunca
las allo cuando las busco.
e) Marcos se hech a rer cuando le dije que
no lo haba hecho.
f) Cuando habri la puerta se encontr con
Mara; si lo hubiera sabido no abra vuelto.
g) ltimamente no ha habido lluvias
en esa regin; va a haber que acudir
al riego artificial.
h) Hasta el da jueves la bandera debe hondear
a media hasta.
Incorpora la h cuando sea necesario en estas
palabras.
alel coete inchado
armona aumar exagonal
oquedad ovario alcool
exibicin exortar exaltar
exaustivo veemencia desauciar
exuberante ueco aondar
ilandera orror alcauete
eniesto coercitivo anelar
ovalado uelo olemos
urfano orfandad ampa
osamenta urtar idrulico
idroterapia reabilitacin ipcrita
eterosexual trasumante aijado
iguera ren baa
emos echo desaogar ematoma
coartar almoada toalla
inumacin echar de menos undir
coaccin eliocntrico urao
seo oler ule
ilacin ediondo oloroso
vado omgrafo incapi
transente inspito error
almoada eptgono mero
retala trasumancia ueso
4
3
2
1
62 4. Fontica y fonologa
M
A
T
E
R
I
A
L

F
O
T
O
C
O
P
I
A
B
L
E

/


O
x
f
o
r
d

U
n
i
v
e
r
s
i
t
y

P
r
e
s
s

E
s
p
a

a
,
S
.
A
.
ACTI VI DADES
4
0B1LLLP(2008).04+s 3/6/08 19:14 Pgina 62
Evaluacin
4
67 4. Fontica y fonologa
1. Resume el contenido de este fragmento de Eduardo
Mendoza. A qu tipo de texto corresponde segn su
intencin comunicativa?
2. Reconoce en el texto los elementos del proceso comu-
nicativo y seala las funciones del lenguaje presentes en
los enunciados.
3. Transcribe fonolgicamente el enunciado del recuadro,
incluyendo los acentos y la entonacin.
4. Enumera los ragos distintivos de cada uno de los fonemas
que intervienen en tu transcripcin.
5. Si el tonema de la actividad 3 fuera el contrario, qu
cambio se producira en el enunciado?
6. Reconoce los grupos fnicos de este otro enunciado
y adjudica a cada uno su tonema correspondiente.
7. Explica la relacin que se establece entre el fonema /x/
y sus grafas, ejemplificando con palabras del texto.
8. Qu rasgos distintivos comparten los siguientes pares
de fonemas? Por cules se oponen?
/e/, /o/ /n/, /c/ /r/, /r/
9. Localiza en el fragmento casos de palabras con hiatos
y diptongos (crecientes, decrecientes y mixtos).
10. Redacta un texto explicativo sobre la produccin de los
sonidos lingsticos. Cuida la ortografa y la presentacin.
As que usted es francs, eh? insisti la seora.
En efecto. Soy de Pars.
Nadie lo dira, oyndole hablar. Su castellano es perfecto.
Dnde lo aprendi?
Mi madre era espaola. Siempre me habl en espaol, de
modo que puede decirse que aprend el espaol desde la cuna,
incluso antes que el francs.
Qu bien, verdad? A m me gustan los extranjeros. Son
muy interesantes, cuentan cosas nuevas y distintas de las que
omos cada da. Nosotros siempre estamos hablando de lo mis-
mo. Y es natural, digo yo, eh? Vivimos en el mismo lugar, vemos
a la misma gente y leemos los mismos peridicos. Por eso debe
de ser que discutimos siempre: por no tener nada de qu
hablar. En cambio con los extranjeros no hace falta discutir:
ellos cuentan sus cosas y nosotros las nuestras. Yo me llevo mejor
con los extranjeros que con los de aqu.
Estoy seguro de que usted se lleva bien con todo el mundo.
Ca, no lo crea. Soy muy gruona. Con los aos el carcter
tambin se deteriora. Todo va de baja. Pero, hablando de extranje-
ros, dgame una cosa, conoci usted al ingeniero Pearson?
Fred Stark Pearson? No, no le conoca, aunque o hablar de
l con gran frecuencia.
Era una gran persona, ya lo creo! Muy amigo de mi difunto
esposo, que en gloria est.
Eduardo MENDOZA
La verdad sobre el caso Savolta, Seix Barral
Sobre el ingeniero Pearson
Conoci usted al ingeniero?
Vivimos en el mismo lugar, vemos a la misma gente
y leemos los mismos peridicos.
Pareja, por Bou.
0B1LLLA.04 31/3/08 21:53 Pgina 67
REFUERZO
1. Morfologa flexiva
La morfologa flexiva se ocupa de los accidentes gramaticales de las pala-
bras variables (sustantivos, adjetivos y verbos). Repasa los conceptos de
tema y de morfemas flexivos (genricos y numricos; vocales temticas,
caractersticas y desinencias), y despus contesta las cuestiones que se
plantean sobre el texto.
Extrae todos los sustantivos comunes y los
adjetivos calificativos del texto.
Seala en ambas clases de palabras sus temas
y los morfemas flexivos genricos y numricos
que presenten. Indica si los temas son simples
o complejos.
Reconoce entre los sustantivos que has
sealado en la actividad 1, uno que sea comn
en cuanto al gnero.
Escribe ejemplos de sustantivos epicenos,
de sustantivos ambiguos y de sustantivos
heternimos.
Escribe ahora las formas verbales no
personales que encuentres en el texto,
diferenciando su base lxica y sus morfemas
flexivos correspondientes (vocal temtica
y caracterstica).
Extrae ahora las formas verbales personales
simples, agrupndolas segn el tiempo verbal
al que pertenezcan.
Atendiendo a su base lxica, di cules
corresponden a verbos regulares e irregulares.
En el caso de los irregulares, seala los
distintos alomorfos del verbo y explica
los tipos de variaciones que presenten.
Indica los morfemas flexivos (vocal temtica,
caracterstica y desinencia) de todas las formas
verbales del fragmento.
Qu desinencias no estn presentes
en las formas verbales del texto? Brinda
ejemplos de ellas conjugando verbos
que figuren en tu lista, y selalas.
Reconoces algn verbo polirrizo en el
fragmento? En caso afirmativo, seala sus
distintas races.
10
9
8
7
6
5
4
3
2
1
73 5. Morfologa
M
A
T
E
R
I
A
L

F
O
T
O
C
O
P
I
A
B
L
E

/


O
x
f
o
r
d

U
n
i
v
e
r
s
i
t
y

P
r
e
s
s

E
s
p
a

a
,
S
.
A
.
ACTI VI DADES
5
El reloj
Cuando Curtis va por el Cantn, ve ya con toda claridad las nubes cenicientas, espesas, terrosas,
como el vaho del rescoldo del fuego rumiante, que forman las humaredas de la Drsena. Cubierto
tambin el cielo hacia la parte de Mara Pita. Sabe que ya no podr dar marcha atrs. Tiene que
seguir adelante para verlo con sus propios ojos.
Mira el reloj en lo alto del Obelisco. Recuerda: Su majestad, la Hora! Pero parece que lleva ah
desde siempre, que las agujas no han completado an la vuelta a la esfera, que marca siglos. Sada
tena razn. Debera ser un reloj de cuco. Si ahora saliese un cuco, pens Cutis, quiz todo sera
distinto. Alzara la frente de los que caminan inquietos, contando interrogantes en las lozas del
suelo como quien pone sus pasos en cuadrculas de ajedrez.Tal vez descompondra el andar marcial
de otros, que trazan una lnea recta.
Quiz el cuco detendra por un instante al joven uniformado con ese gorro de cresta colorada
que parece galopar sobre las ancas de una lnea recta.
Tuvo la impresin de verlo a l, a Sada, cuando pas por la plaza de Pontevedra, y all donde
estaban empezando a formar las tropas de reclutamiento. El ejrcito golpista se haba
impuesto en la ciudad y dominaba Galicia, que iba a ser uno de los territorios de retaguardia
para lo que los sublevados denominaban la nueva reconquista de Espaa. S, le pareci que
Sada estaba all. Tan alto, le resultaba difcil pasar desapercibido. Le pareci tambin que haba
otros rostros conocidos, aunque en aquellos das no solo haba cambiado el humor, sino el rostro
de las gentes, su presencia, sus rasgos fsicos.
Manuel RIVAS
Los libros arden mal, Punto de Lectura
0B1LLLP(2008).05+s 3/6/08 19:19 Pgina 73
DOCUMENTACIN
2. Modelo de constituyentes
Los siguientes cuadros recogen las formas simples de los distintos temas verbales. En cada
una se ha destacado la vocal temtica en negrita, la caracterstica en cursiva y la desinencia
subrayada; el resto es el lexema o raz verbal.
74 5. Morfologa
M
A
T
E
R
I
A
L

F
O
T
O
C
O
P
I
A
B
L
E

/


O
x
f
o
r
d

U
n
i
v
e
r
s
i
t
y

P
r
e
s
s

E
s
p
a

a
,
S
.
A
.
ACTI VI DADES
5
TEMA DE PRESENTE
Presente de indicativo Presente de subjuntivo
1. conj. 2. conj. 3. conj. 1. conj. 2. conj. 3. conj.
estudi-o tem-o part-o estudi-e tem-a part-a
estudi-a-s tem-e-s part-e-s estudi-e-s tem-a-s part-a-s
estudi-a tem-e part-e estudi-e tem-a part-a
estudi-a-mos tem-e-mos part-i-mos estudi-e-mos tem-a-mos part-a-mos
estudi--is tem--is part-s estudi--is tem--is part--is
estudi-a-n tem-e-n part-e-n estudi-e-n tem-a-n part-a-n
TEMA DE PRESENTE
Imperativo
1. conj. 2. conj. 3. conj. 1. conj. 2. conj. 3. conj.
estudi-a-r tem-e-r part-i-r estudi-a-ra tem-e-ra part-i-ra
estudi-a-r-s tem-e-r-s part-i-r-s estudi-a-ra-s tem-e-ra-s part-i-ra-s
estudi-a-r tem-e-r part-i-r estudi-a-ra tem-e-ra part-i-ra
estudi-a-re-mos tem-e-re-mos part-i-re-mos estudi-a-ra-mos tem-e-ra-mos part-i-ra-mos
estudi-a-r-is tem-e-r-is part-i-r-is estudi-a-ra-is tem-e-ra-is part-i-ra-is
estudi-a-r-n tem-e-r-n part-i-r-n estudi-a-ra-n tem-e-ra-n part-i-ra-n
TEMA DE FUTURO
Futuro de indicativo Condicional
1. conj. 2. conj. 3. conj. 1. conj. 2. conj. 3. conj.
estudi-a-ba tem--a part--a estudi- tem- part-
estudi-a-ba-s tem--a-s part--a-s estudi-a-ste tem-i-ste part-i-ste
estudi-a-ba tem--a part--a estudi- tem-i part-i
estudi--ba-mos tem--a-mos part--a-mos estudi-a-mos tem-i-mos part-i-mos
estudi-a-ba-is tem--a-is part--a-is estudi-a-ste-is tem-i-ste-is part-i-ste-is
estudi-a-ba-n tem--a-n part--a-n estudi-a-ro-n tem-ie-ro-n part-ie-ro-n
TEMA DE PRETRITO
Pretrito imperfecto de indicativo Pretrito perfecto simple de indicativo
1. conj. 2. conj. 3. conj. 1. conj. 2. conj. 3. conj.
estudi-a-ra/-se tem-ie-ra/-se part-ie-ra/-se estudi-a-re tem-ie-re part-ie-re
estudi-a-ra/-se-s tem-ie-ra/-se-s part-ie-ra/-se-s estudi-a-re-s tem-ie-re-s part-ie-re-s
estudi-a-ra/-se tem-ie-ra/-se part-ie-ra/-se estudi-a-re tem-ie-re part-ie-re
estudi--ra/-se-mos tem-i-ra/-se-mos part-i-ra/-se-mos estudi--re-mos tem-i-re-mos part-i-re-mos
estudi-a-ra/-se-is tem-ie-ra/-se-is part-ie-ra/-se-is estudi-a-re-is tem-ie-re-is part-ie-re-is
estudi-a-ra/-se-n tem-ie-ra/-se-n part-ie-ra/-se-n estudi-a-re-n tem-ie-re-n part-ie-re-n
Pretrito imperfecto de subjuntivo Futuro de subjuntivo
1. conj. 2. conj. 3. conj.
estudi-a tem-e part-e
estudi-a-d tem-e-d part-i-d
TEMA DE PRETRITO
Participio
1. conj. 2. conj. 3. conj.
estudi-a-do tem-i-do part-i-do
TEMA DE FUTURO
Infinitivo
1.
a
conj. 2.
a
conj. 3.
a
conj.
estudi-a-r tem-e-r part-i-r
Gerundio
1. conj. 2. conj. 3. conj.
estudi-a-ndo tem-ie-ndo part-ie-ndo
0B1LLLP(2008).05+s 3/6/08 19:19 Pgina 74
REFUERZO
3. Morfologa lxica
La morfologa lxica se ocupa de los procedimientos de formacin de palabras. Repasa cada
uno de estos procedimientos y responde despus a las cuestiones que se plantean sobre el
siguiente fragmento.
En la cafetera
Grupos bulliciosos de jvenes se arracimaban, charlando
y fumando, ante la barra de la cafetera, en un hervor humano,
confuso y excitante. Por el suelo se entremezclaban desper dicios
de marisco, huesos de aceitunas, puntas de cigarrillos, envoltu-
ras de azcar y servilletas de papel arrugadas. Vctor se situ en
un pequeo hueco, en el extremo de la barra, junto a la caja.
La muchacha ms vistosa una rubia de brazos pecosos y son-
rosados de las cuatro que atendan el mostrador, se dirigi
sonriente a Vctor al divisarle:
Un vinito? pregunt.
Un vinito, vale dijo Vctor.
Puso un vaso en la barra, cogi una botella de la estantera y
le sirvi:
De viaje otra vez?
Qu remedio!
Siempre de viaje. Cmo marchan las cosas?
Marchan, que no es poco.
Por la puerta de cristales abierta entraba un vaho de hume-
dad, pues apenas haban transcurrido cinco minutos desde el
ltimo chaparrn. En las aceras, hmedas, se vean centenares
de octavillas de colores, embarradas, pegadas al suelo. Por la
calzada, pas un coche con un altavoz estridente, pero iba tan
rpido que apenas pudo escucharse el comienzo de la alocucin
antes de que sus voces fueran sofocadas por el rumor del resto
de los automviles que circulaban por la amplia avenida.
Miguel DELIBES
El disputado voto del seor Cayo, Crculo de Lectores
En este fragmento de Miguel Delibes pueden
localizarse tres de los procedimientos
de formacin de palabras. Indica de cules
se trata y explica en qu consisten a partir
de ejemplos extrados del texto.
Qu procedimientos de formacin de
palabras no aparecen en el pasaje?
Defnelos y escribe algunos ejemplos.
Seala en el texto diez palabras en las que solo
aparezcan morfemas lxicos y morfemas
flexivos, y analiza su estructura interna.
Enumera las palabras derivadas del texto,
explica su formacin y analiza su estructura
interna.
Localiza en estas lneas de Delibes una palabra
compuesta, indica cmo se ha formado
y analzala morfolgicamente.
Aparecen palabras parasintticas en el primer
prrafo del texto? Y en el ltimo?
En caso afirmativo, realiza un anlisis
morfolgico similar al planteado en las
actividades 4 y 5 de esta pgina.
Al inicio del dilogo entre Vctor y la muchacha
rubia del mostrador se da un caso de sufijacin
apreciativa. Explica con qu valor se utiliza este
procedimiento y escribe otros ejemplos con
el mismo sufijo en los que este adquiera
valores diferentes.
7
6
5
4
3
2
1
75 5. Morfologa
M
A
T
E
R
I
A
L

F
O
T
O
C
O
P
I
A
B
L
E

/


O
x
f
o
r
d

U
n
i
v
e
r
s
i
t
y

P
r
e
s
s

E
s
p
a

a
,
S
.
A
.
ACTI VI DADES
5
0B1LLLP(2008).05+s 3/6/08 19:19 Pgina 75
ORTOGRAFA
4. Uso de b y v
El fonema /b/, bilabial sonoro, adopta la representacin grfica de b y v, lo que provoca
muchos problemas ortogrficos. Para evitarlos te ser de gran ayuda recordar algunas reglas.
Se escriben con b:
Los verbos beber, caber, deber, haber y saber: bebi, cabre-
mos, debera, hubo, saben.
Los verbos terminados en -buir y -bir, a excepcin de
hervir, servir, vivir y sus compuestos: distribuirn, atribu,
retribuir, escribiremos, recibi; pero hervimos, sirven,
revivi.
La caracterstica -ba- del pretrito imperfecto de indica-
tivo: iba, cantaban, rezabais.
Las palabras que contienen los prefijos bi-, bis-, biz- y los
elementos compositivos biblio- , bio- / -bio, bien- / bene-:
biangular, bisnieto, bizcocho, biblioteca, biologa, microbio,
bienvenido, beneplcito.
Las palabras que comiencen con las slabas bu-, bur-
y bus-: buceador, burgus, buscar.
Las palabras que acaban en -bundo / a y en -bilidad, a
excepcin de movilidad, civilidad (ya que derivan de los
adjetivos mvil y civil, respectivamente) y sus compues-
tos: vagabundo, amabilidad, inmovilidad.
Las palabras en las que el fonema /b/ precede a otra
consonante o est en final de palabra: abdicar, obtuso,
absolucin, nabab.
Se escriben con v:
Las palabras que empiezan con eva- (con excepcin de
bano), eve-, evi- y evo-: evadir, evento, evitar, evolucin.
Los adjetivos que acaban en -avo / a, -evo / a, -eve, -ivo / a,
y las palabras derivadas de algunos de ellos: esclavo,
esclavitud, esclavismo, nueva, novedad, leve, pasiva.
Las palabras que empiecen con el prefijo vice- / viz- / vi-:
vicedirector, vizconde, virrey.
Las palabras que acaben en -voro / a, con excepcin de
vbora: carnvoro, insectvora.
Los verbos acabados en -olver: volvers, disolvieron.
Los presentes de indicativo y subjuntivo y la segunda
persona del singular de imperativo del verbo ir: vas,
vayamos, ve.
El pretrito perfecto simple del indicativo y el pretrito
imperfecto y futuro del subjuntivo de los verbos andar,
estar, tener y sus derivados: anduvimos, anduvieras,
desanduviere, estuvo, retuviera, sostuviramos, tuve, retu-
viera, sostuviere, mantuviramos.
Las palabras que contengan el fonema /b/ cuando va
precedido por las slabas ad-, sub- y ob-: adversario, sub-
ver sivo, obvio.
Coloca b o v en las siguientes palabras segn corresponda:
reci__iremos __urladero contu__ieron hu__ieron con__alidar
__rtigo sucum__ieron micro__io __isla__as a__o__edada
con__encer __iscosidad hir__iente __iena__enturado longe__idad
__ra__ucn con__irtieron __olumen sir__iente a__negado
acti__idad __re__iario con__idaste __olu__le __i__ienda
a__sol__er disol__ente o__jeti__as con__ocatoria __oracidad
contri__uyente o__iedad decisi__a o__ser__aron con__ulsin
__ulgaridad atri__uido le__edad a__sol__ieron su__junti__o
en__idiosa a__ismo ca__ida ad__er__ial __ayan
__aga__undo fri__olidad com__inacin ca__aron ci__ismo
retu__ieron __est__ulo par__ulario de__ilitar sa__an
permea__ilidad mantu__iste __islum__rar __o__ino esta__le
ca____amos e__ita__le contu__ieran __oca__ulario equ__oco
go__ierno la__a__an omn__ora a__iertas re__erso
__l__ula coha__itacin i__ais her____oras gra__amen
ca__idad __enda__al apro__aron __ipolares estu__imos
1
76 5. Morfologa
M
A
T
E
R
I
A
L

F
O
T
O
C
O
P
I
A
B
L
E

/


O
x
f
o
r
d

U
n
i
v
e
r
s
i
t
y

P
r
e
s
s

E
s
p
a

a
,
S
.
A
.
ACTI VI DADES
5
0B1LLLP(2008).05+s 3/6/08 19:19 Pgina 76
Evaluacin
5
85 5. Morfologa
1. Resume el contenido del texto El cambio climtico,
publicado en la Revista Greenpeace.
2. Reconoce en el fragmento los elementos del proceso de
la comunicacin. De qu tipo de texto se trata, segn
su intencin comunicativa?
3. Reconoce en las siguientes palabras los morfemas flexi-
vos y el tema.
consecuencias grados inundaciones
marinas amenazas bajas
acuferos montaosas climticos
4. Distingue el morfema lxico raz y los morfemas flexivos
en las siguientes formas verbales, indicando el valor de
cada uno de ellos.
son actuamos frenar
podran aumentando aumentarn
hacen acelerar extinguido
5. Qu verbos de la actividad 4 son irregulares? Seala los
alomorfos de cada uno y explica su variacin.
6. Indica la categora gramatical de las palabras derivadas
del recuadro y la base de la que proceden.
Ejemplo climtico: adjetivo denominal
7. Analiza morfolgicamente las palabras ecosistemas,
dramticamente y acuferos. Indica por qu procedi -
miento/ s se ha formado cada una y seala sus morfemas,
indicando el significado lxico y/ o gramatical.
8. En el texto aparece la palabra recrudezca, que es una
forma flexionada del verbo recrudecer, que se conjuga
de manera anloga a reblandecer. Indica su proce -
dimiento de formacin.
9. En qu consisten las siglas y los acortamientos? Pon
ejemplos de ambos procedimientos.
10. Redacta un texto sobre la composicin: concepto, tipos,
estructuras ms frecuentes. No olvides brindar ejemplos.
irreversibles inundaciones probablemente costeras
velocidad existentes rapidez montaosas sequas
Algunas consecuencias del cambio climtico son ya
irreversibles, pero si actuamos con rapidez y firmeza
podemos frenar el aumento y conseguir que las tempe-
raturas no suban ms all de los dos grados. Si no
actuamos y continuamos al ritmo actual, las tempera -
turas podran subir ms de seis grados centgrados
durante este siglo, aumentando la magnitud de las
consecuencias.
Las inundaciones aumentarn dramticamente al pro-
ducirse cada vez ms tormentas violentas y lluvias
torrenciales. Los glaciares del planeta se derretirn a
una velocidad incluso mayor de a la que ya lo hacen,
provocando un aumento de agua en el mar y cambian-
do probablemente corrientes marinas, como la del
Golfo de Mxico. La subida del nivel del mar es una de las
principales amenazas de zonas costeras e islas, y afecta-
r ms a los pases ms pobres. En las costas espaolas,
para finales de este siglo se prev un aumento del nivel
de mar de 50 centmetros a un metro; podr causar pr-
didas de un nmero importante de playas, sobre todo
en el Cantbrico, e inundaciones de las zonas bajas cos-
teras. A la vez que los mares recibirn ms agua de la
fusin de los polos, los ros se secarn en muchas partes
del mundo poniendo seriamente en peligro los suminis-
tros acuferos. Las sequas estn siendo cada vez ms
frecuentes, por ejemplo en frica, Asia y la regin del
Mediterrneo. Millones de personas se ven amenazadas
por el hambre, especialmente en los pases ms pobres
y se espera que se recrudezca en las prximas dcadas.
La extincin de especies se acelerar debido a la dificultad
de animales, plantas y ecosistemas para adaptarse a los
rpidos cambios climticos. Estos peligros son realmente
serios para la fauna y la flora de los arrecifes de coral, bos-
ques, sabanas, regiones polares y cordilleras montaosas.
La comunidad cientfica teme que para 2050 se haya
extinguido un tercio de todas las especies existentes.
Asensio RODRGUEZ
Revista Greenpeace, n.

1, ao 2007
El cambio climtico
Playa de Acech, en el norte de Sumatra, el 26 de diciembre de 2004, despus del tsunami.
0B1LLLA.05 4/4/08 19:38 Pgina 85
REFUERZO
1. Construcciones sintcticas
A continuacin, se ofrecen dos textos del escritor argentino Jorge Luis Borges sobre los que
debes resolver las cuestiones sintcticas que se proponen luego.
Extrae los sintagmas nominales de los
dos textos de Jorge Luis Borges y analiza
sus estructuras.
Cules de dichos sintagmas funcionan
como sujeto?
Reconoce en los textos otros tipos
de sintagmas y de construcciones.
Reconoce y elabora un listado con las
oraciones simples de ambos textos.
Explica la estructura jerrquica que establecen
en cada una de ellas sus constituyentes.
Brinda ejemplos en las oraciones simples
de los textos que sean constituyentes,
pero no construcciones.
6
5
4
3
2
1
87 6. Sintaxis (I). La oracin simple
M
A
T
E
R
I
A
L

F
O
T
O
C
O
P
I
A
B
L
E

/


O
x
f
o
r
d

U
n
i
v
e
r
s
i
t
y

P
r
e
s
s

E
s
p
a

a
,
S
.
A
.
ACTI VI DADES
6
El amenazado
Es el amor. Tendr que ocultarme o que huir.
Crecen los muros de su crcel, como en un sueo atroz. La hermosa mscara ha cambiado, pero como siempre es
la nica. De qu me servirn mis talismanes: el ejercicio de las letras, la vaga erudicin, el aprendizaje de las pala-
bras que us el spero Norte para cantar sus mares y sus espadas, la serena amistad, las galeras de la Biblioteca,
las cosas comunes, los hbitos, el joven amor de mi madre, la sombra militar de mis muertos, la noche intemporal,
el sabor del sueo?
Estar contigo o no estar contigo es la medida de mi tiempo.
Ya el cntaro se quiebra sobre la fuente, ya el hombre se levanta a la voz del ave, ya se han oscurecido los que
miran por las ventanas, pero la sombra no ha trado la paz.
Es, ya lo s, el amor: la ansiedad y el alivio de or tu voz, la espera y la memoria, el horror de vivir en lo sucesivo.
Es el amor con sus mitologas, con sus pequeas magias intiles.
Hay una esquina por la que no me atrevo a pasar.
Ya los ejrcitos me cercan, las hordas.
(Esta habitacin es irreal; ella no la ha visto.)
El nombre de una mujer me delata.
Me duele una mujer en todo el cuerpo.
A un gato
No son ms silenciosos los espejos
ni ms furtiva el alba aventurera;
eres, bajo la luna, esa pantera
que nos es dado divisar de lejos.
Por obra indescifrable de un decreto 5
divino, te buscamos vanamente;
ms remoto que el Ganges y el poniente,
tuya es la soledad, tuyo el secreto.
Tu lomo condesciende a la morosa
caricia de mi mano. Has admitido, 10
desde esa eternidad que ya es olvido,
el amor de la mano recelosa.
En otro tiempo ests. Eres el dueo
de un mbito cerrado como un sueo.
Jorge LuiS BORGES
Obras completas, Emec
0B1LLLP(2008).06+s 4/6/08 06:49 Pgina 87
REFUERZO
2. Enunciados no oracionales
y oracionales en el periodismo
Confecciona una tabla y clasifica en ella
los enunciados oracionales y los no
oracionales que aparecen en estos
titulares.
Analiza la estructura de los enunciados
no oracionales y seala de qu tipo
de construccin se trata en cada caso.
Reconoce el sujeto y el predicado de
las estructuras oracionales atendiendo
a la concordancia.
Analiza en cada oracin los constituyentes
de los sujetos expresos y de los predicados.
(Recuerda las tcnicas de reconocimiento
de los distintos constituyentes.)
4
3
2
1
88 6. Sintaxis (I). La oracin simple
M
A
T
E
R
I
A
L

F
O
T
O
C
O
P
I
A
B
L
E

/


O
x
f
o
r
d

U
n
i
v
e
r
s
i
t
y

P
r
e
s
s

E
s
p
a

a
,
S
.
A
.
ACTI VI DADES
6
En los titulares de los peridicos se pretende condensar el contenido de la informacin de una
noticia o de un artculo de opinin empleando pocas palabras. En ellos son frecuentes tanto las
oraciones simples como los enunciados no oracionales. Obsrvalo en esta seleccin de titula-
res extrados de un peridico de tirada nacional.
Grammy al mejor disco
de pop latino
Columbus se acopla con xito
a la estacin espacial
Los lderes de Hams pasan a la
clandestinidad por temor a un ataque
La extraa dualidad entre
EE UU y Europa
La venta de pisos de Metrovacesa para
los prximos dos aos cae un 60%
Una reforma oportuna
de la Ley del menor
Muere un hombre
en un banco en Getafe
Vestidas con destellos
de opulencia
Un fuego reduce a cenizas la Puerta de Namdaemun,
el principal tesoro nacional de Corea del Sur
La Espaa igual
y la plurinacional
7000 firmas antiaeropuerto
en Getafe
La cartelera se queda
en cueros
Fusiones a la vista en la banca
y el sector de medios de comunicacin
Microsoft corteja a los accionistas
de Yahoo tras el rechazo de su OPA
El reptil volador
ms diminuto
Sonrisas y jbilo bajo una
intensa tromba de agua
El adis ms refinado
al rigor invernal
EE UU detiene a cuatro personas por
espionaje industrial y militar para China
Escalofriante arresto
en Bagdad
Nos esperan tres
semanas duras
Prisin para el detenido por
la cada de su novia
Cuatro facultades de
Medicina ms en 2008
Madrid exige a Londres un
sistema tripartito de vigilancia
Integrados sin derecho
a voto
Las dudas sobre los creditos de AIG causan
otra jornada bajista en el sector financiero
0B1LLLP(2008).06+s 4/6/08 06:49 Pgina 88
REFUERZO
3. Oraciones simples
en el lenguaje literario
El lenguaje literario brinda enunciados oracionales construidos cuidando la seleccin y la com-
binacin de las palabras que los configuran.
Realiza el anlisis sintctico de las siguientes
oraciones simples extradas de poemas de
Juan Ramn Jimnez.
a) En la calma magnfica del parque, reson
el beso con un eco largo.
b) Un ruiseor despierto lanz un dulce quejido
desgarrado.
c) Las avenidas se alargan entre la incierta
penumbra de la arboleda lejana.
d) Y en la onda transparente del cenit verdoso,
vagan misticismos de suspiro
y perfume de plegarias.
e) En el balcn, un instante nos quedamos
los dos.
f) Desde la dulce maana de aquel da, ramos
novios.
g) El paisaje sooliento dorma sus vagos tonos,
bajo el crepsculo gris y rosa del crepsculo
de otoo.
h) Mi alma es hermana del cielo gris y de las
hojas secas.
i) Los rboles del jardn estn cargados
de niebla.
j) El valle se queda ms solo y lbrego.
k) La otra tarde se ha llevado el viento ms
hojas secas!
l) Qu pena tendrn los rboles, esta noche
sin estrellas!
m) La luna camina muerta, sin luz de besos
ni lgrimas, amarilla entre la niebla.
n) Y la noche, tibia, serena y callada, dormir
el mundo, a los rayos de su luna solitaria.
) Y, de repente, una voz melanclica
y distante, ha temblado sobre el agua,
en el silencio del aire.
o) Luz y cristal su voz era en el surco removido.
p) Qu difcil es lo fcil!
q) Tu corazn y mi alma yerran solos por
la sombra de esta larga tarde azul,
tarde doliente de aromas.
r) Pero, acaso puede hablar de sus rosales
un corazn sepulcrado?
s) Negro sobre el cielo liso, revolotea un
murcilago.
t) Por las verjas se ve luz en las casas.
u) Los montes, con el confuso pinar
de la soledad, parecen de los difuntos.
v) En la paz del campo van dejando los troncos
muertos un olor fresco y honrado a corazn
descubierto.
w) Los caminos de la tarde se hacen uno,
con la noche.
x) Por encima de los oscuros tejados, verde,
lloroso de grillos y de esquilas, est el campo.
y) El cielo le da lagunas azules, suaves caadas,
llenas de nveos rosales y de abrigadas
cabaas.
z) Como una rueda visible del cielo, la luna roja
va acarreando la noche sobre la campia sola.
aa) Campana de Francia, lloras por mis amadas
de Espaa?
bb) La armona romntica del poniente de oro
va resbalando sobre el ro vespertino
cc) Huele a rosas pisadas.
dd) Libra de sus penosas dudas a este constante
asedio de mis penas.
ee) El luto duro y refrescante de la hora hablaba
a la ilusin de un imprevisto ambiente.
ff) Un agudo cuchillo de luz agria y equvoca
orna el medroso instante de un extrao
esplendor, delirante y amarillo.
gg) Dnde se han escondido los colores en este
da negro y blanco?
hh) Nos quedbamos solos en la hora serena.
ii) La muerte no sabr de nuestra dulce alma.
jj) Ya el rbol se ha resignado a su sereno morir.
kk) Las flores azules huelen a imposible,
entre dulces luces.
ll) El celeste se torna azul sonoro.
mm) Las estrellas empiezan a contemplar
el mundo.
1
89 6. Sintaxis (I). La oracin simple
M
A
T
E
R
I
A
L

F
O
T
O
C
O
P
I
A
B
L
E

/


O
x
f
o
r
d

U
n
i
v
e
r
s
i
t
y

P
r
e
s
s

E
s
p
a

a
,
S
.
A
.
ACTI VI DADES
6
0B1LLLP(2008).06+s 4/6/08 06:49 Pgina 89
REFUERZO
4. Anlisis sintctico
de anuncios publicitarios
La publicidad hace uso de distintos tipos de estructuras sintcticas que atraigan la atencin de
los receptores, los cuales combina con imgenes. El lenguaje verbal complementa, aclara o,
incluso, contradice lo que muestra el anuncio.
Reconoce en los enunciados el sujeto y el
predicado, aplicando la regla de la concordancia.
Si el sujeto es explcito, analzalo internamente,
estableciendo sus elementos constitutivos.
Analiza ahora el predicado, aplicando las
tcnicas de reconocimiento de los distintos
complementos verbales.
En cada predicado, diferencia complementos
argumentales y no argumentales.
Identifica en los anuncios las perfrasis
verbales y explica cul es su constitucin.
Indica el valor temporal o la modalidad
subjetiva de las perfrasis que has localizado
en la actividad 5.
6
5
4
3
2
1
90 6. Sintaxis (I). La oracin simple
M
A
T
E
R
I
A
L

F
O
T
O
C
O
P
I
A
B
L
E

/


O
x
f
o
r
d

U
n
i
v
e
r
s
i
t
y

P
r
e
s
s

E
s
p
a

a
,
S
.
A
.
ACTI VI DADES
6
El 70% de la superficie de la
tierra es agua. Disfrtelo.
El Crculo OLAY
lo hace posible.
Sonra. Tenemos grandes soluciones
para su piel por solo 7 euros.
Sonra. Tenemos grandes soluciones
para su piel por solo 7 euros.
Ahora con Ariel puedes conseguir
una limpieza ms brillante.
Dentro de un Suzuki la vida
se ve de otra manera.
Los caprichos de los grumetes
ahora puedes hacerlos al horno.
Todos tenemos recetas de
familia. Cul es la tuya?
La vida se disfruta
ms en compaa.
Zaragoza 2008, la Expo del agua y del desarrollo
sostenible, cuenta con el apoyo de Bob Dylan.
Solo en algunas ocasiones, unas memorias se convierten
en un relato tan apasionante e indispensable.
Llena de fragancia
toda la habitacin.
Qu le pides a tu
seguro del hogar?
El plan Activia 15 das
tambin es para ti.
Olvdate de la caspa.
Actimel ayuda a las defensas
en pocas de sobreesfuerzo.
Se acabaron las carreteras
aburridas.
Dale un buen mordisco
a la vida.
Qu parte de mi cuerpo
te gusta ms?
Cuenta con
Lizipaina
Concede a tu pelo
tres deseos.
Entre otras muchas cosas,
aparca solo.
Dale color
al momento.
Disfruta de un consumo
responsable.
Nunca un tratamiento ha aportado tanto a la piel
de una mujer: una nueva vida, un nuevo volumen,
una nueva edad.
Con Braun la suavidad se hace irresistible.
Acrcate a la suavidad.
0B1LLLP(2008).06+s 4/6/08 06:49 Pgina 90
ORTOGRAFA
5. Uso de g, gu, g y j
Recuerda:
Las slabas /ga/, /go/ y /gu/ se escriben con g a / o / u: gato, goloso, gula.
Las slabas /ge/ y /gi/ se escriben con gu e / i: guerra, guillotina.
Las slabas /gue/ y /gui/ se escriben con g e / i: cigea, lingstica.
Las slabas /xa/, /xo/ y /xu/ se escriben con j a / o / u: jalar, joya, jugar.
Las slabas /xe/ y /xi/ se escriben en unos casos con g e / i y en otros con j e / i: general, jefe, gitano, jirafa.
Coloca g o gu segn corresponda.
a)

errero d) a

obiante
b)

ila e) si

ientes
d) m

ico f)

itarra
Enuncia la regla por la que has optado por una u otra grafa en el ejercicio anterior.
Coloca diresis donde corresponda.
a) pinguino e) ceguera
b) cigueal f) linguista
c) liguero g) desague
d) halagueo h) aguita
Enuncia la regla por la que has colocado la diresis en algunas palabras de la actividad 3.
Coloca g o j, segn corresponda, en las palabras siguientes.

eografa salva

e co

ear apo

eo

eol

ica
di

eras mar

en

enial

eometra tri

simo
introdu

e neolo

ismo ma

ia

eringa presti

ioso

irafa pere

il re

ional cerra

ero peda

ico
conser

e parad

ico

il

uero

ersey antolo

a
fonolo

a bendi

e ind

ena fin

ir fin

o
fin

maldi

iste sur

en ho

ear reco

i
aborda

e enco

en te

i aco

ida esco

eremos
pea

e gran

ero aprendiza

e su

erencia condu

e
contradi

iste

estiones di

estivo a

encia in

erencia
dedu

imos au

e corre

ir efi

ie tra

n
exi

ente contra

e fu

itivo su

ecin pr

imo
infli

ir infrin

ir o

iva he

emona mu

ido
mensa

ero ma

estad r

ida nostal

ia

irn
lison

era ter

iversar intan

ible

inete

erarquizar
in

ertar transi

ir vi

encia homena

ear prote

er
ele

iste

ara

e con

etura produ

e emer

ieron
cru

ir exi

irn exi

ente cru

iente

il
ener

a verti

inosa can

ear le

islacin tra

edia

ante ba

eza li

ereza ena

enar pela

e
extran

ero re

a morfolo

a in

eniosa masa

e
Despus de corregir esta ltima relacin de palabras, escribe nuevamente aquellas en que te hayas
equivocado y forma voces derivadas de las mismas cuando sea posible. En los casos en que se trate
de una forma verbal, escribe, adems, otras de su conjugacin con el mismo lexema.
6
5
4
3
2
1
91 6. Sintaxis (I). La oracin simple
M
A
T
E
R
I
A
L

F
O
T
O
C
O
P
I
A
B
L
E

/


O
x
f
o
r
d

U
n
i
v
e
r
s
i
t
y

P
r
e
s
s

E
s
p
a

a
,
S
.
A
.
ACTI VI DADES
6
0B1LLLP(2008).06+s 4/6/08 06:49 Pgina 91
Evaluacin 6
101 6. Sintaxis (I). La oracin simple
1. Caracteriza el fragmento de Azorn atendiendo a los
siguientes aspectos:
Intencin comunicativa.
Modalidades del discurso.
2. Las siguientes construcciones han sido extradas del
texto. Indica de qu tipo es cada una de ellas, analiza
sus constituyentes y establece la estructura jerrquica.
a) En la pequea ciudad.
b) El quicio de una reja.
c) Frente a la catedral.
d) En la taza de la fuente.
e) Espa todos los ruidos de la ciudad.
3. Localiza los verbos pronominales que aparecen en el
pasaje de Don Juan.
4. Seala las perfrasis verbales que aparecen en el texto,
explica su estructura e indica su valor.
5. Reconoce todas las oraciones simples que componen el
fragmento de Azorn.
6. Analiza sintcticamente las oraciones que has identificado
en la actividad 5.
7. Enuncia los argumentos exigidos por los siguientes verbos
del texto.
aparecer ser bajar reposar
ver cesar dirigirse espiar
8. Realiza el anlisis sintctico de las siguientes oraciones,
construidas a partir del texto.
a) Un pasadizo une la catedral al palacio del obispo.
b) Chirra estrepitoso el quicio de una puerta.
c) Encontraba relajante el ruido del chorro del agua en
la fuente.
d) Todas las maanas, el obispo es seguido por dos
familiares.
9. Escribe una oracin que responda a la estructura que se
indica en el recuadro.
10. Compara por escrito los complementos directo y de
rgimen: comienza por sus semejanzas y establece des-
pus sus diferencias, atendiendo tanto a la estructura
como al reconocimiento. Pon ejemplos de ambos.
S (Det N Ady Ap) - P (N (Perf V) C Rg CD)
El obispo
Una dbil claridad aparece en las altas vidrieras de la cate-
dral. Es la hora del alba. A esta hora baja el obispo a la catedral.
El palacio del obispo est unido a la catedral por un pasadizo
que atraviesa la calle. A la hora en que el obispo entra en la cate-
dral todo reposa en la pequea ciudad. La catedral est casi a
oscuras; resuenan, de cuando en cuando, unos pasos; chirra el
quicio de una reja. En la pequea ciudad la luz de la maana va
esclareciendo las callejas. Se ve ya, en la plaza que hay frente a la
catedral, caer el chorro del agua en la taza de la fuente; el ruido
de esta agua, que haba estado percibindose toda la noche, ha
cesado ya.
El obispo est ciego, ciego como el dulce y santo obispo
francs Gastn Adrin de Sgur. Entra en la catedral despacito,
va sostenindose en un cayado; [ ] le van siguiendo dos fami-
liares. La amplia capa cae en pliegues majestuosos hasta las
losas. Se dirige el buen prelado hacia la capilla del maestre don
Ramiro. De cuando en cuando se detiene, apoyado en su bas-
tn, con la cabeza baja, como meditando. Su pelo es abundante
y blanqusimo. Destaca su noble cabeza en el vivo morado de
las ropas talares
1
. No puede ya ver el obispo su catedral, ni su
ciudad. Pero desde su cuartito, l, todas las maanas, a la hora
en que rompe el alba, espa todos los ruidos de la ciudad, que
renace a la vida: el canto de un gallo, el tintn de una herrera, el
grito de un vendedor, el ruido de los pasos.
Jos MARTNEZ RUIZ, AZORN
Don Juan, Espasa Calpe
1
talar: que llega hasta los talones. Interior de la catedral de Mnster, por Lorenz Ritter.
0B1LLLA.06 31/3/08 22:00 Pgina 101
REFUERZO
1. Clases de oraciones simples
Las oraciones simples pueden clasificarse atendiendo a dos criterios: segn la actitud del
hablante y segn sus relaciones sintcticas.
Analiza sintcticamente las oraciones.
Clasifcalas atendiendo a la actitud del
hablante.
En cada una de las oraciones con se, distingue
el tipo de pronombre de que se trata.
Clasifica las oraciones atendiendo a las
relaciones sintcticas:
Sujeto-predicado / impersonales.
Atributivas / predicativas.
Transitivas / intransitivas.
Pronominales / no pronominales.
4
3
2
1
115 7. Sintaxis (II). Clases de oraciones. La coordinacin
M
A
T
E
R
I
A
L

F
O
T
O
C
O
P
I
A
B
L
E

/


O
x
f
o
r
d

U
n
i
v
e
r
s
i
t
y

P
r
e
s
s

E
s
p
a

a
,
S
.
A
.
ACTI VI DADES
7
a) A qu hora llegarn los invitados?
b) Se arrodill ante el altar.
c) No ocurri nada importante.
d) Los pisos superiores fueron rpidamente alcanzados por las llamas.
e) No se la asusta fcilmente.
f) Se ha avisado ya a la polica?
g) Se sabe de memoria la leccin.
h) Finalmente result ser la ganadora.
i) Cada vez amanece ms temprano.
j) No se puede comer todava el postre.
k) Le va muy bien en sus estudios.
l) Qu te falta en la maleta?
m) S atenta con los invitados.
n) Ojal no llueva este fin de semana.
) No se saludaron esta maana.
o) En ese negocio se necesita un ayudante.
p) Siempre hay mosquitos y lucirnagas por la noche junto al ro.
q) Los rumores fueron divulgados por esa cadena de televisin.
r) Por dnde se llega antes al pueblo?
s) Se convoc a todos los alumnos.
t) Ya se lo han dicho?
u) Dos empleados fueron agredidos a la salida del trabajo.
v) ltimamente se estudia muy poco.
w) Desde nuestra ventana se puede contemplar el atardecer.
x) Debis dejar limpias las aulas.
y) Se trata de los estudios de su hijo.
0B1LLLP(2008).07+s 3/6/08 16:04 Pgina 115
REFUERZO
2. Oraciones coordinadas
Localiza en los textos las oraciones simples.
Reconoce las oraciones compuestas e indica
las que las forman.
Seala el tipo de coordinacin que une
las oraciones e indica los nexos.
Qu tipo de coordinadas predominan
en estas oraciones?
Qu coordinacin est ausente en esta
relacin de oraciones? Construye cinco
ejemplos de este tipo.
5
4
3
2
1
116 7. Sintaxis (II). Clases de oraciones. La coordinacin
M
A
T
E
R
I
A
L

F
O
T
O
C
O
P
I
A
B
L
E

/


O
x
f
o
r
d

U
n
i
v
e
r
s
i
t
y

P
r
e
s
s

E
s
p
a

a
,
S
.
A
.
Las siguientes oraciones, extradas todas ellas de una obra de Azorn, se ofrecen para practicar
el anlisis sintctico de oraciones compuestas por coordinacin.
a) Parece unas veces perdida la mirada de la seora en una lontananza invisible; otras, pasa y repasa
sobre el haz de las cosas a manera de silenciosa caricia. De pronto, un pensamiento triste conturba a
la desconocida: la mirada se eleva y un instante resalta en lo trigueo de la faz lo blanco de los ojos.
b) Desde el balcn se divisa un panorama de tejados; abajo culebrea la calle estrecha y pendiente.
El piso de la estancia es de ladrillos rojos; algunas de estas losetas han perdido el barniz y se deshacen
en un polvillo tenue.
c) Doa Ins se levanta y se acerca a la puerta de la sala. No ha sido nada, reina el silencio. Los visillos
del balcn son ladeados por la fina mano; la mirada pasea vagamente por el panorama de los te ja-
dos y baja hasta el fondo de la calle. No est intranquila la dama y no acaba de sentir perfecto sosiego.
d) Doa Ins ha cogido la carta, la ha rasgado en cien pedazos y ha abierto el balcn. La mano fuera
del balcn lanzaba los cien pedacitos de papel blanco. Los mltiples pedacitos de papel caan, vola-
ban, revoloteaban en la luz penumbrosa del crepsculo, y una encendida estrella rutilaba en el cielo
difano.
e) Unos ojos negros tienen destellos de bondad, unas veces; otras, miran de hito en hito y misteriosos.
Y unos brazos se levantan y dejan recortado en el fondo indefinido un busto firme y esbelto.
f) En la ciudad todo se desenvuelve automticamente; todo obedece a la luminosidad de la hora y de
la estacin. El sol con su luz viva suscita el bullicio y el estruendo de los moradores. Va declinando
la viva luz solar: el estrpito y el trfago se van amortiguando. As, al par de la carrera del sol por el
firmamento, crece o decrece en la ciudad el oleaje del tumulto y de los mil ruidos.
g) Ya le dan de vestir a la seora; el palacio bulle de gente; ya le trae una camarera el blanco brial; en el
palacio los caballeros y las damas van y vienen por corredores y galeras. Ya una camarera le ajusta
el corpio a la dama, y otra la baa con aguas de olor. En el palacio los juglares ren y chancean con los
caballeros.
h) Doa Beatriz estaba muy triste. Abatida andaba por el palacio; sus camareras la miraban con
melancola; no se hablaba nada en la cmara de la dama; pero todos tenan fijo el pensamiento
en el trovador.
i) Don Santiago me encanta por su bondad; no le he tratado mucho; pero adora apasionadamente
a sus hijos y es amigo de todos los nios, y no puede verlos sufrir.
j) Una valenciana vestida de negro es un prodigio. La tez es blanca, marfilea, de una blancura mate,
suave; las facciones son llenas, con redondeces mrbidas; acaso en la blancura se advierten las rami-
ficaciones sutiles de las venillas azules. Y lo negro hace resaltar la blancura maravillosa de esta piel
y pone un poco de veladura melanclica en la languidez de la mirada.
k) La vida entre aquella gente era grata y amena. Haban llenado de dibujos curiosos y de rtulos cho-
carreros las puertas, las paredes y las mesas. Yo me hice amigo de uno de aquellos pintores; le acom-
paaba algunos ratos en sus excursiones; tena un verdadero talento de paisajista. Nos levan t -
bamos antes de amanecer; l gustaba de pintar la luz fina de la maana, y en esa luz, la hojarasca
sutil de la primavera.
AZORN
Doa Ins, Castalia
ACTI VI DADES
7
0B1LLLP(2008).07+s 3/6/08 16:04 Pgina 116
ORTOGRAFA
3. Uso del parntesis, la raya
y el guin
El parntesis encierra elementos aclaratorios o secundarios incluidos en un enunciado. Se utiliza
en los siguientes casos:
Cuando se intercala un inciso que constituye una aclaracin o una informacin secundaria:
Quiere terminar el Bachillerato (que le brindar ms posibilidades en el futuro) y se esfuerza
muchsimo. Para estos incisos tambin se pueden usar rayas.
Para intercalar algn dato (fechas, lugares, significados de siglas, autor u obra citados): Rubn
Daro (1867-1916) naci en Metapa (Nicaragua); Las actuaciones del FMI (Fondo Monetario
Internacional) han sido discutidas; Lo bueno, si breve, dos veces bueno; y aun lo malo, si poco, no
tan malo. (Gracin)
Cuando se transcriben textos se utilizan tres puntos entre parntesis () para indicar que se
omite un fragmento.
Para las letras o nmeros que sealan enumeraciones o clasificaciones:
Los parntesis se usan en los siguientes casos:
a) Cuando se intercala un inciso.
La raya se puede usar aisladamente o bien como signo de apertura y de cierre que incluye un
elemento o enunciado. Se utiliza en los siguientes casos:
Para intercalar incisos o aclaraciones (en este empleo pueden ser sustituidas por parntesis
o por comas): Quiere abandonar el Bachillerato est muy desanimada y ponerse a trabajar.
Para sealar cada una de las intervenciones de un dilogo:
No puedes abandonar ahora.
T crees que soy capaz de aprobar esas asignaturas?
Para intercalar las intervenciones del narrador en el dilogo de los personajes.
Piensa con calma en lo que vas a hacer dijo la profesora. Piensa en tu futuro.
El guin es un trazo ms corto que la raya y se usa para separar los dos elementos de una palabra
compuesta o para marcar la divisin de una palabra al final del rengln: terico-prctico; ren-gln.
Coloca los parntesis que faltan en los
siguientes enunciados.
a) Las palabras que empiezan por los
elementos compositivos xeno-
extranjero, xero- seco, rido
y xilo- madera se escriben
con x.
b) Jos Mara Merino A Corua, 1941 ha
obtenido numerosos premios literarios
a lo largo de su trayectoria.
c) La novela picaresca se inici con
el Lazarillo de Tormes 1554.
d) Poderoso caballero es don Dinero Francisco
de Quevedo.
e) La letra x, al final de palabra o en posicin
intervoclica, representa el grupo
consonntico ks o gs en pronunciacin
relajada.
f) Cierta tarde de otoo fresca y lluviosa como
casi todas ese ao, haba tomado aquella
drstica decisin.
Escribe los parntesis, rayas y comillas
que faltan en estos enunciados (extrados
de La magia de leer, de Jos Antonio Marina).
a) Nacemos con una insaciable hambre de
estmulos. En este sentido y en otros, por
supuesto, nos parecemos mucho a nuestros
primos los primates.
b) Hay lectores a quienes les gusta releer el mismo
libro Blanca, mi hija pequea, lleg a leerse el
primer libro de Harry Potter cincuenta veces!
y los hay vidos de novedades.
c) Sin la ayuda del lenguaje somos estpidos,
inarticulados, toscos e insociables. Posibilidades
o mejor, imposibilidades tristes todas ellas.
d) Otra fundacin, la FAD Fundacin de Ayuda
contra la Drogadiccin, realiz, en 2004,
una gua de lectura, basndose en el libro
Adivina cunto te quiero, escrito por un
maestro irlands, Sam McBratney.
e) La revolucin francesa fue una revolucin
de lectores: En Pars dice un testigo todo el
mundo lee.
2 1
117 7. Sintaxis (II). Clases de oraciones. La coordinacin
M
A
T
E
R
I
A
L

F
O
T
O
C
O
P
I
A
B
L
E

/


O
x
f
o
r
d

U
n
i
v
e
r
s
i
t
y

P
r
e
s
s

E
s
p
a

a
,
S
.
A
.
ACTI VI DADES
7
0B1LLLP(2008).07+s 3/6/08 16:04 Pgina 117
Evaluacin 7
115 7. Sintaxis (II). Clases de oraciones. La coordinacin
1. Resume el contenido del fragmento de Flix de Aza.
2. Qu modalidades oracionales encuentras en el texto
Agosto?
3. Escribe ejemplos (propios y originales) de las modalidades
oracionales que no aparezcan.
4. Por qu dice el autor que la tercera adolescente ha
hundido a sus amigas? Qu inferencias ha hecho?
5. Localiza en el discurso del narrador dos verbos intran-
sitivos, dos verbos transitivos y todos los verbos que
presenten atributo.
6. Identifica en esos mismos enunciados las oraciones
simples y las compuestas.
7. Seala la funcin sintctica que cumplen los pronombres
tonos subrayados en el pasaje.
8. Analiza sintcticamente las siguientes oraciones del texto
y descrbelas segn los criterios estudiados.
a) La madre es castellana y se tuesta al sol.
b) Tres adolescentes, a mi derecha, comentan sus asuntos.
c) La primera suspira y se tumba en la toalla.
9. Qu usos del pronombre se no aparecen en el texto?
Pon un ejemplo de cada uno de ellos.
10. Contina el texto: imagina que ests en esa playa (o en
otra cualquiera) e informa de lo que ocurre; incluye
reflexiones sobre lo que ves y sobre tus sensaciones.
A esta hora la playa est ya bastante concurrida pero a
las doce no se podr dar un paso. Tengo delante a una familia
cosmopolita. La abuela es gallega, va vestida de negro y se cobija
bajo una sombrilla. Es una estampa antigua y no se pierde detalle.
CAy, carallo, exclama de vez en cuando. La madre es castellana
y se tuesta al sol. El padre habla en cataln con la hija, una nia
que, disgustada por el tamao de sus muslos, se cubre de cintura
para abajo con una toalla. El padre quiere que se la quite pero
ella hace mohnes. Tres adolescentes, a mi derecha, comentan
sus asuntos. Xabi es que es el ms guapo de la colla, dice una.
Y multimillonario, matiza la segunda. Pero un plomo, cuando
te llama por telfono te tiene una hora, yo miro la tele mientras
habla, afirma la tercera. Las ha hundido. La primera suspira
y se tumba en la toalla, voy a dormir un rato que ayer estuve
hasta las cinco con, bueno, ni s cmo se llamaba, deja caer
para compensar.
La abuela, muy animada, nos anuncia un evento, ya sale, ya
sale la rapacia.
Flix DE AZA
Esplendor y nada, Lector
Agosto
Playa azul, por Marcel Gromaire.
0B1LLLA.07 7/4/08 13:48 Pgina 115
AMPLIACIN
1. La vaguedad de las palabras
El siguiente texto trata sobre una caracterstica muy importante del lenguaje humano: la
vaguedad de las palabras. Este rasgo de las lenguas plantea a veces dificultades de comuni-
cacin a la hora de designar realidades concretas.
Palabras exactas y palabras inexactas
Decimos que un trmino es vago cuando sus lmites designa -
tivos son imprecisos. En tales circunstancias siempre existen reali-
dades que pueden ser denotadas tanto por l como por signos veci-
nos en el sistema. No es infrecuente que una mujer de 25 aos sea
designada con los nombres de nia, chica, muchacha, mujer e
incluso seora. Ante un montn de tablas viejas unos hablantes
aplicaban el trmino lea y otros la palabra madera. En un artculo
de prensa se designaba una va de comunicacin urbana ya como
calle ya como avenida. De mediana edad es un trmino vago en
el sentido que acabamos de especificar. Uno no es de mediana
edad ni a los cinco ni a los ochenta aos; s lo es a los cincuenta;
pero, qu es a los 39, a los 41 o a los 60? A cada lado de los casos
claros de edad parece haber una franja respecto a la cual no est
claro qu es lo que debemos decir (Alston, 1974).
La vaguedad es un fenmeno de designacin, no de significa-
cin. Deriva de las fronteras borrosas que posee la clase designativa
de los signos de las lenguas naturales. En el significado se estable-
cen oposiciones, saltos cualitativos. Los significados son unidades
discretas, pero en la designacin la transicin es imperceptible.
Dnde est el lmite en la realidad entre claro y oscuro? []
Los signos de las lenguas naturales conocen un alto grado de
vaguedad. El lenguaje cientfico y administrativo, que persigue la
precisin y exactitud, aspira a evitarla en la medida de lo posible.
Esta es la razn por la que en la ley el paso de menor a mayor de
edad sea fijado minuciosamente (las cero horas del da en que se
cumplen los 18 aos). Esta precisin es un ideal o desideratum
que rara vez se consigue, pues las definiciones del lenguaje cientfico
utilizan trminos del lenguaje natural (que a su vez son vagos). []
El reglamento del ftbol pretende delimitar con minuciosidad las
circunstancias del penalti, pero cada jornada deportiva aparecen
casos dudosos y discutibles.
Salvador GUTIRREZ ORDEZ
Introduccin a la semntica funcional
Sntesis
Escribe un resumen en el que recojas las
ideas principales expuestas en el texto
anterior.
Cmo se organizan los contenidos que
desarrolla este fragmento de Gutirrez
Ordez?
En el caso de los nombres con que puede ser
designada una mujer, qu factores
contextuales o situacionales pueden intervenir
en la eleccin de un trmino u otro?
Menciona otros ejemplos de trminos vagos
cuyo empleo sea habitual en el lenguaje
cotidiano.
Menciona voces del mbito escolar cuya
aplicacin se halle delimitada y establecida
y que, sin embargo, en ocasiones planteen
distintas interpretaciones.
Qu procedimientos pueden usarse
en el lenguaje cotidiano para restringir
la vaguedad de algunas palabras?
6
5
4
3
2
1
131 8. Semntica y lxico
M
A
T
E
R
I
A
L

F
O
T
O
C
O
P
I
A
B
L
E

/


O
x
f
o
r
d

U
n
i
v
e
r
s
i
t
y

P
r
e
s
s

E
s
p
a

a
,
S
.
A
.
ACTI VI DADES
8
0B1LLLP(2008).08+s 3/6/08 18:42 Pgina 131
REFUERZO
2. Cuestiones semnticas
Cmo designa el narrador de Sin noticias
de Gurb a los seres humanos que viven cada
etapa evolutiva y qu rasgos semnticos
usa para caracterizar estas fases?
Establece el campo semntico de las edades
o etapas de desarrollo de las personas
y compn un texto en el que las caracterices.
Escribe los sinnimos posibles para designar
a las personas que viven cada una de las
diferentes etapas.
Con qu dificultad nos podemos encontrar
a la hora de aplicarle a las dems personas
trminos como nio, adolescente, joven,
anciano, seora, etc.? Qu ocurre con estas
palabras en relacin con lo que designan?
Algunas de estas voces pueden usarse
tambin con valores connotativos;
pon ejemplos que ilustren estos casos.
Para aludir a una determinada etapa de la vida,
se usa tambin la expresin tercera edad.
Qu clasificacin supone esta denominacin?
Justifica tu respuesta.
Lee atentamente las normas propuestas
en el fragmento del Libro de estilo de El Pas.
Responden al uso habitual que hacemos
de esos trminos?
Menciona ejemplos de situaciones
comunicativas en las que exista coincidencia
o disidencia con lo que se afirma en este
texto.
8
7
6
5
4
3
2
1
132 8. Semntica y lxico
M
A
T
E
R
I
A
L

F
O
T
O
C
O
P
I
A
B
L
E

/


O
x
f
o
r
d

U
n
i
v
e
r
s
i
t
y

P
r
e
s
s

E
s
p
a

a
,
S
.
A
.
Los siguientes textos plantean la cuestin de los agrupamientos lxicos, en este caso, el cam-
po semntico concreto de las edades del hombre, y el problema que surge en muchas ocasio-
nes para determinar el contenido de los miembros de un campo.
El primer fragmento pertenece a una novela humorstica de Eduardo Mendoza, Sin noticias de
Gurb, cuyo protagonista es un extraterrestre que llega a la tierra y describe lo que observa des-
de su perspectiva.
El segundo texto pertenece al Libro de estilo del diario El Pas. Este diario considera necesario
establecer para sus redactores normas que determinen el trmino adecuado para refe rirse a
las personas segn su edad.
ACTI VI DADES
8
Texto 1
Los seres humanos, en cambio, a semejanza de los
insectos, atraviesan por tres fases o etapas de desarrollo,
si el tiempo se lo permite. A los que estn en la primera
etapa se les denomina nios; a los de la segunda,
currantes; y a los de la tercera, jubilados. Los nios
hacen lo que se les manda; los currantes, tambin, pero
son retribuidos por ello; los jubilados tambin perciben
unos emolumentos, pero no se les deja hacer nada, por-
que su pulso no es firme y suelen dejar caer las cosas de
las manos, salvo el bastn y el peridico. [] Entre los
seres humanos, como entre nosotros, se da tambin una
cuarta etapa o condicin, no retribuida, que es la de
fiambre, y de la que ms vale no hablar.
Eduardo MENDOZA
Sin noticias de Gurb
Seix Barral
Texto 2
Hay que evitar expresiones tan desafortunadas (y frecuentes) como una joven de 33 aos. La norma es la siguiente: beb, menos de un ao; nia o nio, de 1 a 12 aos; joven y adolescente, de 13 a 18 aos; hombre o mujer, ms de 18 aos; anciana o anciano, ms de 65 aos. Este ltimo trmino, sin embargo, solo debe emplearse muy excepcionalmente, y ms como expo- nente de decrepitud fsica que como un estadio de edad. En tales casos, soslyese con expresiones como un hombre de 65 aos.
Libro de estilo
El Pas
0B1LLLP(2008).08+s 3/6/08 18:42 Pgina 132
REFUERZO
3. Nuevos significados
A lo largo del tiempo, las palabras adquieren nuevos significados. El siguiente texto aborda
este fenmeno semntico. Lelo atentamente y responde a las cuestiones que se plantean a
continuacin.
Perdonar
En el mbito futbolstico se ha desarrollado hace poco con viru-
lencia agresiva una metfora que juzgo incurable. La omos a diario
(esta temporada, literalmente a diario): el equipo se estira, el delan-
tero le gana la espalda (?) a la defensa, est solo ante el portero en
uno contra uno, el gol se ruge ya por la multitud, pero el crack chuta
y manda la pelota a hacer gambetas al bandern. Y en ese instante,
indefectiblemente, el comentarista-filsofo que suele acompaar
en las retransmisiones al narrador, emite su solvente excogitacin:
el Zaragoza (lo nombro porque lo quiero y porque es diestro en esa
pifia) est perdonando mucho. Luego, el exegeta asevera grave:
El equipo que perdona mucho acaba perdiendo. Y enseguida,
sentenciar ms hondo an: El ftbol es as.
Es probable que toda la comunidad hablante adopte pronto el
verbo perdonar con ese inusitado significado intransitivo: En el ft-
bol, desperdiciar repetidamente un equipo las ocasiones de meter
gol, antes se deca simplemente fallar. La nueva acepcin, por el
momento, solo pertenece a la jerga balompdica, pero como el ft-
bol sale hasta por el tubo del dentfrico, el vocablo ser muy pronto
de conocimiento general. Y de este modo, un tropo inventado como
graciosa creacin personal por un ignoto artista de la crnica
deportiva, ha cundido hasta rebosar por toda la extensin de las
ondas y del papel.
Ello constituye buena prueba de que el desenfado de muchos
tales comentaristas puede convertir en triunfo el dislate. Porque per-
donar significa en el habla comn alzar la pena, eximir o liberar de
una obligacin a alguien. Y el arquero no tena obligacin de dejar-
se meter gol: no haba que eximirlo, al contrario. Por otra parte,
quien perdona lo hace adrede y cobra fama de misericordioso, pero
las gradas embravecidas suelen llamar imbcil al futbolista o al
equipo que, queriendo arrasar al contrario todo menos perdonar-
lo!, marra el tanto tenindolo a huevo.
Evidentemente, el idioma del estadio y de sus aledaos periods -
ticos es el ms desenfadado de todos, y en l se produce la mayor
creatividad imaginable, en gran parte bastante estlida.
Fernando LZARO CARRETER
El dardo en la palabra
Crculo de Lectores
En el texto, el autor reflexiona sobre un nuevo
significado otorgado por los comentaristas
deportivos a la palabra perdonar en su uso
metafrico. Haz un resumen de las ideas
expuestas en estas lneas.
Ests de acuerdo con la ltima afirmacin
del profesor Lzaro Carreter? Por qu?
Cita otras metforas usuales en la jerga
del ftbol.
En las crnicas de ftbol, a qu campo
asociativo suelen pertenecer los trminos
utilizados metafricamente? Por qu?
Escribe enunciados en los que las siguientes
palabras tengan significados distintos
a los del texto: equipo, defensa, portero,
chutar, gol.
Redacta la definicin de crack, delantero
y estadio.
Crees que en los comentarios de otras
competiciones deportivas (ciclismo,
baloncesto, tenis, etctera) se utilizan
o crean metforas semejantes a la que se
critica en el texto? En caso afirmativo, pon
algn ejemplo.
7
6
5
4
3
2
1
133 8. Semntica y lxico
M
A
T
E
R
I
A
L

F
O
T
O
C
O
P
I
A
B
L
E

/


O
x
f
o
r
d

U
n
i
v
e
r
s
i
t
y

P
r
e
s
s

E
s
p
a

a
,
S
.
A
.
ACTI VI DADES
8
0B1LLLP(2008).08+s 3/6/08 18:42 Pgina 133
REFUERZO
4. Extranjerismos lingsticos
El lenguaje cientfico y tcnico incorpora muchos extranjerismos; he aqu una reflexin del lin-
gista Rafael Lapesa acerca de este fenmeno:
El equilibrio justo
S, estamos inmersos en el extranjerismo lingstico. Nos rodea, nos aturde, nos invade. Se
interpone entre las cosas y nosotros, se infiltra en nuestra manera de verlas. Modifica paula-
tinamente la estructura de nuestra lengua y la configuracin de nuestro pensamiento. Ocurre
as en gran parte por frivolidad o ignorancia; en otra gran parte por cuquera propagandstica:
el marchamo forneo deja boquiabiertos a los papanatas, y estos abundan tanto que es
prctico deslumbrarlos. Pero hay otra causa innegable y ms profunda, y es que desde hace
tres siglos vamos a remolque del restante mundo occidental, tanto en las innovaciones con
que la ciencia y la tcnica han cambiado las condiciones de la vida humana, cuanto en la
exploracin de nuevos derroteros para las ideas. Primero fuimos a la zaga de Francia; des-
pus, a la de Francia, Inglaterra y Alemania; ahora, adems, a la de Estados Unidos. Con las
ideas y las cosas vienen inevitablemente las palabras. Ante este hecho, las lamentaciones son
tan intiles como las protestas. La nica actitud positiva consistir en aprovecharnos del
impulso ajeno tan hbilmente que en un futuro ms o menos prximo podamos seguir nues-
tro camino con autonoma; y por lo que se refiere al lenguaje, asimilar lo necesario para que
nuestro idioma se mantenga a la altura de los tiempos, sin dejar de responder a lo que estos
exigen; pero evitar en lo posible que tal puesta al da menosprecie su belleza y peculiaridad.
El aluvin de extranjerismos no es problema que afecte solo al espaol. Por todas partes
se oyen clamores parecidos, como ya dice Madariaga. Es que el mundo se nos queda chico.
Tenemos noticia inmediata de lo que est pasando en Los ngeles, Ciudad del Cabo, Nueva
Delhi o Gteborg, y con frecuencia lo vemos en directo. La comunicacin es tan rpida que
no deja tiempo a traducciones reposadas. Las novedades tcnicas desde el ciclotrn al
humilde rollito de cinta adherente se difunden a toda prisa con su nombre de origen,
antes que se les busque sustituto, y muchas veces sin que quepa buscrselo por ser marca
registrada. La frecuencia de viajes, el intercambio de turistas, estudiosos y emigrantes, todo
conduce a la creciente uniformacin de horizontes y ambientes vitales, a la familiaridad con
lo que antes era extrao, a su infiltracin en ideas, costumbres, maneras y lenguaje. En un
mundo en que se internacionalizan sociedades masificadas el purismo lingstico est con-
denado al ms rotundo fracaso. Pero tampoco debemos cruzarnos de brazos y abandonar
a la evolucin ciega un instrumento de tan rica tradicin y maravillosa capacidad como es
nuestra lengua. []
En resumen, las circunstancias reclaman que en el uso de nuestra lengua tengamos una
actitud de razonable abertura frente al neologismo: importar lo imprescindible y lo conve-
niente, sin fruncir demasiado el ceo ante alguna fruslera de contrabando. Los sagaces
casuistas de antao saban que en ciertas ocasiones es necesario lo superfluo. El proteccio-
nismo deseable debe consistir en sacar partido a los recursos naturales del idioma y a los de
su patrimonio grecolatino; limitar el capricho con hbil flexibilidad; e imponer montaje
fonolgico y gramatical espaol a lo que exhiba patente aliengena. As podremos seguir
yendo a Calatayud y no a Kahlahtahyood; pero con nimo de no detenernos en la recia
localidad aragonesa, con miras a horizontes ms ambiciosos que los divisables desde sus
campanarios mudjares.
Rafael LAPESA
El espaol moderno y contemporneo
Crtica
Cules son las causas ms importantes
de la presencia de extranjerismos
en el lenguaje cientfico y tcnico?
Explica la postura de Rafael Lapesa a este
respecto. Qu propuestas hace el autor
en este texto?
Realiza un resumen de este fragmento
del profesor Lapesa.
Crees que los medios de comunicacin
de masas contribuyen de alguna manera
a la difusin de los neologismos? Por qu?
Ilustra tu respuesta con ejemplos.
Cita nombres comunes del lenguaje ordinario
que provengan de marcas registradas.
Pon ejemplos de extranjerismos innecesarios
y sustityelos por expresiones castellanas.
6
5
4
3
2
1
134 8. Semntica y lxico
M
A
T
E
R
I
A
L

F
O
T
O
C
O
P
I
A
B
L
E

/


O
x
f
o
r
d

U
n
i
v
e
r
s
i
t
y

P
r
e
s
s

E
s
p
a

a
,
S
.
A
.
ACTI VI DADES
8
0B1LLLP(2008).08+s 3/6/08 18:42 Pgina 134
REFUERZO
5. Neologismos
Seala los neologismos presentes
en estos textos, reconoce de qu tipo son
y, si corresponde, di cmo se han formado.
Explica el significado de los neologismos
identificados en la actividad 1.
Indica algunos de los casos en que ha habido
cambios semnticos y menciona las figuras
retricas que aparecen.
Seala algunos trminos pertenecientes
a lenguajes especializados.
4
3
2
1
135 8. Semntica y lxico
M
A
T
E
R
I
A
L

F
O
T
O
C
O
P
I
A
B
L
E

/


O
x
f
o
r
d

U
n
i
v
e
r
s
i
t
y

P
r
e
s
s

E
s
p
a

a
,
S
.
A
.
ACTI VI DADES
8
A continuacin, se reproducen titulares publicados en el diario El Pas y se proponen activi -
dades relacionadas con los fenmenos lxico-semnticos estudiados en esta unidad.
Vinoterapia en pareja
Y la economa salv el 11-S
La cuna del hip-hop se salva
Lorenzo se estrena
con pole
Vuelve ABARTH,
el tuning legendario
Tanger, un fin de semana bajo coste
Mam, quiero ser punk
Los policias antimafia, incomunicados en comisaria
Cae una red que fabricaba al da 80000 copias pirata
Actuaban a la carta y servan a los manteros de un da para otro
El sndrome de la clase turista ya alarma a la OMS
Bolsos XXL para todos
Marketing online en Espaa
Para que funcione la globalizacin
Detectives oenegeros
Graffitero de interior
Un chip podria diagnosticar el mal celiaco
De Eurovisin a Frikivisin
Comerciantes y vecinos
abrazan la videovigilancia
EE UU desliga la ayuda antisida de la castidad
La brecha digital en Espaa: carrete o memoria?
La ciberdelincuencia se traslada a la web
Uno de los servidores ha sido neutralizado
Un western rural sobre incomunicacion y violencia
Tres detenidos tras un robo por alunizaje
en una perfumera
Millones de navegantes comparten fotos, vdeos
y artculos a travs de Internet
Bruselas quiere castigar a los empresarios que empleen
a sin papeles
La OMS destaca la dificultad de conseguir medicamentos
de ltima generacin baratos
Cientos de sin papeles esperan al alba en
plazas de Madrid a ser reclutados
para trabajar en urbanizaciones
Diez piratas controlan el 80%
del correo basura que circula por Internet
Castilla y Len ha dejado de aplicar
el veneno antirroedores
El BBVA dota con 200 millones una
fundacin para microcrditos
El Congreso de la Lengua debate en su clausura
sobre el ciberespacio
Un ataque pirata convierte ms de 11 000 webs
en focos de virus
Un temporal de troyanos convierte
400000 ordenadores en zombies
Experiencia sensorial en el spa
Motera tenas que ser
Trono antiestrs
El sablista estresado
0B1LLLP(2008).08+s 3/6/08 18:42 Pgina 135
ORTOGRAFA
6. Uso de x y s
La letra x representa sonidos diferentes:
Si se encuentra en final de palabra o en posicin intervoclica representa el grupo conso-
nntico ks: taxi, relax.
En posicin inicial de palabra, se pronuncia frecuentemente como s: xerografa.
En posicin final de slaba, se pronuncia como s o ks: extremo.
Adems de estas diferencias, normalmente se pronuncia de manera relajada, como gs, o inclu-
so s, por lo cual son frecuentes los errores ortogrficos entre s y x.
Existen solo unas pocas reglas para el uso de x:
Las palabras que empiezan por los prefijos ex- y extra-: excntrico, extrajudicial.
Las palabras que empiezan por las races clsicas xeno- (extranjero), xero- (seco, rido)
y xilo- (madera): xenfobo, xerfila, xilfono.
Las palabras que empiezan por la slaba ex- seguida del grupo -pr- y muchas que tengan la
misma slaba inicial seguida del grupo -pl-: expresar, explicar. En el segundo caso, se excluyen
esplendor y sus derivados, entre otras voces.
Coloca s o x, segn corresponda: 1
e__acerbar
e__ageracin
e__cabroso
e__cafandra
e__altacin
e__calofriante
e__ange
e__camotear
e__caparate
e__nime
e__capulario
e__asperado
e__carcelar
e__terminar
e__cavacin
e__caqueo
e__cedente
e__celente
e__caramuza
e__centricidad
e__cepcin
e__cardar
e__cepto
e__carmentar
e__ceso
e__carpado
e__casez
e__cesivo
e__catimar
e__citable
e__clamar
e__cayola
e__clusin
e__tincin
e__clusivo
e__cptico
e__clarecer
e__cluyente
e__clavo
e__comulgar
e__clerosis
e__coriacin
e__crecin
e__cocer
e__tintor
e__cremento
e__culpar
e__cursin
e__coger
e__cusa
e__colapio
e__cusar
e__collo
e__combro
e__ecrable
e__corbuto
e__egesis
e__tirpar
e__encin
e__foliacin
e__coria
e__equias
e__foliante
e__corpin
e__halacin
e__cote
e__haustivo
e__cotilla
e__cozor
e__hibicin
e__hortar
e__humacin
e__cudriar
e__igencia
e__iguo
e__torsionador
e__crpulo
e__iliado
e__imente
e__crutar
e__culido
e__istencial
__ito
e__cupir
e__currir
__odo
e__traccin
e__geno
e__onerar
e__orbitante
e__fnter
e__orcismo
e__pantoso
e__tico
e__pandir
e__parcir
e__pectculo
e__pansivo
e__pecfico
e__patriar
e__tradicin
e__pectacin
e__pectro
e__pesura
e__pectativa
e__trafino
e__pectorar
e__pedicin
e__plndido
e__pediente
e__polear
e__tralimitarse
e__pedir
e__peler
e__pontneo
e__pendedor
e__pulgar
e__traeza
e__periencia
e__putar
e__perimental
e__queje
e__perto
e__piacin
e__planada
e__playarse
e__quilar
e__plicable
e__plicitar
e__plorar
e__quivo
e__talactita
e__plosin
e__treo
e__poliar
e__trarradio
e__ponente
e__portar
e__presar
e__tereotipo
e__tigma
e__presionismo
e__presivo
e__trabismo
e__trafalario
e__propiar
e__trangular
e__travagancia
e__puesto
e__pugnacin
e__pulsar
e__purgar
e__quisitez
e__tremidad
e__tratagema
e__tender
e__trategia
e__tenuante
e__trato
e__uberante
e__teriorizar
e__croto
136 8. Semntica y lxico
M
A
T
E
R
I
A
L

F
O
T
O
C
O
P
I
A
B
L
E

/


O
x
f
o
r
d

U
n
i
v
e
r
s
i
t
y

P
r
e
s
s

E
s
p
a

a
,
S
.
A
.
ACTI VI DADES
8
Construye oraciones con las siguientes palabras parnimas; si desconoces algn significado, bscalo
en el diccionario:
a) esotrico / exotrico c) esttico / exttico e) espiar / expiar g) espirar / expirar
b) esclusa / exclusa d) contesto / contexto f) estirpe / extirpe h) laso / laxo
2
0B1LLLP(2008).08+s 3/6/08 18:42 Pgina 136
Evaluacin 8
131 8. Semntica y lxico
1. Resume la historia que se narra en el artculo El negro, de
la periodista Rosa Montero.
2. Explica los elementos de los que se compone el signo
lingstico, utilizando como ejemplo una palabra extrada
del texto.
3. Autoservicio es una traduccin de la expresin inglesa
self-service. De qu tipo de neologismo se trata? Pon
otros ejemplos de la misma clase.
4. Menciona otras clases de prstamos que se hayan incor-
porado recientemente y pon tres ejemplos de cada uno.
Incluye los que aparecen en el texto en la clase que
corresponda.
5. Localiza en el texto la palabra paritariamente y explica
cmo est formada. Cul es el significado de este tr-
mino en el contexto en el que aparece? Enuncia los
procedimientos que ofrece la lengua para crear nuevas
palabras y cita ejemplos de cada uno.
6. Escribe ejemplos de trminos sinnimos, antnimos,
homnimos y polismicos a partir de palabras tomadas
del texto.
7. Menciona los sinnimos contextuales o referenciales
con los que se designa en el artculo a los protagonistas
de la historia.
8. Por qu se producen cambios de significado? Qu
formas retricas adoptan? Escribe algunos ejemplos de
estos cambios semnticos.
9. Seala las caractersticas de los tecnicismos y explica su
formacin segn su origen y en funcin de los proce-
dimientos de creacin.
10. Expresa tu opinin argumentada sobre la incesante intro-
duccin de extranjerismos en la actualidad. Incluye refle-
xiones sobre quines los difunden, en qu mbitos de la
vida cotidiana abundan especialmente y cul es su
arraigo, comprensin y uso en la mayora de los hablantes.
Estamos en un comedor estudiantil de una universidad
alemana. Una alumna rubia e inequvocamente germana
adquiere su bandeja con el men en el mostrador del auto-
servicio y luego se sienta a una mesa. Entonces advierte
que ha olvidado los cubiertos y vuelve a levantarse a
cogerlos. Al regresar, descubre con estupor que un chico
negro, probablemente subsahariano por su aspecto, se ha
sentado en su lugar y est comiendo de su bandeja.
De entrada, la muchacha se siente desconcertada y agre -
dida; pero enseguida corrige su pensamiento y supone
que el africano no est acostumbrado al sentido de la
propiedad privada y de la intimidad del europeo, o incluso
que quiz no disponga de dinero suficiente para pagarse la
comida, aun siendo esta barata para el elevado estndar de
vida de nuestros ricos pases. De modo que la chica decide
sentarse frente al tipo y sonrerle amistosamente. A lo cual,
el africano contesta con otra blanca sonrisa. A continua-
cin, la alemana comienza a comer de la bandeja intentan-
do aparentar la mayor normalidad y compartindola con
exquisita generosidad y cortesa con el chico negro. Y as, l
se toma la ensalada, ella apura la sopa, ambos pinchan
paritariamente del mismo plato de estofado hasta acabarlo
y uno da cuenta del yogur y la otra de la pieza de fruta.
Todo ello trufado de mltiples sonrisas educadas, tmidas
por parte del muchacho, suavemente alentadoras y com-
prensivas por parte de ella.
Acabado el almuerzo, la alemana se levanta en busca de
un caf. Y entonces descubre, en la mesa vecina detrs
de ella, su propio abrigo colocado sobre el respaldo de
una silla y una bandeja de comida intacta.
Dedico esta historia deliciosa, que adems es autntica, a
todos los espaoles que, en el fondo, recelan de los inmi-
grantes y les consideran individuos inferiores. A todas
esas personas que, aun bienintencionadas, les observan
con condescendencia y paternalismo. Ser mejor que nos
libremos de los prejuicios o corremos el riesgo de hacer
el mismo ridculo que la pobre alemana, que crea ser el
colmo de la civilizacin mientras el africano, l s inmen-
samente educado, la dejaba comer de su bandeja y tal vez
pensaba: Pero qu chiflados estn los europeos.
Rosa MONTERO
El Pas, 17 de mayo 2005
') !#, *
Das de verano, por Dale Kennington.
0B1LLLA.08 7/4/08 18:03 Pgina 131
REFUERZO
1. Coherencia, cohesin
y adecuacin textuales
A continuacin, figuran tres fragmentos en los que puedes observar las propiedades textuales
bsicas: coherencia, cohesin y adecuacin.
Explica por qu es coherente el texto 1
atendiendo al tema, a la relacin entre
las proposiciones y a las presuposiciones.
Indica cmo se da la conexin lxica
en el fragmento de Vidal Lamquiz.
Seala los elementos decticos del texto 2
y explica en cada caso de qu tipo de deixis
se trata.
Indica en el fragmento de Juan Luis Cebrin
la presencia de un conector y di de qu tipo es.
Busca en ese mismo texto ejemplos de anforas.
Analiza todos los elementos que aseguran
la cohesin del texto 3.
Son adecuados los tres textos anteriores?
Justifica tu respuesta.
7
6
5
4
3
2
1
149 9. El texto
M
A
T
E
R
I
A
L

F
O
T
O
C
O
P
I
A
B
L
E

/


O
x
f
o
r
d

U
n
i
v
e
r
s
i
t
y

P
r
e
s
s

E
s
p
a

a
,
S
.
A
.
ACTI VI DADES
9
Texto 1
La lengua es ese instrumento codificado de
comunicacin verbal entre las personas. Adqui rida
por herencia tras el nacimiento, perfeccio nada por
la experiencia y el estudio, la lengua es algo que
lleva puesto toda persona, como grficamente
se ha dicho.
Tanto si decimos que el hombre primitivo inven-
t el lenguaje como si decimos que la invencin del
lenguaje hizo que un primate inteligente se convir-
tiera en hombre, decimos la misma cosa (B. Malm-
berg, 1969: 13). En los criterios de la ciencia actual,
tanto para los filsofos desde Wittgenstein como
para los lingistas desde Chomsky, el hombre es ho-
mo loquens antes que faber o sapiens (M. Bunge,
1983: 9). En su experiencia social la persona huma-
na se sita en una prctica comunicativa de con-
texto comn que se comparte. Es decir, est instala-
da en el mundo de la vida que se interpreta en la
intersubjetividad de los miembros de una sociedad
organizada.
Vidal LAMQUIZ
El enunciado textual. Anlisis lingstico del discurso, Ariel
Texto 3
Cuando hablamos de elementos del texto, podemos aludir tanto a elementos del significante, como a
elementos del significado. Por supuesto que los segundos estn implcitos y apare cen como un de sarrollo
a partir de los primeros. Cualquier anlisis del texto debe por tanto tener su punto de partida en el hecho
de que el texto est constituido por una sucesin de significantes grficos o monemas, formados por gru-
pos de grafemas. Y es esta sucesin, invariable si se prescinde del desgaste producido por la tradicin, la
que desarrolla despus, en el acto de la lectura, los significados.
Cesare SEGRE
Principios de anlisis del texto literario, Crtica
Texto 2
Hubiera querido disimular. Le habra gusta-
do hacer como en las pelculas, mirar fija e inex-
presivamente a la chica, dejar correr el silencio
algunos segundos, tamborilear con delicadeza
sus dedos sobre la mesa del bar, reposar la taza
otra vez sobre su plato y decir escueto: No s de
qu me est usted hablando. Pero, en cambio,
la glotis equivoc su camino, se atragant con
el caf con leche y se puso colorado como un pi-
miento mientras tosa con estrpito.
Comprender que de eso no puedo infor-
marle. Es un secreto militar articul al fin a
duras penas.
Exactamente ese es el problema en este
caso, mi amigo. Aqu todo es un secreto, incluso
el nombre de la chica. No me la presenta?
Juan Luis CEBRIN
La isla del viento, Alfaguara
0B1LLLP(2008).09+s 3/6/08 16:02 Pgina 149
REFUERZO
2. Estructuradores de la informacin
y otros marcadores
Resume el contenido de los textos 1 y 2.
Cul es el tema comn que los vincula?
Reconoce en ellos estructuradores de la
informacin.
A qu subtipo pertenecen? Explica su valor.
Explica el contenido del tercer fragmento.
Qu tipo de marcadores aparecen en l?
Seala su valor.
Qu formas tienen los marcadores presentes
en los tres fragmentos?
6
5
4
3
2
1
150 9. El texto
M
A
T
E
R
I
A
L

F
O
T
O
C
O
P
I
A
B
L
E

/


O
x
f
o
r
d

U
n
i
v
e
r
s
i
t
y

P
r
e
s
s

E
s
p
a

a
,
S
.
A
.
Los textos ensaysticos abundan en estructuradores de la informacin y otros tipos
de marcadores. Corrobralo en estos fragmentos de la obra Las preguntas de la vida, de
Fernando Savater.
ACTI VI DADES
9
Texto 1
Quiz parezca extrao que un libro que quiere
iniciar en cuestiones filosficas se abra con un cap-
tulo dedi cado a la muerte. No desanimar un tema
tan lgubre a los nefitos? No sera mejor comen-
zar hablando de la libertad o del amor? Pero ya he
indicado que me propongo invitar a la filosofa a
partir de mi propia expe riencia intelectual y en mi
caso fue la revelacin de la muerte de mi muer-
te como certidumbre lo que me hizo ponerme a
pensar. Y es que la evidencia de la muerte no solo le
deja a uno pensativo, sino que le vuelve a uno pen-
sador. Por un lado, la conciencia de la muerte nos
hace madurar personalmente: todos los nios se
creen inmortales (los muy pequeos incluso piensan
que son omnipotentes y que el mundo gira a su alre-
dedor; salvo en los pases o en las familias atroces
donde los nios viven desde muy pronto amenaza-
dos por el exterminio y los ojos infantiles sorprenden
por su fatiga mortal, por su anormal veterana),
pero luego crecemos cuando la idea de la muerte
crece dentro de nosotros. Por otro lado, la certidum-
bre personal de la muerte nos humaniza, es decir, nos
convierte en verdaderos humanos, en mortales.
Entre los griegos humano y mortal se deca con
la misma palabra, como debe ser.
Texto 3
Parece evidente que el peso de los condicionamientos subjetivos vara grandemente segn el campo de verdad que
en cada caso estemos considerando: si de lo que hablamos es de mitologa, de gastronoma o de expresin potica, el
peso de nuestra cultura o nuestra idiosincrasia personal es mucho ms concluyente que cuando nos referimos a ciencias
de la naturaleza o a los principios de la convivencia humana. En cualquier caso, tambin para determinar hasta qu
punto nuestros conocimientos estn teidos de subjetivismo necesitamos un punto de vista objetivo desde el que com-
pararlos unos con otros y todos con una cierta realidad ms all de ellos a la que se refieren! En fin, hasta para des-
confiar de los criterios universales de razn y de verdad necesitamos algo as como una razn y una verdad que sirvan
de criterio universal. Sin embargo, la aportacin ms valiosa del relativismo consiste en subrayar la imposibilidad de
establecer una fuente ltima y absoluta de la que provenga todo conocimiento verdadero.
Fernando SAVATER
Las preguntas de la vida, Ariel
Texto 2
Inquietarse por los aos y los siglos en que ya no
estaremos entre los vivos resulta tan caprichoso co-
mo preocuparse por los aos y los siglos en que an
no habamos venido al mundo. Ni antes nos doli
no estar ni es razonable suponer que luego nos do-
ler nuestra definitiva ausencia. En el fondo, cuan-
do la muerte nos hiere a travs de la imaginacin
pobre de m, todos tan felices disfrutando del sol
y del amor; todos menos yo, que ya nunca ms,
nunca ms! es precisamente ahora que toda-
va estamos vivos. Quiz deberamos reflexionar un
poco ms sobre el asombro de haber nacido, que
es tan grande como el espantoso asombro de la
muerte. Si la muerte es no ser, ya la hemos vencido
una vez: el da que nacimos. Es el propio Lucrecio
quien habla en su poema filosfico de la mors ae-
terna, la muerte eterna de lo que nunca ha sido ni
ser. Pues bien, nosotros seremos mortales pero de
la muerte eterna ya nos hemos escapado. A esa
muerte enorme le hemos robado un cierto tiempo
los das, meses o aos que hemos vivido, cada
instante que seguimos viviendo y ese tiempo pa-
se lo que pase siempre ser nuestro, de los triunfal-
mente nacidos, y nunca suyo, pese a que tambin
debamos luego irremediablemente morir.
0B1LLLP(2008).09+s 3/6/08 16:03 Pgina 150
ORTOGRAFA
3. Coherencia y signos
de puntuacin
Recuerda:
El punto separa oraciones dentro de un texto.
La palabra que sigue a un punto y seguido o a un punto y aparte se escribe con mayscula inicial.
La coma marca pausas internas dentro de una oracin.
El punto y coma marca una pausa mayor que la de la coma y menor que la del punto y seguido.
Despus de los dos puntos suele figurar una aclaracin o la consecuencia de lo dicho previamente.
Vuelve a escribir el texto de Umberto Eco
colocando punto, coma, punto y coma y dos
puntos donde corresponda y explica en cada
caso por qu lo haces.
Aade las tildes que faltan en el texto anterior.
Completa con coma, punto y coma y dos puntos
las siguientes oraciones:
a) Nuestra profesora adems de qumica
es fsica.
b) Todo en ella era muy hermoso sus ojos
su boca su pelo.
c) Caballero por favor decdase usted
por alguno de estos perfumes cerramos
dentro de dos minutos.
d) Limpia la lentilla as qutatela despus de
parpa dear colcala en la palma frtala con
limpiador diario y aclrala con solucin
salina.
e) El recuento para mi sorpresa arroj
este resultado el viaje a Pars tres votos
la excursin por Italia dos las siete noches
de Ibiza ninguno y los siete das de
senderismo por Sierra Nevada veintisiete.
Explica la diferencia de significado entre estos
pares de oraciones:
a) Los vecinos, a quienes les toc la lotera,
se han separado.
Los vecinos a quienes les toc la lotera
se han se parado.
b) Han decidido que no; se instalar
un portero automtico nuevo.
Han decidido que no se instalar
un portero automtico nuevo.
c) Los perros, hambrientos, robaron
la comida.
Los perros hambrientos robaron
la comida.
d) Me sorprende esa respuesta; de tu hermana
no me fo.
Me sorprende esa respuesta de tu hermana;
no me fo.
e) Sus sobrinas sacaron gran beneficio de la
herencia; la mayor, parte de los inmuebles.
Sus sobrinas sacaron gran beneficio de la
herencia: la mayor parte de los inmuebles.
4
3
2
1
Supongamos que el seor Sigma en el curso de un via-
je a Paris empieza a sentir molestias en el vientre utilizo
un termino generico porque el seor Sigma por el
momento tiene una sensacion confusa se concentra e
intenta definir la molestia ardor de estomago? espas-
mos? dolores viscerales? intenta dar nombre a unos esti-
mulos imprecisos y al darles un nombre los culturaliza es
decir encuadra lo que era un fenomeno natural en unas
rubricas precisas y codificadas o sea intenta dar a una
experiencia personal propia una calificacion que la haga
similar a otras ex periencias ya expresadas en los libros de
medicina o en los articulos de los periodicos por fin descu-
bre la palabra que le parece adecuada esta palabra vale
por la molestia que siente y dado que quiere comunicar
sus molestias a un medico sabe que podra utilizar la pala-
bra (que el medico esta en condiciones de entender) en
vez de la molestia (que el medico no siente y que quizas
no ha sen tido nunca en su vida) Todo el mundo estara dis-
puesto a reconocer que esta palabra que el seor Sigma
ha individualizado es un signo pero nuestro problema es
mas complejo el seor Sigma decide pedir hora a un
medico consulta la guia telefonica de Paris unos signos
graficos precisos le indican quienes son medicos y como
llegar hasta ellos sale de casa busca con la mirada una
seal particular que conoce muy bien entra en un bar si se
tratara de un bar italiano intentaria localizar un angulo
proximo a la caja donde podria estar un telefono de color
metalico pero como sabe que se trata de un bar frances
tiene a su disposicion otras reglas interpretativas del
ambiente busca una escalera que descienda al sotano
sabe que en todo bar parisino que se respete alli estan los
lavabos y los telefonos es decir el ambiente se presenta
como un sistema de signos orientadores que le indican
desde donde podra hablar
Umberto ECO
Signo, Labor
151 9. El texto
M
A
T
E
R
I
A
L

F
O
T
O
C
O
P
I
A
B
L
E

/


O
x
f
o
r
d

U
n
i
v
e
r
s
i
t
y

P
r
e
s
s

E
s
p
a

a
,
S
.
A
.
ACTI VI DADES
9
Un mundo decodificable
0B1LLLP(2008).09+s 3/6/08 16:03 Pgina 151
Evaluacin
9
143 9. El texto
1. Cul es el tema del texto que acabas de leer? Explica el
contenido de cada uno de los prrafos en los que se
estructura.
2. Analiza la coherencia del fragmento atendiendo a los
siguientes aspectos.
Presuposiciones.
Conocimiento del mundo.
Implicaciones.
Marco.
3. Reconoce en el texto un elemento anafrico y seala su
referente.
4. Analiza la deixis presente en esta noticia de El Pas.
5. En el texto hay dos elipsis: localzalas y di qu elementos
se han omitido.
6. En el fragmento tambin puede observarse un caso de
cohesin lxica por hiperonimia. Selalo.
7. Analiza la cohesin temporal manifestada por el empleo
de las formas verbales en el texto.
8. En el fragmento se observan varios marcadores: reco-
ncelos, clasifcalos y explica su funcin. Aade t
ejemplos de la o las clases que no aparecen en el texto.
9. Consideras el texto apropiado, adecuado y oportuno?
Justifica tu respuesta.
10. Elabora un esquema de todos los elementos que permi-
ten asegurar la cohesin de un texto. Incluye ejemplos.
AA)A AAA-A.
A. A-u AAu -A A-A. A AAAAA A) AA-( AAA, )
AAAA-( uAA A-A.
AA A-A A-A. AA- Au( A)AA AA1A.
A A-AA A-A. AAA) 1A-. AAAA) AAA.
Ayer, tras recibir el parte de incidencias,
en los despachos de la DGT cunda el
desnimo. Sobre todo porque los datos
negativos, que han roto la tendencia
positiva que se arrastraba desde que
comenz el ao, han llegado justo cuan-
do se haban intensificado las campaas
de concienciacin. A los desplazamientos
de fin de semana se suman los de salida
o retorno por cambio de quincena, los
viajes a poblaciones en fiestas y los de
inmigrantes portugueses y magrebes
que se dirigen a sus pases para pasar las
vacaciones desde Espaa o Francia.
An as, Trfico entiende que no hay
razones para alarmarse. En primer lugar,
porque no se pueden tomar como refe-
rencia los accidentes que han ocurrido en
solo tres das, y menos comparndolos
con los del mismo fin de semana del ao
pasado, que fue especialmente bueno,
segn advirti un portavoz de la DGT.
Tampoco se pueden sacar conclusiones
objetivas, de acuerdo con Trfico, a partir
de las cifras de las dos primeras semanas
de agosto. El ao pasado, adems, se aca-
baba de estrenar el carn por puntos, que
entr en vigor en julio de 200. La DGT
interpreta que la novedad estimul la
prudencia de los conductores.
En definitiva, los responsables de Trfico
se consuelan observando que el balance
anual de mortalidad sigue siendo posi-
tivo: hasta el pasado domingo, los acci-
dentes de trfico haban causado 1 99
fallecidos. A estas alturas, el ao pasado
ya haba 1 891 muertos, con lo cual el
descenso se sita en el 10,1 %.
El ltimo fin de semana, por otro lado,
ha vuelto a confirmar que corren malos
tiempos para la seguridad de los moto-
ristas. De los 37 fallecidos, 11 viajaban a
bordo de motocicletas, el nico vehculo
de todo el parque mvil en el que la
siniestralidad aumenta ao tras ao.
El Pas, 14 de agosto de 2007
' ) <( !, *! 0: .i( - ( *, . ' !- ! . , "i* , !!/) 8 %!) # *- . *
Retencin en un da de operacin salida.
0B1LLLA.09 7/4/08 18:53 Pgina 143
REFUERZO
1. Textos y modalidades
En un mismo libro, pueden presentarse las distintas modalidades textuales bsicas: narracin,
descripcin, exposicin y argumentacin. Es el caso de los textos 2, 3 y 4, que pertenecen a la
misma obra, Las preguntas de la vida, de Fernando Savater. El texto 1 es un fragmento de la novela
La hora violeta, de Montserrat Roig.
161 10. Las modalidades textuales
M
A
T
E
R
I
A
L

F
O
T
O
C
O
P
I
A
B
L
E

/


O
x
f
o
r
d

U
n
i
v
e
r
s
i
t
y

P
r
e
s
s

E
s
p
a

a
,
S
.
A
.
ACTI VI DADES
10
Texto 1
La mar embravecida golpea las rocas. La espuma vuela y lame furiosamente la arena. La mujer
del pescador, que lleva una bata de cuadritos y red en la cabeza, mira con insistencia hacia el faro,
hacia La Mola. El tiempo se ha revuelto y el marido no regresa. Oigo como la mujer del pescador ex-
plica a una extranjera que la barca es pequea, pero fuerte. La extranjera no sabe qu decirle, la con-
suela y veo que procura prepararla para la resignacin. Otros extranjeros, indiferentes, mojan pan
en unos enormes tazones de leche. El cielo es de color gris y, por poniente, est moteado de manchas
negras. Las olas insisten, como si alguna fuerza oscura las empujase. Una tras otra van muriendo, fu-
riosas y cansadas, sobre los arrecifes. En el mar hay una escala, tenue, matizada, de colores: verde bo-
tella-azul celeste-azul cobalto.
Montserrat ROIG
La hora violeta
Alianza
Texto 2
Las leyendas ms antiguas no pretenden consolarnos de la
muerte sino solo explicar su inevitabilidad. La primera gran epo-
peya que se conserva, la historia del hroe Gilgamesh, se com-
puso en Sumeria aproximadamente 2700 a. C. Gilgamesh y su
amigo Enkidu, dos valientes guerreros y cazadores, se enfrentan
a la diosa Isthar, que da muerte a Enkidu. Entonces Gilgamesh
emprende la bsqueda del remedio de la muerte, una hierba
mgica que renueva la juventud para siempre, pero la pierde
cuando est a punto de conseguirla. Despus aparece el espritu
de Enkidu, que explica a su amigo los sombros secretos del reino
de los muertos, al cual Gilgamesh se resigna a acudir cuando
llegue su hora. Ese reino de los muertos no es ms que un sinies-
tro reflejo de la vida que conocemos, un lugar profundamente
triste. Lo mismo que el Hades de los antiguos griegos. En la Odi-
sea de Homero, Ulises convoca los espritus de los muertos y
entre ellos acude su antiguo compaero Aquiles. Aunque su
sombra sigue siendo tan majestuosa entre los difuntos como lo
fue entre los vivos, le confiesa a Ulises que preferira ser el ltimo
porquerizo en el mundo de los vivos que rey en las orillas de la
muerte. Nada deben envidiar los vivos a los muertos. En cambio,
otras religiones posteriores, como la cristiana, prometen una
existencia ms feliz y luminosa que la vida terrenal para quienes
hayan cumplido los preceptos de la divinidad (por contrapar-
tida, aseguran una eternidad de refinadas torturas a los que han
sido desobedientes).
0B1LLLP(2008).10+s 3/6/08 18:44 Pgina 161
A qu modalidad textual corresponde cada
uno de los fragmentos? Justifica tu respuesta
atendiendo al contenido y a la finalidad.
En el texto 1 predomina una de las
modalidades, pero incluye tambin otra;
de qu manera lo hace?
El texto 2 pertenece a una de las modalidades
textuales bsicas, pero incluye otra de ellas.
Razona esta afirmacin.
Realiza un resumen del texto 2 y explica
su estructura.
Enuncia el tema y la tesis del texto 3.
Determina los argumentos que sustentan
dicha tesis.
Extrae la idea principal del texto 4
Analiza las caractersticas lingsticas
y textuales de los textos 1 y 3.
8
7
6
5
4
3
2
1
162 10. Las modalidades textuales
R
E
F
U
E
R
Z
O
1
.

T
e
x
t
o
s

y

m
o
d
a
l
i
d
a
d
e
s
M
A
T
E
R
I
A
L

F
O
T
O
C
O
P
I
A
B
L
E

/


O
x
f
o
r
d

U
n
i
v
e
r
s
i
t
y

P
r
e
s
s

E
s
p
a

a
,
S
.
A
.
A
C
T
I
V
I
D
A
D
E
S
1
0
Texto 3
Las plantas y los animales no son mortales porque no saben que van a morir, no saben que tienen que
morir: se mueren pero sin conocer nunca su vinculacin individual, la de cada uno de ellos, con la muerte.
Las fieras presienten el peligro, se entristecen con la enfermedad o la vejez, pero ignoran (o parece que ig-
noran?) su abrazo esencial con la necesidad de la muerte. No es mortal quien muere, sino quien est seguro
de que va a morir. Aunque tambin podramos decir que ni las plantas ni los animales estn por eso mismo
vivos en el mismo sentido en que lo estamos nosotros. Los autnticos vivientes somos solo los mortales,
porque sabemos que dejaremos de vivir y que en eso precisamente consiste la vida. Algunos dicen que los
dioses inmortales existen y otros que no existen, pero nadie dice que estn vivos: solo a Cristo se le ha lla-
mado Dios vivo y eso porque cuentan que encarn, se hizo hombre, vivi como nosotros y como noso -
tros tuvo que morir.
[] Es precisamente la certeza de la muerte la que hace la vida mi vida, nica e irrepetible algo
tan mortalmente importante para m. Todas las tareas y empeos de nuestra vida son formas de resis-
tencia ante la muerte, que sabemos ineluctable. Es la conciencia de la muerte la que convierte la vida en
un asunto muy serio para cada uno, algo que debe pensarse. Algo misterioso y tremendo, una especie de
milagro precioso por el que debemos luchar, a favor del cual tenemos que esforzarnos y reflexionar. Si la
muerte no existiera habra mucho que ver y mucho tiempo para verlo pero muy poco que hacer (casi
todo lo hacemos para evitar morir) y nada en qu pensar.
Texto 4
Recuerdo muy bien la primera vez que comprend de veras que antes o despus tena que morirme.
Deba andar por los diez aos, nueve quiz, eran casi las once de una noche cualquiera y estaba ya acos-
tado. Mis dos hermanos, que dorman conmigo en el mismo cuarto, roncaban apaciblemente. En la ha-
bita cin contigua mis padres charlaban sin estridencias mientras se desvestan y mi madre haba puesto
la radio que dejara sonar hasta tarde, para prevenir mis espantos nocturnos. De pronto me sent a oscuras
en la cama: yo tambin iba a morirme!, era lo que me tocaba, lo que irremediablemente me correspon-
da!, no haba escapatoria! No solo tendra que soportar la muerte de mis dos abuelas y de mi querido
abuelo, as como la de mis padres, sino que yo, yo mismo, no iba a tener ms remedio que morirme. Qu
cosa tan rara y terrible, tan peligrosa, tan incomprensible, pero sobre todo qu cosa tan irremediable-
mente personal!
[] A los diez aos cree uno que todas las cosas importantes solo les pueden pasar a los mayores: re-
pentinamente se me revel la primera gran cosa importante de hecho, la ms importante de todas
que sin duda ninguna me iba a pasar a m. Iba a morirme, naturalmente dentro de muchos, muchsimos
aos, despus de que se hubieran muerto mis seres queridos (todos menos mis hermanos, ms pequeos
que yo).
Fernando SAVATER
Las preguntas de la vida
Ariel
0B1LLLP(2008).10+s 3/6/08 18:44 Pgina 162
REFUERZO
2. Narraciones literarias
Quim Monz es uno de los grandes narradores de la actualidad. Sus cuentos, que han sido tra-
ducidos a muchas lenguas, han merecido numerosos premios. A continuacin, reproducimos
un cuento completo y un fragmento de otro, ambos publicados por la editorial Anagrama en el
libro Ochenta y seis cuentos.
Realiza un esquema de las acciones principales
de cada uno de los textos.
Menciona los otros elementos propios de una
narracin y explica cmo se manifiestan en
estos textos.
Los dos relatos remiten a cuentos tradicionales.
Qu partes y elementos estn modificados?
En qu consiste el cambio y cul puede ser
la finalidad?
Analiza en ambos textos el narrador,
la perspectiva y seala con qu estilo se
presenta el discurso de los personajes.
Reconoce los rasgos lingsticos propios
de la narracin que aparecen en estos textos.
A partir de otro cuento tradicional
que recuerdes, crea uno nuevo alterando
el esquema narrativo o las caractersticas
de los elementos principales del relato.
6
5
4
3
2
1
163 10. Las modalidades textuales
M
A
T
E
R
I
A
L

F
O
T
O
C
O
P
I
A
B
L
E

/


O
x
f
o
r
d

U
n
i
v
e
r
s
i
t
y

P
r
e
s
s

E
s
p
a

a
,
S
.
A
.
ACTI VI DADES
10
La bella durmiente
En medio de un claro, el caballero ve el cuerpo de
la muchacha, que duerme sobre una litera hecha
con ramas de roble y rodeada de flores de todos los
colores. Desmonta rpidamente y se arrodilla a su
lado. Le coge una mano. Est fra. Tiene el rostro
blanco de una muerta. Y los labios finos y morados.
Consciente de su papel en la historia, el caballero la
besa con dulzura. De inmediato la muchacha abre
los ojos grandes, almendrados y oscuros, y lo mira:
con una mirada de sorpresa que enseguida (una vez
ha cavilado quin es y dnde est y por qu est all
y quin ser ese hombre que tiene al lado y que, su-
pone, acaba de besarla) se tie de ternura. Los labios
van perdiendo el tono morado y, una vez recobrado
el rojo de la vida, se abren en una sonrisa. Tiene unos
dientes bellsimos. El caballero no lamenta nada
tener que casarse con ella, como estipula la tradi-
cin. Es ms: ya se ve casado, siempre junto a ella,
compartindolo todo, teniendo un primer hijo, luego
una nena y por fin otro nio. Vivirn una vida feliz y
envejecern juntos.
Las mejillas de la muchacha han perdido la
blancura de la muerte y ya son rosadas, sensuales,
para morderlas. l se incorpora y le alarga las
manos, las dos, para que se coja a ellas y pueda le-
vantarse. Y entonces, mientras (sin dejar de mi-
rarlo a los ojos, enamorada) la muchacha (dbil por
todo el tiempo que ha pasado acostada) se incor-
pora gracias a la fuerza de los brazos masculinos, el
caballero se da cuenta de que (unos veinte o treinta
metros ms all, mucho antes de que el claro d
paso al bosque) hay otra muchacha dormida, tan
bella como la que acaba de despertar, igualmente
acostada en una litera de ramas de roble y rodeada
de flores de todos los colores.
El sapo
El prncipe inclina el cuerpo y adelanta la cara. El sapo est
justo frente a l. La papada se le hincha y deshincha sin cesar.
Ahora que lo ve tan cerca siente que lo invade el asco; pero no
tarda en reponerse y acerca los labios al morro del anfibio.
Mua.
En menos de una milsima de segundo, con un ruido
ensordecedor, el sapo se convierte en un prisma de cien mil
colores, que multiplica infinitamente las caras, hasta que
todas las caras y colores se convierten en una muchacha pre-
ciosa de cabellos de oro. Y una corona encima que demuestra
la nobleza de su linaje. Por fin el prncipe ha encontrado
a la mujer que siempre ha buscado, esa con la que compartir
el trono y la vida.
Por fin has llegado le dice ella. Si supieras cmo he
esperado al prncipe que deba librarme del hechizo.
Lo comprendo. Te he buscado siempre, desde que era
nio. Y siempre he sabido que te encontrara.
Se miran a los ojos, se cogen las manos. Es para siem-
pre, y los dos son conscientes de ello.
Era como si este momento no fuera a llegar nunca
dice ella.
Pues ya ha llegado.
S.
Qu bien, no?
Ests contento?
S. Y t?
Yo tambin.
El prncipe mira el reloj. Qu ms debe decirle? De qu
deben hablar? Debe invitarla enseguida a su casa o se lo
tomar a mal? En realidad no hay ninguna prisa. Tienen
toda la vida por delante.
En fin
S.
Ya ves
Tanto esperar y de repente, plaf, ya est.
S, ya est.
0B1LLLP(2008).10+s 3/6/08 18:44 Pgina 163
REFUERZO
3. Descripciones literarias
La descripcin es una modalidad presente en el gnero narrativo y, generalmente, muestra las
particularidades de la tendencia literaria a la que pertenecen las novelas en que se hallan
incluidas y ciertos rasgos caractersticos del estilo del escritor.
164 10. Las modalidades textuales
M
A
T
E
R
I
A
L

F
O
T
O
C
O
P
I
A
B
L
E

/


O
x
f
o
r
d

U
n
i
v
e
r
s
i
t
y

P
r
e
s
s

E
s
p
a

a
,
S
.
A
.
ACTI VI DADES
10
Texto 1
La noche le pareci interminable: dio vueltas y ms vueltas; apagaron la luz elctrica, los tranvas cesaron
de pasar, la plaza qued a oscuras!
Entre la calle de la Montera y la de Alcal iban y venan delante de un caf, con las ventanas iluminadas,
mujeres de trajes claros y pauelos de crespn, cantando, parando a los noctmbulos; unos cuantos chulos,
agazapados tras de los faroles, las vigilaban y charlaban con ellas, dndoles rdenes
Luego fueron desfilando busconas, chulos y celestinas. Todo el Madrid parsito, holgazn, alegre, abando-
naba en aquellas horas las tabernas, los garitos, las casas de juego, las madrigueras y los refugios del vicio, y
por en medio de la miseria que palpitaba en las calles, pasaban los trasnochadores con el cigarro encendido,
hablando, riendo, bromeando con las busconas, indiferentes a las agonas de tanto miserable desharrapado,
sin pan y sin techo, que se refugiaba temblando de fro en los quicios de las puertas.
Quedaban algunas viejas busconas en las esquinas, envueltas en el mantn, fumando
Tard mucho en aclarar el cielo; an de noche se armaron puestos de caf; los cocheros y los golfos se
acercaron a tomar su vaso o su copa. Se apagaron los faroles de gas.
Danzaban las claridades de las linternas de los serenos en el suelo gris, alumbrado vagamente por el pli-
do claror del alba, y las siluetas negras de los traperos se detenan en los montones de basura, encorvndose
para escarbar en ellos. Todava algn trasnochador plido, con el cuello del gabn levantado, se deslizaba
siniestro como un bho ante la luz, y mientras tanto comenzaban a pasar obreros El Madrid trabajador, y
honrado se preparaba para su ruda faena diaria.
Po BAROJA
La busca, Caro Raggio
Texto 2
Lul era una muchacha graciosa, pero no
bonita; tena los ojos verdes, oscuros, som-
breados por ojeras negruzcas; unos ojos que
a Andrs le parecieron muy humanos; la dis-
tancia de la nariz a la boca y de la boca a la
barba era en ella demasiado grande, lo que
le daba cierto aspecto simio; la frente peque-
a, la boca, de labios finos, con una sonrisa
entre irnica y amarga; los dientes blancos,
puntiagudos; la nariz un poco respingona, y
la cara plida, de mal color.
Lul demostr a Hurtado que tena gra-
cia, picarda e ingenio de sobra; pero le falta-
ba el atractivo principal de una muchacha:
la ingenuidad, la frescura, la candidez. Era un
producto marchito por el trabajo, por la
miseria y por la inteligencia. Sus dieciocho
aos no parecan juventud.
Po BAROJA
El rbol de la ciencia, Caro Raggio
0B1LLLP(2008).10+s 3/6/08 18:44 Pgina 164
Indica qu se describe en cada uno de los
textos y qu aspectos o elementos se tratan.
Qu tipo de descripciones son segn la
perspectiva del emisor? Menciona las
caractersticas lingsticas que demuestran
la pertenencia a una clase u otra.
Analiza los rasgos lingsticos generales
propios de la modalidad descriptiva
y ejemplifcalos con enunciados de
los textos.
Reconoce los recursos expresivos que
aparezcan en los fragmentos.
4
3
2
1
165 10. Las modalidades textuales
R
E
F
U
E
R
Z
O
3
.

D
e
s
c
r
i
p
c
i
o
n
e
s

l
i
t
e
r
a
r
i
a
s
M
A
T
E
R
I
A
L

F
O
T
O
C
O
P
I
A
B
L
E

/


O
x
f
o
r
d

U
n
i
v
e
r
s
i
t
y

P
r
e
s
s

E
s
p
a

a
,
S
.
A
.
A
C
T
I
V
I
D
A
D
E
S
1
0
Texto 4
Yo recordaba nebulosamente aquel antiguo jardn donde los mirtos seculares dibuja-
ban los cuatro escudos del fundador en torno de una fuente abandonada. El jardn y el
Palacio tenan esa vejez seorial y melanclica de los lugares por donde en otro tiempo
pas la vida amable de la galantera y del amor. Bajo la fronda de aquel laberinto, sobre
las terrazas y en los salones, haban florecido las risas y los madrigales, cuando las manos
blancas que en los viejos retratos sostienen apenas los paolitos de encaje, iban desho-
jando las margaritas que guardan el cndido secreto de los corazones. Hermosos y leja-
nos recuerdos! Yo tambin los evoqu un da lejano, cuando la maana otoal y dorada
envolva el jardn hmedo y reverdecido por la constante lluvia de la noche. Bajo el cielo
lmpido, de un azul herldico, los cipreses venerables parecan tener el ensueo de la vida
monstica. La caricia de la luz temblaba sobre las flores como un pjaro de oro, y la brisa
trazaba en el terciopelo de la yerba, huellas ideales y quimricas como si danzasen incre-
bles hadas.
Ramn M. DEL VALLE-INCLN
Sonata de otoo
Texto 3
Este es un casino amplio, nuevo, cmodo. Est rodeado de un jar-
dn; el edificio consta de dos pisos, con balcones de piedra torneada.
Primero aparece un vestbulo enladrillado de menuditos mosaicos
pintorescos; los motantes de las puertas cierran con vidrieras de
colores. Despus se pasa a un saln octgono; enfrente est el gabi-
nete de lectura, con una agradable sillera gris y estantes llenos de
esos libros grandes que se imprimen para ornamentacin de las
bibliotecas en que no lee nadie. A la derecha hay un gran saln vaco
(porque no hace falta tanto local), y a la izquierda otro gran saln
igual al anterior, donde los socios se renen con preferencia. Mesas
cuadradas y redondas de mrmol se hallan esparcidas ac y all,
alternando con otras de tapete verde; junto a la pared, corre un
ancho divn de peluche rojo; en un ngulo destaca un piano de cola,
y verdes jazmines cuajados de florecillas blancas festonean las ven-
tanas.
Son los primeros das de otoo; los balcones estn cerrados; el
viento mueve un leve murmullo en el jardn; poco a poco van llegan-
do los socios a su recreo de la noche; brillan las lmparas elctricas.
AZORN
Antonio Azorn
Ctedra
0B1LLLP(2008).10+s 3/6/08 18:44 Pgina 165
REFUERZO
4. Textos explicativo-
argumentativos
El desarrollo de las nuevas tecnologas ha producido cambios enormes, no solo en el campo
del conocimiento y de las comunicaciones, sino tambin en la vida cotidiana de las personas. El
siguiente texto de Joan Maj ofrece unas reflexiones sobre las posibilidades de la llamada
revolucin digital para crear un mundo mejor.
La sociedad del conocimiento: un mundo mejor?
Hemos aprendido a hacer lo que queramos con la informacin. A generar toda la
que deseemos, a almacenarla, a transmitirla, sin ningn lmite desde el punto de vista
del espacio, del tiempo o de la velocidad. Como consecuencia de ello hemos pasado de
un mundo en el que la informacin era un recurso escaso, a otro en el que es un recurso
abundante, incluso a menudo excesivo. Tambin hemos pasado de un mundo en el
que la informacin era un recurso estable, a otro en el que es un recurso enormemente
cambiante. Puesto que la informacin es la materia prima para que nuestro cerebro
elabore conocimientos, cada vez estamos en condiciones de adquirir ms (siempre
que no nos dejemos ahogar por el exceso de informacin y siempre que aprendamos
a actualizar constantemente los conocimientos que van quedando anticuados!).
Si esto es lo que ha ocurrido, vamos a ver qu cambios produce y producir en nuestra
sociedad.
Llamamos a esta sociedad futura la sociedad del conocimiento porque este va a
ser un elemento muy abundante, pero sobre todo porque va a ser el factor clave de
nuestro bienestar. Su gran cantidad, disponibilidad y capacidad innovadora han con-
vertido el conocimiento en un recurso econmico fundamental. El acceso al conoci-
miento se ha convertido en un elemento clave de la estrategia de las empresas y en un
componente fundamental de nuestro bienestar personal. Las empresas sustituyen
constantemente la utilizacin de recursos materiales por la aplicacin de nuevos co-
nocimientos y nosotros vamos aumentando ms y ms la parte de nuestro presu-
puesto personal que dedicamos a acceder a elementos inmateriales, al tiempo que
disminuimos nuestro consumo de elementos materiales. Pensemos en la diferencia
entre comprar un disco (almacn de material de msica) o escuchar msica a la carta
a travs de Internet o de un canal especializado. O bien en la diferencia existente entre
comprar un reloj (una mquina de medir el tiempo) para saber la hora, o tener acceso
a la hora a travs de la red. Si este proceso de desmaterializacin de nuestra vida
tiene lugar, va a suponer la posibilidad de cambios muy positivos en la organizacin
social, porque una economa basada en gran medida en el conocimiento tiene unas re-
glas de funcionamiento distintas de la economa industrial. []
A partir del momento en el que una parte importante de nuestro bienestar (nunca
todo, evidentemente) comienza a depender del acceso a bienes inmateriales, el pro-
blema de la escasez y el del reparto adquieren otra dimensin. Deja de ser un pro-
blema la escasez, porque el conocimiento es reproducible, y cambia el problema del
reparto porque no hay que renunciar a algo para que otro pueda tenerlo. Esto abre
unas perspectivas sociales muy esperanzadoras.
Joan MAJ
Clo, abril de 2002, n. 6
Qu cambios, segn Joan Maj, se han
producido en la informacin debido
a las nuevas tecnologas?
En qu se funda la denominacin
de la sociedad del conocimiento?
Qu cambios econmicos y sociales produce
el acceso a la informacin mediante las
nuevas tecnologas?
Realiza el resumen del texto.
Determina la estructura del fragmento
y reconoce las caractersticas lingsticas
propias de este tipo de textos.
Identifica los argumentos sobre los que
se sostiene la conclusin del texto.
Escribe un texto reflexivo donde se presenten
argumentos referidos a la dificultad o el
impedimento de lo expresado en la conclusin
(difusin de la red, posibilidades de acceso y
regulacin del mismo, instalacin de lneas).
7
6
5
4
3
2
1
166 10. Las modalidades textuales
M
A
T
E
R
I
A
L

F
O
T
O
C
O
P
I
A
B
L
E

/


O
x
f
o
r
d

U
n
i
v
e
r
s
i
t
y

P
r
e
s
s

E
s
p
a

a
,
S
.
A
.
ACTI VI DADES
10
0B1LLLP(2008).10+s 3/6/08 18:44 Pgina 166
REFUERZO
5. Argumentaciones
La argumentacin constituye una modalidad imprescindible en un ensayo, ya que es el tipo de
texto en que se exponen y defienden unas ideas determinadas sobre un tema. A continuacin
se citan fragmentos en los que se utilizan distintos tipos de argumentos.
En el texto 1 el autor diferencia dos tipos
de miedo, cules son y cmo justifica esa
distincin?
En el texto 2 se afirma que el aislamiento
acrecienta el miedo. Menciona los argumentos
(y sus tipos) con que desarrolla esta idea.
A qu tema se refiere el segundo prrafo del
texto 3?Qu relacin tiene con lo explicado
anteriormente?
Con qu afirmacin se inicia el texto 3?
Qu comparacin se realiza? Seala qu
tipos de interrogaciones son las que se
incluyen a continuacin. Qu contraste
se establece posteriormente?
Qu tipo de recursos expresivos cierran
el texto 4? Cul es su sentido y cmo se
relacionan con las ideas anteriores?
5
4
3
2
1
167 10. Las modalidades textuales
M
A
T
E
R
I
A
L

F
O
T
O
C
O
P
I
A
B
L
E

/


O
x
f
o
r
d

U
n
i
v
e
r
s
i
t
y

P
r
e
s
s

E
s
p
a

a
,
S
.
A
.
ACTI VI DADES
10
Texto 1
Resulta sorprendente que los especialistas en miedos
hayan estudiado tan poco las tcticas de amedren -
tamiento, los mecanismos de utilizacin del temor,
porque son la contrafigura del miedo. Hay terrores no
intencionados y terrores provocados voluntariamente,
estratgicos. La erupcin de un volcn, el feroz encrespa-
miento de un tsunami o una epidemia de peste no preten-
den suscitar miedo. Lo causan, sin ms. Pero la arbitrarie-
dad de un Estado policiaco, los atentados terroristas, los
chantajes de cualquier tipo, las crueldades domsticas, los
ataques preventivos, las operaciones de escarmiento se
sirven del miedo para alcanzar unos objetivos.
Texto 2
En el mundo animal, los cazadores intentan aislar a
su presa o buscar presas solitarias. El grupo es la gran
defensa. En los seres humanos sucede lo mismo, por ello
es fcil comprender que todas las dictaduras anulen el
ejercicio del derecho de reunin. En su teora relacional
del valor, Judith Jordan sostiene que la valenta no es un
rasgo interno, sino que surge en la conexin con los
dems: As human beings we encoura one another.
Nos animamos unos a otros. []
Los estudios sobre la violencia de gnero demuestran
que una de las tcticas del violento, del que pretende la
sumisin de la otra persona, es aislarla, obligarla a rom-
per los vnculos con su familia, anular su posibilidad de
comunicarse con los dems, obligarla a prescindir de
todos los sistemas de apoyo social que podran ayudarla.
El proceso suele comenzar con crticas a la familia de la
vctima, prohibicin de visitas, reclusin en casa, celos,
tcnicas de aislamiento, en una palabra.
Texto 3
El ser humano siente miedo y responde psico-
lgicamente al miedo con mecanismos muy pr-
ximos a los que usan los animales: huida, ataque,
inmovilidad y sumisin. Biolgicamente, el miedo
no plantea ningn problema. Qu otra cosa va a
hacer el ciervo sino huir del leopardo? Qu otra
cosa va a hacer el escarabajo sino hacerse el
muerto cuando lo toco? Son respuestas adapta -
tivas eficaces para todos los animales. Pero el
hombre no se encuentra cmodo en esas rutinas
tan contrastadas. El ser humano quiere vivir por
encima del miedo. Sabe que no puede eliminarlo,
sin caer en la locura o en la insensibilidad, como
ya deca Aristteles, pero quiere actuar a pesar
de l. Aqu se revela nuestra naturaleza parad -
jica: no podemos vivir sin que nuestros sentimien-
tos nos orienten, pero no queremos vivir a merced
de nuestros sentimientos.
Texto 4
Nuestra bsqueda de la felicidad es con fre-
cuencia desgarradora, porque estamos movidos
por dos deseos contradictorios: el bienestar y la
superacin. Necesitamos estar cmodos y nece-
sitamos crear algo de lo que nos sintamos orgu-
llosos y por lo que nos sintamos reconocidos.
Una actividad que d sentido a nuestra existen-
cia, por muy ilusorio que sea ese sentido. Tene-
mos, pues, que armonizar anhelos contradicto-
rios. Necesitamos construir la casa y descansar
en ella. Necesitamos estar refugiados en el puer-
to y navegando.
Jos Antonio MARINA
Anatoma del miedo, Crculo de Lectores
0B1LLLP(2008).10+s 3/6/08 18:44 Pgina 167
ORTOGRAFA
6. Uso de c y z
Palabras con cc y c
La letra c representa dos fonemas distintos: el oclusivo velar sordo ante -a, -o y -u (caro, coral,
cuyo), ante consonante y en posicin final de slaba o de palabra (acto, vivac); y el fricativo
interdental sordo ante -e, -i (cereza, ciento).
En las palabras con cc, la primera representa el sonido oclusivo velar y la segunda el fricativo
interdental y adems pertenecen a slabas distintas: correc-cin.
En muchos casos, si en una familia de palabras existe un morfema lxico con ct, las palabras
derivadas terminadas en -cin se escriben con cc: atractivo, atraccin; ficticio, ficcin; redactar,
redaccin.
Palabras con z
La letra z representa el fonema fricativo interdental sordo ante -a, -o, -u (zapato, zorro, zumo)
y en posicin final de slaba o de palabra (pez). En este ltimo caso, el plural se escribe con c
(peces).
La regla general presenta bastantes excepciones: zjel, zigzag, zigzagueo, enzima (la), nazi,
nazismo, zepeln, zipizape, zen
Se escriben con z las formas irregulares de la 1. persona del singular del presente de indica-
tivo y todo el presente de subjuntivo de los verbos terminados en -acer, -ecer, -ocer y -ucir
(merezco, merezcas).
Algunas palabras pueden escribirse con c o con z, aunque la RAE prefiere la variante escrita con
c: cebra / zebra, cenit / zenit, eccema / eczema, cigoto / zigoto, zinc / cinc, cedilla / zedilla.
En las zonas donde existe el seseo, c (ante e, i) y z representan el sonido de s.
Escribe el plural de las siguientes palabras:
nuez juez vez veloz
raz tenaz infeliz capaz
Escribe el singular de paredes, voluntades,
rapaces, veraces, disfraces.
Escribe las familias de palabras de objeto
y abstraccin.
Escribe c, cc o z, segn corresponda:
a) diale

to m) a

in
b) infe

in n) fi

in
c) cara

terstica ) t

ita
d) inspe

in o) precau

in
e) rela

iones p) fra

in
f) conjun

in q) interje

in
g) afli

in r) discre

in
h) a

eso s) obje

in
i) ju

gar t) produ

in
j) afe

in u) indu

in
k) co

in v) convi

in
l) defla

in w) sele

in
Crea oraciones con cada una de estas tres
parejas de palabras:
a) Adicin / adiccin.
b) Inflacin / infraccin
c) aficin - afeccin
Escribe los trminos derivados formados
con el morfema -cin que pertenezcan a
las familias de las palabras que figuran
a continuacin:
a) sujetar j) inhalar r) inducir
b) conducir k) deducir s) actuar
c) cocer l) afectar t) asociar
d) contraer m) situar u) reducir
e) corregir n) exhalar v) construir
f) sustraer ) redactar w) destruir
g) inventar o) satisfacer x) dirigir
h) distraer p) instruir y) inyectar
i) concretar q) proyectar z) relacionar
Escribe la 1. y la 3. persona del singular
y del plural del presente de indicativo y del
subjuntivo de los siguientes verbos: reconocer,
pertenecer, reducir, crecer, conducir, convencer.
Conjuga en la 1. persona singular del presente
de subjuntivo y del pretrito perfecto simple
de indicativo de utilizar, empezar, realizar
y generalizar.
Forma verbos parasintticos a partir de las
palabras flaco, grande, loco, noble, dulce.
A continuacin, conjgalos en la 1. persona
del singular del presente de indicativo y de
subjuntivo.
Escribe verbos acabados en -ocer y -ucir y
conjgalos en las mismas personas de la
actividad 9.
5
10
9
8
7
6
4
3
2
1
168 10. Las modalidades textuales
M
A
T
E
R
I
A
L

F
O
T
O
C
O
P
I
A
B
L
E

/


O
x
f
o
r
d

U
n
i
v
e
r
s
i
t
y

P
r
e
s
s

E
s
p
a

a
,
S
.
A
.
ACTI VI DADES
10
0B1LLLP(2008).10+s 3/6/08 18:44 Pgina 168
Evaluacin 10
159 10. Las modalidades textuales
1. Redacta un resumen del contenido del texto de Fernando
Savater.
2. Determina el tema (o los temas) que se desarrollan en el
fragmento.
3. A qu modalidad textual corresponde el texto? Justifica
tu respuesta.
4. Establece la estructura y, segn corresponda, los procedi-
mientos que se utilizan para desarrollar la informacin o
la tesis y los argumentos que sustentan las afirmaciones
del autor.
5. Reconoce en el texto las caractersticas propias de la
modalidad empleada y las funciones del lenguaje que
predominan.
6. Enuncia las finalidades de los textos expositivos y argu-
mentativos y cita mensajes escritos con tales modalidades.
7. En qu tipos de textos orales y escritos pueden apare-
cer la narracin y la descripcin? Con qu objetivo?
8. Realiza el anlisis morfolgico de las siguientes pala-
bras: imprecisas, absolutamente, acostumbran.
9. Realiza el anlisis sintctico de las siguientes oraciones.
a) Las opiniones se convierten en expresiones irreba -
tibles de la personalidad del sujeto.
b) A ello se une la obligacin beatfica del respeto a las
opiniones ajenas.
10. Escribe una reflexin crtica sobre el tema del fragmento.
Formula una tesis y desarrolla los argumentos.
En vez de ser consideradas propuestas imprecisas, limitadas
por la insuficiencia de conocimientos o el apresuramiento, las
opiniones se convierten en expresin irrebatible de la personali-
dad del sujeto: esta es mi opinin, eso ser su opinin, como
si lo relevante en ellas fuese a quin pertenecen en lugar de en
qu se fundan. La antigua y poco elegante frase que suelen decir
los tipos duros de algunas pelculas yanquis l as opiniones son
como los culos: cada cual tiene la suya cobra vigencia, porque
ni de las opiniones ni de los traseros cabe por lo visto discusin
alguna ni nadie puede desprenderse ni de unas ni de otro aunque
lo quisiera. A ello se une la obligacin beatfica de respetar las
opiniones ajenas, que si de verdad se pusiera en prctica parali-
zara cualquier desarrollo intelectual o social de la humanidad.
Por no hablar del derecho a tener su opinin propia, que no es
el de pensar por s mismo y someter a confrontacin razonada lo
pensado sino el de mantener la propia creencia sin que nadie
interfiera con molestas objeciones. Este subjetivismo irracional
cala muy pronto en nios y adolescentes, que se acostumbran a
suponer que todas las opiniones es decir, la del maestro que
sabe de lo que est hablando y la suya, que parte de la ignoran-
cia valen igual y que es seal de personalidad autnoma no
dar el brazo a torcer y ejemplo de tirana tratar de convencer al
otro de su error con argumentos e informacin adecuada.
La tendencia a convertir las opiniones en parte simblica de
nuestro organismo y a considerar cuanto las desmiente como una
agresin fsica (ha herido mis convicciones!) no solo es una difi-
cultad para la educacin humanstica sino tambin para la convi-
vencia democrtica. Vivir en una sociedad plural impone asumir
que lo absolutamente respetable son las personas, no sus opinio-
nes, y que el derecho a la propia opinin consiste en que esta sea
escuchada y discutida, no en que se la vea pasar sin tocarla como
si de una vaca sagrada se tratase.
Fernando SAVATER
El valor de educar, Ariel
xito de taquilla, por Dale Kennington.
El valor de las opiniones
0B1LLLA.10 7/4/08 18:30 Pgina 159
REFUERZO
1. El gnero lrico
A continuacin puedes leer un poema de Blas de Otero (1916-1979) y otro de Gloria Fuertes
(1918-1998). Ambos autores pertenecen a la poesa social, que se desarroll en Espaa a partir
de los aos cincuenta.
Justifica la adscripcin de estos dos textos
al gnero lrico.
Localiza en el poema de Blas de Otero
al emisor y al receptor y explica
el contenido del mensaje.
Explica la mtrica de ambos textos: la rima,
la medida de los versos, el agrupamiento
estrfico y el tipo de poema.
Qu recurso expresivo abunda en el poema
Djame? Localiza, adems, un apstrofe
y un caso de similicadencia.
Cul es el mensaje del poema de Gloria
Fuertes? Qu crees que significa: En el verso
hay ms sangre que tinta?
Qu recurso expresivo de repeticin
predomina en el segundo poema?
6
5
4
3
2
1
181 11. La literatura y los textos literarios
M
A
T
E
R
I
A
L

F
O
T
O
C
O
P
I
A
B
L
E

/


O
x
f
o
r
d

U
n
i
v
e
r
s
i
t
y

P
r
e
s
s

E
s
p
a

a
,
S
.
A
.
ACTI VI DADES
11
Djame
Me haces dao, Seor. Quita tu mano
de encima. Djame con mi vaco,
djame. Para abismo, con el mo
tengo bastante. Oh Dios, si eres humano,
compadcete ya, quita esa mano 5
de encima. No me sirve. Me da fro
y miedo. Si eres Dios, yo soy tan mo
como t. Y a soberbio, yo te gano.
Djame. Si pudiese yo matarte,
como haces t, como haces t! Nos coges 10
con las dos manos, nos ahogas. Matas
no se sabe por qu. Quiero cortarte
las manos. Esas manos que son trojes
del hambre, y de los hombres que arrebatas.
Blas DE OTERO
Poesa espaola contempornea (1939-1980), Alhambra
El planeta tierra
El planeta tierra
debera llamarse planeta agua.
En la tierra hay ms agua que cuerpo,
en el cuerpo hay ms cuerpo que alma,
en la tierra hay ms peces que aves, 5
en las aves ms plumas que alas.
En el verso hay ms sangre que tinta,
en la tinta ms sombra que nada,
en la nada hay ms algo que alga
y ese algo se mueve y reluce 10
y nace la palabra.
Gloria FUERTES
Poesa espaola contempornea (1939-1980), Alhambra
0B1LLLP(2008).11+s 3/6/08 15:59 Pgina 181
REFUERZO
2. El gnero narrativo
El siguiente relato pertenece al escritor argentino Jorge Luis Borges (1899-1986), quien ha
destacado tanto por sus cuentos como por sus poemas. Entre sus obras narrativas se encuen-
tran El jardn de los senderos que se bifurcan, El Sur, El libro de arena y El Aleph, al que pertenece
este cuento.
Los dos reyes y los dos laberintos
Cuentan los hombres dignos de fe (pero Al sabe ms) que en los primeros das
hubo un rey de las islas de Babilonia que congreg a sus arquitectos y magos y les
mand construir un laberinto tan perplejo y sutil que los varones ms prudentes no
se aventuraban a entrar, y los que entraban se perdan. Esa obra era un escndalo,
porque la confusin y la maravilla son operaciones propias de Dios y no de los
hombres. Con el andar del tiempo vino a su corte un rey de los rabes, y el rey de
Babilonia (para hacer burla de la simplicidad de su husped) lo hizo entrar en el
laberinto, donde vag afrentado y confundido hasta la declinacin de la tarde.
Entonces implor socorro divino y dio con la puerta. Sus labios no profirieron queja
ninguna, pero le dijo al rey de Babilonia que l en Arabia tena otro laberinto y que,
si Dios era servido, se lo dara a conocer algn da. Luego regres a Arabia, junt a
sus capitanes y sus alcaides y estrag los reinos de Babilonia con tan venturosa
fortuna que derrib sus castillos, rompi sus gentes e hizo cautivo al mismo rey.
Lo amarr encima de un caballo veloz y lo llev al desierto. Cabalgaron tres das,
y le dijo: Oh, rey del tiempo y sustancia y cifra del siglo!, en Babilonia me quisiste
perder en un laberinto de bronce con muchas escaleras, puertas y muros; ahora el
Poderoso ha tenido a bien que te muestre el mo, donde no hay escaleras que subir,
ni puertas que forzar, ni fatigosas galeras que recorrer, ni muros que te veden el
paso.
Luego le desat las ligaduras y lo abandon en mitad del desierto, donde muri de
hambre y de sed. La gloria sea con Aquel que no muere.
Jorge Luis BORGES
Obras completas, Emec
182 11. La literatura y los textos literarios
M
A
T
E
R
I
A
L

F
O
T
O
C
O
P
I
A
B
L
E

/


O
x
f
o
r
d

U
n
i
v
e
r
s
i
t
y

P
r
e
s
s

E
s
p
a

a
,
S
.
A
.
Justifica por qu podemos afirmar que
estamos en presencia de un texto narrativo.
A qu subgnero narrativo pertenece
este texto de Jorge Luis Borges?
Argumenta tu respuesta.
Resume el contenido de la historia narrada
por Borges.
Explica los conceptos de tiempo
de la historia y tiempo del discurso
a partir de este texto.
Qu movimientos narrativos reconoces
en el texto?
Caracteriza al narrador y su perspectiva.
Seala en el cuento ejemplos de discurso
referencial, valorativo, universal, directo
e indirecto.
En el discurso directo presente en el fragmento
tambin puede observarse discurso potico;
explica por medio de qu recurso expresivo
se materializa.
8
7
6
5
4
3
2
1
ACTI VI DADES
11
0B1LLLP(2008).11+s 3/6/08 16:00 Pgina 182
REFUERZO
3. El gnero dramtico
El siguiente fragmento pertenece a uno de los principales dramaturgos espaoles del siglo XX,
Antonio Buero Vallejo. Entre sus principales obras se encuentran Historia de una escalera
(1949), que abri un nuevo panorama en el drama de posguerra, En la ardiente oscuridad
(1950), Casi un cuento de hadas (1953), Irene o el tesoro (1954), El concierto de San Ovidio (1962),
El tragaluz (1967) y La fundacin (1974).
La confusin
Explica lo que ocurre en este fragmento
de la obra En la ardiente oscuridad,
de Antonio Buero Vallejo.
Justifica la adscripcin de esta obra al gnero
dramtico.
En qu subgnero lo incluiras? Argumenta
tu respuesta.
Diferencia en el fragmento el texto principal
y el texto secundario.
A qu hacen referencia las acotaciones
que aparecen?
Transforma el fragmento La confusin
en un texto narrativo. Recuerda que puedes
incluir dilogo en l.
6
5
4
3
2
1
183 11. La literatura y los textos literarios
M
A
T
E
R
I
A
L

F
O
T
O
C
O
P
I
A
B
L
E

/


O
x
f
o
r
d

U
n
i
v
e
r
s
i
t
y

P
r
e
s
s

E
s
p
a

a
,
S
.
A
.
ACTI VI DADES
11
ALBERTO.[] Se oye un bastn.
JUANA.Tambin yo lo oigo.
( Todos atienden. Pausa. Por la derecha, tan-
teando el suelo con su bastn y con una expresin
de vago susto, aparece IGNACIO. Es un muchacho
delgaducho, serio y reconcentrado, con cierto
desalio en su persona: el cuello de la camisa
desabrochado, la corbata floja, el cabello peinado
con ligereza. Viste de negro intemporalmente,
durante toda la obra. Avanza unos pasos indeciso
y se detiene.)
LOLITA.Qu raro!
(IGNACIO se estremece y retrocede un paso.)
MIGUEL.Quin eres?
(Temeroso, IGNACIO se vuelve para salir por don-
de entr. Despus, cambia de idea y sigue hacia
la izquierda, rpido.)
ANDRS.No contestas?
(IGNACIO tropieza con el silln de JUANA. Tiende el
brazo y ella coge su mano.)
MIGUEL.(Levantndose.) Espera, hombre! No te
marches.
(Se acerca a palparle, mientras JUANA dice, inquie-
ta:)
JUANA.Me ha cogido la mano No le conozco.
(IGNACIO la suelta y MIGUELN le sujeta por un brazo.)
MIGUEL.Ni yo.
(ANDRS se levanta y se acerca tambin para co-
gerle por el otro brazo.)
IGNACIO. (Con temor.) Djame.
ANDRS.Qu buscas aqu?
IGNACIO.Nada. Dejadme. Yo soy un pobre ciego.
LOLITA.(Riendo.) Te ha salido un competidor,
Migueln.
ESPERANZA.Un competidor? Un maestro!
ALBERTO.Debe de ser algn gracioso del primer
curso.
MIGUEL.Dejdmelo a m. Qu has dicho que eres?
IGNACIO.(Asustado.) Un ciego.
MIGUEL.Oh, pobrecito, pobrecito! Quiere que le
pase a la otra acera? (Los dems se desterni-
llan.) Largo, idiota! Vete a rer de los de tu curso.
ANDRS.Realmente la broma es de muy mal gusto.
Anda, mrchate.
(Lo empujan. IGNACIO retrocede hacia el prosce-
nio.)
IGNACIO.(Violento, quiz al borde del llanto.) Os
digo que soy ciego!
MIGUEL.Qu bien te has aprendido la palabrita!
Largo!
(Avanzan hacia l, amenazadores. ALBERTO se le-
vanta tambin.)
IGNACIO.Pero, es que no lo veis?
MIGUEL.Cmo?
(JUANA y CARLOS, que comentaban en voz baja el
incidente, intervienen.)
CARLOS.Creo que estamos cometiendo un error
muy grande, amigos. l dice la verdad. Sentaos
otra vez.
MIGUEL.Atiza!
CARLOS.(Acercndose con JUANA a IGNACIO.) Nosotros
tambin somos ciegos, como t dices.
Antonio BUERO VALLEJO
En la ardiente oscuridad, Magisterio Espaol
0B1LLLP(2008).11+s 3/6/08 16:00 Pgina 183
Evaluacin 11
177 11. La literatura y los textos literarios
1. Justifica por qu la Rima LII es un texto literario y por
qu pertenece al gnero lrico.
2. Identifica al emisor y a los receptores del proceso comu-
nicativo interno del poema y comenta sus caractersticas.
3. Qu actitud ofrece la voz potica en esta composicin?
Explica por qu.
4. Explica el contenido del mensaje en esta rima.
5. Qu tema desarrolla? Indica qu motivos aparecen.
6. Analiza la mtrica. Atiende a la medida de los versos, al
esquema y el tipo de rima y al agrupamiento en estrofas.
7. Consulta el anexo de las pginas 352-353 de este libro
y seala los recursos de repeticin que se emplean en el
texto de Bcquer.
8. Reconoce en esa misma composicin otros recursos
expresivos.
9. Qu rasgos debera presentar esta composicin de
Gustavo Adolfo Bcquer para pertenecer a cada uno de
los otros dos gneros literarios?
10. Redacta el comentario de la Rima LII del Libro de los
gorriones integrando todos los aspectos que has anali-
zado en las respuestas anteriores.
Olas gigantes que os rompis bramando
en las playas desiertas y remotas,
envuelto entre la sbana de espumas,
llevadme con vosotras!
Rfagas de huracn que arrebatis 5
del alto bosque las marchitas hojas,
arrastrado en el ciego torbellino,
llevadme con vosotras!
Nubes de tempestad que rompe el rayo
Los espritus de la tormenta, por Evelyn de Morgan.
y en fuego encienden las sangrientas orlas, 10
arrebatado entre la niebla oscura,
llevadme con vosotras!
Llevadme por piedad a donde el vrtigo
con la razn me arranque la memoria.
Por piedad! tengo miedo de quedarme 15
con mi dolor a solas!
Gustavo Adolfo BCQUER
Libro de los gorriones, Planeta
Rima LII
0B1LLLA.11 1/4/08 09:46 Pgina 177
AMPLIACIN
1. Lrica culta gallego-portuguesa.
Las Cantigas de Santa Mara
Aunque se admite que el rey Alfonso X el Sabio es el autor de algunas cantigas, se supone
que la mayora fueron creadas por un reducido grupo de escritores cortesanos. De las 420
Cantigas de Santa Mara, solo 64 son de carcter lrico; la mayor parte son narrativas y relatan
milagros de la Virgen. Todas van acompaadas de melodas y de preciosas miniaturas que
permiten conocer aspectos de la vida cotidiana de la poca.
Adems de la Introduccin y los dos prlogos, el resto de las cantigas lricas son de loor
o alabanza a la Virgen. Entre ellas predominan las de tipo hmnico, que celebran a Mara como
auxiliadora o se refieren a festividades marianas y cristolgicas.
En las Cantigas se registran cerca de 300 combinaciones mtricas distintas, de las cuales las
ms irregulares corresponden a las de carcter lrico. Los versos varan entre las dos y las
vein ticuatro slabas, y entre ellos destaca el alejandrino. En cuanto a las estrofas empleadas,
predomina el zjel.
Por qu pertenece a la lrica la Cantiga 10?
Para contestar, consulta la UNIDAD 11 y justifica
tu respuesta.
Explica cmo se presenta la figura de la Virgen
en este poema mariano.
Analiza la mtrica y los recursos expresivos
empleados en la cantiga.
Qu composiciones de la lrica culta pueden
haber influido en las Cantigas? Para responder,
apyate en ejemplos extrados de la Cantiga 10.
4
3
2
1
193 12. La lrica medieval
M
A
T
E
R
I
A
L

F
O
T
O
C
O
P
I
A
B
L
E

/


O
x
f
o
r
d

U
n
i
v
e
r
s
i
t
y

P
r
e
s
s

E
s
p
a

a
,
S
.
A
.
ACTI VI DADES
12
Cantiga 10
ESTA DE LOOR DE SANTA MARA, COM FREMOSA E BOA E GRAN PODER
Rosas das rosas e Fror das frores,
Dona das donas, Sennor das sennores.
Rosa de beldad e de parecer
e Fror d alegria e de prazer,
Dona en mui piadosa seer, 5
Sennor en toller coitas e doores.
Rosa das rosas e Fror das frores
A tal Sennor dev ome muit amar,
que de todo mal o pode guardar;
e pode-ll os peccados perd oar, 10
que faz no mundo per maos sabores.
Rosa das rosas e Fror das frores
Devemo-la muit amar e servir,
ca punna de nos guardar de falir;
des i dos erros nos faz repentir, 15
que nos fazemos come pecadores.
Rosa das rosas e Fror das frores
Esta dona que tenno por Sennor
e de que quero seer trobador,
se eu per ren poss aver seu amor, 20
dou ao demo os outros amores.
Rosa das rosas e Fror das frores
Alfonso X EL SABIO
Cantigas de Santa Mara, Castalia
Rosa de las rosas y Flor de las flores,
Seora de las seoras, Seor de los seores.
Rosa de beldad y jucio
y Flor de alegra y placer,
Seora muy piadosa en su ser, 5
Seor en quitar penas y dolores.
Rosa de las rosas
A tal Seor debe el hombre mucho amar,
que de todo mal lo puede guardar,
y puede perdonarle los pecados, 10
que acta en el mundo contra los sinsabores.
Rosa de las rosas
La debemos amar mucho y servir,
porque lucha por guardarnos de caer;
de los errores nos hace arrepentir, 15
que nosotros hacemos como pecadores.
Rosa de las rosas
Esta seora que tengo por Seor
y de quien quiero ser trovador,
si yo por fortuna puedo tener su amor, 20
doy al demonio otros amores.
Rosa de las rosas
Traduccin de los autores
0B1LLLP(2008).12+s 3/6/08 18:45 Pgina 193
REFUERZO
2. La lrica primitiva popular
En la Pennsula, la lrica primitiva popular se manifiesta en tres tipos de composi ciones: las
jarchas, las cantigas de amigo y los villancicos. Repasa las caractersticas formales y tem ticas
de estas producciones literarias y despus lee detenidamente los siguientes textos.
194 12. La lrica medieval
M
A
T
E
R
I
A
L

F
O
T
O
C
O
P
I
A
B
L
E

/


O
x
f
o
r
d

U
n
i
v
e
r
s
i
t
y

P
r
e
s
s

E
s
p
a

a
,
S
.
A
.
Identifica entre estos cinco poemas cules
son jarchas. En qu aspecto o aspectos
te has basado para reconocerlas?
Caracteriza el emisor, receptor y contenido
del mensaje en cada una de ellas.
Analiza el nmero de versos y la rima
empleada en estas composiciones.
En una de las jarchas figura una metfora;
localzala y explica su valor.
Entre los poemas se incluyen tambin
villancicos castellanos. Selalos y explica
en qu te has basado para reconocerlos.
Establece el proceso de la comunicacin
en esos poemas: emisor, receptor
y contenido del mensaje.
Qu tipo de poema configura la cantiga
de amigo? Para responder, consulta, si es
necesario, el apndice (pginas 358 y 359)
y la pgina 184 de tu libro de texto.
Establece las semejanzas de contenido
entre el poema del texto 5, las jarchas
y los villancicos; seala, igualmente,
las semejanzas y diferencias formales
entre estos tres tipos de composiciones
de la lrica primitiva popular.
4
8
7
6
5
3
2
1
ACTI VI DADES
12
Texto 1
Non dormireyo, mamma,
a rayo de maana.
Bon Abu-l-Qasim,
la facha de matrana.
No dormir, madre,
al rayar la maana.
El buen Abu-l-Qasim,
la cara de la aurora.
Texto 2
Quiero dormir y no puedo,
que el amor me quita el sueo.
Texto 3
Qu far, mamma?
Meu al-habib est ad yana.
Qu har, madre?
Mi amigo est en la puerta.
Texto 4
Con amores, mi madre,
con amores me adorm.
Lrica espaola de tipo popular
Ctedra
Texto 5
Mia irmana fremosa, treydes comigo
a la igreja de Vig, u o mar salido:
E miraremos las ondas!
Mia irmana fremosa, treydes de grado
a la igresa de Vig, u o mar levado: 5
E miraremos las ondas!
A la igresa de Vig, u o mar salido,
e verr i, mia madre, o meu amigo:
E miraremos las ondas!
A la igresa de Vig, u o mar levado, 10
e verr y, mia madre, o meu amado:
E miraremos las ondas!
(Invita a su hermana a visitar la iglesia de Vigo durante
la marea alta, y dice a su madre que all ir su amigo.)
Antologa de la poesa gallego-portuguesa
Alhambra
0B1LLLP(2008).12+s 3/6/08 18:45 Pgina 194
REFUERZO
3. La lrica del siglo XV
El siglo XV es una poca de eclosin de la poesa. El Romancero constituye un hito en nuestra
tradicin literaria y crea una tipologa que permanece hasta la actualidad. La poesa culta de los
cancioneros ser la fuente de la que bebern los poetas renacentistas.
Reconoce en el texto 1 la mtrica empleada
por su autor, Jorge Manrique, en las Coplas
por la muerte de su padre y seala en la estrofa
otras caractersticas de esta obra.
Quin es el receptor del texto 2?
Qu le dice la voz potica? Explica
qu intencin manifiesta en los tres ltimos
versos.
Reconoce la mtrica empleada por Juan
de Mena en la obra a la que pertenece
la estrofa del texto 2 y seala en ella
otras caractersticas de esta obra.
A qu tipo de composiciones corresponden
los otros tres textos? Explica su mtrica
y su estructura.
Texto 1
Texto 2
Texto 3
Explica brevemente el contenido de cada
uno de los poemas reproducidos en esta
pgina.
Seala en los textos 3, 4 y 5 los recursos
expresivos caractersticos de este tipo
de composiciones.
6
5
4
3
2
1
195 12. La lrica medieval
M
A
T
E
R
I
A
L

F
O
T
O
C
O
P
I
A
B
L
E

/


O
x
f
o
r
d

U
n
i
v
e
r
s
i
t
y

P
r
e
s
s

E
s
p
a

a
,
S
.
A
.
ACTI VI DADES
12
Y pues vemos lo presente
cmo en un punto se es ido
y acabado,
si juzgamos sabiamente,
daremos lo non venido 5
por pasado.
No se engae nadie, no,
pensando que a de durar
lo que espera
ms que dur lo que vio, 10
porque todo ha de pasar
por tal manera.
Jorge MANRIQUE
Poesa, Crtica
Texto 3
Ven, muerte, tan escondida
que no te sienta conmigo,
porque el gozo de contigo
no me torne a dar la vida.
Ven como rayo que hiere: 5
que hasta que ha herido
no se siente su ruido,
por mejor herir do quiere.
As sea tu venida;
si no desde aqu me obligo 10
que el gozo que habr contigo
me dar de nuevo vida.
Poesa medieval castellana
Taurus
Texto 4
Vos me matis de tal suerte,
con pena tan glorosa,
que no s ms dulce cosa
que los trances de mi muerte.
Y de ella soy tan ufano, 5
tan penado y tan contento,
que no trocar un tormento
por mil bienes de otra mano.
Y pues que quiso mi suerte
darme pena gloriosa, 10
no quiero ms dulce cosa
que los trances de mi muerte.
Poesa medieval castellana
Taurus
Texto 5
Soy contento, vos servida,
ser penado de tal suerte,
que por vos quiero la muerte
mas que no, sin vos, la vida.
Quiero ms por vos tristura, 5
siendo vuestro sin mudanza,
que placer sin esperanza
de enamorada ventura.
No tengis la fe perdida
pues la tengo yo tan fuerte, 10
que por vos quiero la muerte
mas que no, sin vos, la vida.
Poesa medieval castellana
Taurus
Texto 2
Tus casos falaces, Fortuna, cantamos,
estados de gentes que giras y trocas,
tus grandes discordias, tus firmezas pocas,
y los que en tu rueda quejosos hallamos,
hasta que al tiempo de ahora vengamos; 5
de hechos pasados codicia mi pluma,
y de los presentes, hacer breve suma;
d fin Apolo, pues nos comenzamos.
Juan DE MENA
El laberinto de la Fortuna, Espasa Calpe
Texto 1
0B1LLLP(2008).12+s 3/6/08 18:45 Pgina 195
LITERATURA UNIVERSAL
4. El arte de los trovadores
El arte de los trovadores provenzales fue producto de un largo proceso de aprendizaje
de potica, retrica y msica. Ya desde la primera mitad del siglo XII se diferencian claramente
dos grupos de poetas segn su modo de expresin: los partidarios de una poesa sencilla el
trobar leuy los que cultivan un arte ms complejo el trobar clus y el trobar ric.
El trobar leu
El trobar leu o arte de la versi -
ficacin sencilla fue practicado,
entre otros, por el poeta Bernart
de Ventadorn, que ha sido consi-
derado el primero de los trovado-
res amorosos de la literatura
pro venzal. A l pertenece este
fragmento prosificado.
El trobar clus y el trobar ric
Entre los poetas partidarios del arte de la poesa difcil, algunos practicaron el
denominado trobar clus o cerrado, que utilizaba un lenguaje sumamente
enigmtico, recargado de conceptos, aunque a veces incorporaba tambin un
registro popular y bajo.
Otro grupo de trovadores, por el contrario, se volcaron hacia el trobar ric,
un arte ms complejo en el que se atenda esencialmente a la belleza formal,
a la sonoridad, a la sugerencia del sonido y el ritmo, y a la seleccin de un voca-
bulario inusual.
El trobar clus surgi entre poetas realistas y de condicin humilde, que recha -
zaban el amor corts, como Marcabr. El trobar ric, en cambio, posea un carcter
aristocrtico, aceptaba los postulados de ese cdigo amoroso y no era en abso-
luto moralizante. Estos rasgos se observan, por ejemplo, en la obra de Arnaut
Daniel.
Qu sentimiento embarga al yo potico del
texto 1? Qu lo causa y qu efecto produce?
A quines ataca especialmente el poeta
Marcabr en el fragmento seleccionado?
Qu anhela el emisor de la composicin de
Arnaut Daniel?
Explica qu provoca en el emisor de texto 3
el estado en el que se encuentra.
4
3
2
1
196 12. La lrica medieval
M
A
T
E
R
I
A
L

F
O
T
O
C
O
P
I
A
B
L
E

/


O
x
f
o
r
d

U
n
i
v
e
r
s
i
t
y

P
r
e
s
s

E
s
p
a

a
,
S
.
A
.
ACTI VI DADES
12
Texto 1
Tengo mi corazn tan lleno de alegra que todo me lo transfigura: el
fro me parece flor blanca, roja y amarilla, pues con el viento y con la
lluvia me crece la ventura; por lo que mi mrito aumenta y sube y mi
canto mejora. Tanto amor tengo en el cora zn, tanta alegra y dulzura,
que el hielo me parece flor y la nieve verdor.
Bernart de VENTADORN
en Historia de la literatura universal, Planeta
Texto 2
Homicidas y traidores, simo -
nacos
1
, saludadores, lujuriosos
y usureros que viven de ruines
menesteres, y los que hacen
hechiceras y las repugnantes
hechiceras estarn por igual
en el fuego ardiente. Borrachos
y cornudos, falsos curas y fal-
sos abades, falsas reclusas y fal -
sos reclusos, all penarn, dice
Marcabr.
MARCABR
Texto 3
El aura amarga hace aclarar los
bosque cillos ramosos, que la
dulce espes con hojas, y man-
tiene balbucientes y mudos los
alegres picos de los pjaros de
las ramas, aparejados y no apa-
rejados. Por qu yo me esfuer-
zo en hacer y decir cosas agra-
dables a muchos? Por aquella
que me ha vuelto de arriba
abajo, de lo que temo morir si
no me da fin a los afanes.
Arnaut DANIEL
en Historia de la literatura universal
Planeta
1
simonaco: el que compra o vende
bienes espirituales.
0B1LLLP(2008).12+s 3/6/08 18:46 Pgina 196
Evaluacin 12
193 12. La lrica medieval
1. El texto 1 corresponde a una cantiga de la poesa can-
cioneril del siglo &V. Reconoce en ella las caractersticas
formales y temticas de este tipo de composiciones.
2. El texto 2 es la traduccin prosificada de una estrofa de
una moaxaja rabe. Seala los rasgos propios de estos
poemas y explica el contenido de la estrofa reproducida.
3. Qu otra especie potica culta fue cultivada en al-
ndalus? Explica en qu se diferencia de la moaxaja.
4. Qu poesa se cultiv en el este de la Pennsula en los
siglos &II y &III? Explica su vertiente amorosa.
5. En qu manifestacin culta de la poesa del oeste
peninsular influye la lrica aludida en la actividad 4?
Comenta sus rasgos principales.
6. Los textos 3 y 4 brindan muestras de dos tipos de poesa
primitiva popular. Identifcalas y explica sus rasgos.
7. Qu tipo de poesa popular no aparece representada?
Seala semejanzas y diferencias con los anteriores.
8. El texto 5 es una estrofa de una obra del marqus de
Santillana. Explica su contenido y analiza su mtrica.
De qu tipo de poesa cancioneril es muestra?
9. Cul es la gran creacin potica de Jorge Manrique?
Explica los siguientes aspectos de esa obra:
Contenido de cada parte. Temas.
Mtrica. Estilo.
10. Redacta una exposicin sobre la poesa medieval culta
y popular en gallego-portugus.
Texto 1
Plega Dios que alguno quieras
como yo, mi bien, te quiero,
porque penes, porque mueras,
porque sepas que de veras
por tu sola causa muero. 5
Amada ni ms ni menos
seas como yo de ti,
porque defectos ajenos
te hagan mencin de m;
y penes porque debieras 10
quererme como te quiero;
y por me cobrar te mueras
y no puedas aunque quieras
siendo yo muerto primero.
Pedro LVAREZ OSORIO
CONDE DE TRASTMARA y MARQUS DE ASTORGA
Poesa espaola 2. Edad Media: Lrica y cancioneros
Crtica
Texto 2
Ojal alcanzara la unin mi corazn, que est en continua
angustia! Todo corazn que arde en la pasin del amor est
inquieto por la agitacin que sufre y que deshace al que en
ansias se acongoja.
Ibn RUHAYM
en F. CORRIENTE, Poesa dialectal rabe y romance en al-ndalus
Gredos
Texto 3
Garid vos, ay yermaniellas!,
cmcontenir el mio male?
Sin el habib non vivreyo:
ad ob li rey demandare?
Lrica espaola de tipo popular
Ctedra
(TRADUCCIN: Decidme, ay hermanitas!, / cmo contener mi mal? / Sin el amado no
vivir: / adnde ir a buscarlo?)
Texto 4
A quin contar yo mis quejas,
mi lindo amor?
A quin contar yo mis quejas
si a vos no?
Lrica espaola de tipo popular, Ctedra
Texto 5
Qu se hizo la moneda
que guard para mis daos
tantos tiempos, tantos aos,
plata, joyas, oro o seda?
Ca de todo no me queda 5
sino este cadahalso.
Mundo malo, mundo falso,
no es quin contigo pueda.
Marqus DE SANTILLANA
Poesa cancioneril castellana, Akal
Ilustracin de la Divina comedia, de Dante Alighieri.
0B1LLLA.12 1/4/08 09:52 Pgina 193
REFUERZO
1. El Cantar de Mio Cid
La pica castellana tiene su mejor exponente en el Cantar de Mio Cid, que exalta la figura del
infanzn Rodrigo Daz de Vivar, el Cid Campeador, tanto en su dimensin hist rica como en
las circunstancias personales de la vida del hroe.
Qu episodio del Cantar narra este
fragmento? Indica a qu parte del poema
corresponde.
Seala las apelaciones al auditorio presentes
en estos versos. Qu funciones del lenguaje
predominan en ellos?
Reconoce el empleo de eptetos picos y seala
a quin se califica por medio de este recurso.
Mide los versos y analiza la rima de esta tirada
del Cantar de Mio Cid.
Diferencia la presencia del discurso
del narrador del de los personajes en
este fragmento.
Para relatar los sucesos, el narrador emplea
dos tiempos verbales; de cules se trata?
Qu efecto se produce con el cambio
de formas verbales?
Explica cmo se caracterizan los personajes
mencionados en este pasaje.
8
7
6
5
4
3
2
1
207 13. La narrativa medieval
M
A
T
E
R
I
A
L

F
O
T
O
C
O
P
I
A
B
L
E

/


O
x
f
o
r
d

U
n
i
v
e
r
s
i
t
y

P
r
e
s
s

E
s
p
a

a
,
S
.
A
.
ACTI VI DADES
13
Comienzan ya a preparar la gran sala de palacio.
Los suelos con mucha alfombra, todo bien encortinado.
Cunta seda y cunta prpura y cunto pao preciado!
Gusto os dara vivir y comer en el palacio!
Los caballeros del Cid aprisa all se juntaron, 5
y entonces en aquel punto por los Infantes mandaron.
Ya cabalgan los Infantes, camino van del palacio,
con muy ricas vestiduras, galanamente ataviados.
A pie y con muy buena cara, Dios qu discretos entraron!
Recibiolos nuestro Cid; con l todos sus vasallos. 10
Ante el Cid y su mujer los Infantes se inclinaron.
A sentar ellos se fueron en un muy precioso escao.
Los de la casa del Cid, siempre en todo mesurados,
estn atentos mirando al que naci afortunado.
All el Cid Campeador ved que en pie se ha levantado: 15
Puesto que hacerlo tenemos, por qu lo vamos tardando?
Venid ac, mi Alvar Fez, el que tanto quiero y amo.
Aqu tenis mis dos hijas; yo las pongo en vuestras manos.
Sabis que al Rey esto mismo se lo tengo as rogado.
No quiero faltar en nada de lo que fue concertado. 20
A los dos Infantes, vos ddselas con vuestras manos,
que tomen las bendiciones, y vayamos acabando.
Entonces dijo Minaya: Esto har yo de buen grado.
De pie las dos se levantan, y l las tom de las manos.
Y a los Infantes, Minaya esto mismo les va hablando: 25
Aqu estis ante el Minaya, vosotros, los dos hermanos.
De manos del Rey Alfonso, pues a m me lo ha mandado,
estas dueas yo os entrego, que ambas son hijasdalgo;
que las tomis por mujeres, dadles honras y cuidados.
Los de Carrin las reciben con amor y de buen grado. 30
Al Cid y a doa Jimena, van a besarles las manos.
As que esto hubieron hecho, se salieron del palacio.
Aprisa, a Santa Mara, hacia all van caminando.
Muy pronto se revisti don Jernimo, el prelado,
y a la puerta de la iglesia estbalos esperando. 35
Las bendiciones les dio; despus la misa ha cantado.
Al salirse de la iglesia cabalgaron a buen paso.
Afuera de la ciudad, en un arenal cercano,
Dios, y qu buen juego de armas hizo el Cid a sus vasallos!
Poema del Cid
Castalia
0B1LLLP(2008).13+s 3/6/08 18:40 Pgina 207
REFUERZO
2. Un milagro,
de Gonzalo de Berceo
208 13. La narrativa medieval
M
A
T
E
R
I
A
L

F
O
T
O
C
O
P
I
A
B
L
E

/


O
x
f
o
r
d

U
n
i
v
e
r
s
i
t
y

P
r
e
s
s

E
s
p
a

a
,
S
.
A
.
Caracteriza al protagonista de este milagro.
Qu le sucede?
Resume por escrito el contenido de El labrador
avaro.
Caracteriza al narrador de esta historia
de Gonzalo de Berceo.
Diferencia en el milagro que acabas de leer las
partes narrativas de las intervenciones
de los personajes.
Cmo se introducen estas ltimas?
Seala las partes en que se divide la narracin
de Berceo.
Reconoce los movimientos narrativos
y los tipos de discurso que aparecen
en el texto.
Cul es la finalidad de este milagro?
Relacinala con la del resto de la obra
a la que pertenece.
Se cumple en este milagro en concreto
lo que en tu libro de texto se plantea en
general para todas estas narraciones?
Justifica tu respuesta.
Escribe en prosa la historia narrada por Berceo
en El labrador avaro. No olvides cuidar la
ortografa y la presentacin de tu redaccin.
10
9
8
7
6
5
4
3
2
1
ACTI VI DADES
13
A continuacin, tienes uno de los Milagros de Nuestra Seora, de Gonzalo de Berceo; lelo
con atencin y observa su estructura, pues esta se repite en los restantes milagros que com-
ponen la coleccin.
El labrador avaro
rase en una tierra un hombre labrador
que usaba de la reja ms que de otra labor;
ms amaba la tierra que no a su Criador,
y de muchas maneras era revolvedor.
Haca una enemiga bien sucia de verdad: 5
cambiaba los mojones por ganar heredad;
haca en todas formas tuertos y falsedad,
tena mal testimonio entre su vecindad.
Aunque malo, quera bien a Santa Mara,
oa sus milagros, muy bien los acoga; 10
saludbala siempre, decale cada da:
Ave grata plena que pariste al Mesas.
Fin el arrastrapajas de tierra bien cargado,
de los diablos fue luego en soga cautivado;
lo arrastraban con cuerdas, de coces bien sobado, 15
le pechaban al doble el pan que dio mudado.
Dolironse los ngeles de esta alma mezquina
por cuanto la llevaban los diablos en rapina;
quisieron socorrerla, ganarla por vecina,
mas para hacer tal pasta mengubales harina. 20
Si les decan los ngeles de bien una razn,
ciento decan los otros malas, que buenas non;
los malos a los buenos tenan en un rincn,
la alma por sus pecados no sala de prisin.
Levantndose, un ngel dijo: Yo soy testigo, 25
verdad, es no mentira, esto que ahora os digo:
el cuerpo que traa esta alma consigo
fue de Santa Mara buen vasallo y amigo.
Siempre la mencionaba al yantar y a la cena,
decale tres palabras: Ave, grata plena. 30
Boca por que sala tan santa cantilena
no mereca yacer en tan mala cadena.
Luego que este nombre de la Santa Rena
oyeron los demonios, salieron tan ana,
derramronse todos como una neblina, 35
desampararon todos a esa alma mezquina.
Los ngeles la vieron quedar desamparada,
de manos y de pies con sogas bien atada,
estaba como oveja cuando yace enzarzada:
fueron y la llevaron junto con su majada. 40
Nombre tan adornado, lleno de virtud tanta,
y que a los enemigos los seguda y espanta,
no nos debe doler ni lengua ni garganta
que no digamos todos: Salve, Regina sancta.
Gonzalo DE BERCEO
Milagros de Nuestra Seora
Castalia
0B1LLLP(2008).13+s 3/6/08 18:40 Pgina 208
REFUERZO
3. Fragmentos del
Libro de buen amor
Los dos fragmentos siguientes corresponden al Libro de buen amor. El primero, de carcter narrativo,
pertenece a la serie de aventuras que el protagonista vive en la sierra, donde se enfrenta a mujeres de
caractersticas muy especiales; el segundo, lrico, se inserta en el conjunto de poemas intercalados en
la historia narrada. Estos dos textos, adems, presentan diferencias en cuanto a la mtrica empleada.
Atiende a la descripcin de la serrana Aldara
y explica cmo aparece caracterizada.
Seala las referencias al receptor. Con qu
finalidad aparecen?
Recuerda los rasgos de la pastorela y justifica
por qu esta serrana es un texto pardico.
Explica los recursos que emplea el narrador
para degradar la figura de la serrana.
Analiza la mtrica y la rima de los Gozos.
Seala la diferencia mtrica existente entre
estos versos de los Gozos y la utilizada,
en general, en el Libro de buen amor.
Con qu recursos se exalta en la segunda
composicin la figura de la Virgen Mara?
Justifica por qu los Gozos de Santa Mara
constituyen un texto de carcter lrico.
8
7
6
5
4
3
2
1
209 13. La narrativa medieval
M
A
T
E
R
I
A
L

F
O
T
O
C
O
P
I
A
B
L
E

/


O
x
f
o
r
d

U
n
i
v
e
r
s
i
t
y

P
r
e
s
s

E
s
p
a

a
,
S
.
A
.
ACTI VI DADES
13
La serrana Aldara
Sus miembros y su talle no son para callar,
me podis creer, era gran yegua caballar;
quien con ella luchase mal se habra de hallar,
si ella no quiere, nunca la podrn derribar. []
Tena la cabeza mucho grande y sin guisa 5
cabellos cortos, negros, como corneja lisa,
ojos hundidos, rojos; ve poco y mal divisa;
mayor es que de osa su huella, cuando pisa.
Las orejas, mayores que las de aal borrico,
el su pescuezo, negro, ancho, velludo, chico; 10
las narices, muy gordas, largas, de zarapico,
sorbera bien pronto un caudal de hombre rico!
Su boca era de alano, grandes labios muy gordos,
dientes anchos y largos, caballunos, moxmordos;
sus cejas eran anchas y ms negras que tordos. 15
Los que quieran casarse, procuren no estar sordos!
Mayores que las mas tiene sus negras barbas;
yo no vi ms en ella, pero si ms escarbas,
hallars, segn creo, lugar de bromas largas,
aunque ms te valdr trillar en las tus parvas. [] 20
Ms anchas que mi mano tiene la su mueca,
velluda, pelos grandes y que nunca est seca;
voz profunda y gangosa que al hombre da jaqueca,
tarda, enronquecida, muy destemplada y hueca.
Es su dedo meique mayor que mi pulgar, 25
son los dedos mayores que puedes encontrar,
que, si algn da, ella te quisiere espulgar,
daarn tu cabeza cual vigas de lagar.
Tena en el justillo las sus tetas colgadas,
dbanle en la cintura porque estaban dobladas, 30
que, de no estar sujetas, diranle en las ijadas;
de la ctara al son bailan, aun no enseadas.
Arcipreste DE HITA
Libro de buen amor
Castalia
Gozos de Santa Mara
Madre de Dios, gloriosa
Virgen Santa Mara
hija y leal esposa
de tu hijo Mesa,
t, Seora, 5
dame ahora
la tu gracia a toda hora;
que te sirva en toda va.
Porque honrarte codicio,
pecador yo!, por tanto 10
ofrezco en tu servicio
los gozos que te canto.
El primero
en que tercero
a ti el ngel mensajero 15
fue del Espritu Santo.
Concebiste a tu Padre;
fue tu gozo segundo,
cuando pariste, Madre,
sin dolor sali al mundo. 20
Cual naciste
(t pariste),
intacta permaneciste
Oh Virgen del santo mundo!
El tercero, la estrella, 25
luciente meteoro;
los Reyes, con luz de ella,
trajeron su tesoro
y alabaron
y adoraron 30
y a tu Hijo presentaron
el incienso, mirra y oro.
Arcipreste DE HITA
Libro de buen amor
Castalia
0B1LLLP(2008).13+s 3/6/08 18:40 Pgina 209
210 13. La narrativa medieval
M
A
T
E
R
I
A
L

F
O
T
O
C
O
P
I
A
B
L
E

/


O
x
f
o
r
d

U
n
i
v
e
r
s
i
t
y

P
r
e
s
s

E
s
p
a

a
,
S
.
A
.
ACTI VI DADES
13
AMPLIACIN
4. Novela de caballeras
y novela sentimental
Los siguientes fragmentos corresponden a dos de las principales orientaciones de la narrativa
del siglo XV: la novela de caballeras y la novela sentimental.
Describe cmo aparecen caracterizados
los personajes del primer texto y justifica
por qu son prototpicos del tipo de novela
al que pertenecen.
En el primer pasaje se plantean
dos situaciones; de cules se trata?
A qu caractersticas de la novela de
caballeras remite esta circunstancia?
Teniendo en cuenta lo que sabes acerca de la
novela a la que pertenece el primer fragmento,
explica la frase con la que concluye (perdera
la gloria que por ella he ganado).
A qu clase de texto corresponde el
fragmento de Diego de San Pedro? Explica
de qu modelo narrativo del siglo XV es propio.
Justifica por qu su contenido tambin
corresponde a ese tipo de novela.
Analiza en el fragmento de Diego de San Pedro
recursos estilsticos de este tipo de textos.
Qu funciones del lenguaje predominan
en cada uno de los fragmentos?
Compara los dos pasajes y seala las
diferencias ms relevantes.
8
7
6
5
4
3
2
1
Texto 1
Amads viva en una gran tristeza y soledad. Un da
cabalgaba por la orilla del mar, y al subir a unas peas
vio que se acercaba una nave. En la nave vena Durn,
hermano de la doncella de Dinamarca. Gandaln lo
abraz y lo condujo hasta donde estaba Amads.
Cuando estuvieron cerca, vieron una especie de gigante
envuelto en pieles que esgrima un venablo para arro-
jarlo contra Amads. Gandaln y Durn empezaron a dar
grandes voces, y el venablo pas por encima de la cabe-
za de Amads. Amads se levant de un salto y pudo sal -
varse de un segundo venablo. Ech mano a la espada,
pero aquella figura extraa sali huyendo, subi en el
caballo de Amads y se alej, diciendo a grandes voces:
Yo soy Andandona, la giganta de la Isla Triste, tu ene-
miga, y si ahora no acab lo que deseaba, otra vez ser.
Amads dej que Gandaln la persiguiese y abraz a
Durn y le pregunt por el objeto de su venida. Durn le
dio una carta de Oriana y le dijo que ella haba tenido
un hijo que criaba la abadesa del monasterio de Mira -
flores, y que le rogaba que no partiese de Gaula hasta
tener su mando.
Volvi Gandaln, con el caballo de Amads y la cabeza
de Andandona atada al pretal por los cabellos largos
y canos. Amads envi la cabeza a Bruneo de Bonamar
y dijo a Durn:
Vuelve y dile a tu seora que le beso las manos por
la carta que me enva y por lo que t me has dicho de
su parte, y que le ruego que tenga piedad de mi honra
y no me deje holgar mucho tiempo, porque lo atribuiran
a poquedad de corazn y perdera la gloria que por
ella he ganado.
Amads de Gaula
Castalia
Texto 2
Lucenda: es tan grande merced esta que hacerme quisiste,
que si yo de ms valer o ella de menos sus tancia no fuese,
servicio ninguno satisfacerla podra, si pena en cuenta
de servir recibida no fuese; la cual si recibir no quisieres,
si merecida te la tengo en m lo puedes bien ver, porque
en mi desfiguramiento a ella y a tus obras pintadas vers;
y no solamente mis l grimas a acrecentar mis dolores me
han bastado, mas viendo mi parecer, a muchos enamo -
rados de amar he hecho temerosos; y puesto que as es,
te suplico que, arrepentida de lo pasado, lo por venir
enmiendes.
No seas en el daar siempre una; pon con tus obras mis
guerras en paz; no s por qu pudiendo has dejado de
servida ser. Todas lo quieren y t lo rehyes; bien parece
que yo tengo mayor necesidad de tus mer cedes, que t
voluntad de mis servicios tenas. Oh qu combates de
mi mucha fe y de tu poca esperanza he recibido!, los
cuales, como ves, la fuerza de mi salud han enflaque -
cido. Cuanto t menos de mi dolor te dolas, ms mi dolor
me dola. Si pudiese en la boca poner lo que en el alma
he sentido, cunta culpa por mi pena te daras! Nunca
nadie menos bien tuvo; nunca nadie ms mal sufri;
nunca de tanta memoria tanto olvido se tuvo; mi afec-
cin y tu menosprecio des truyen mi salud.
Todo esto, Lucenda, te digo, porque ms y en ms mi
querer tengas, y porque en mi sufrimiento mi firmeza
conozcas; que ni por todo mi mal jams en mu danza
pens, ni de sufrir cansado me vi; antes ga nado, porque
t me perdas, siempre me hall, pero no por pequea
causa, que no tena yo menos razn para penar que t
hermosura para penarme.
Diego DE SAN PEDRO
Tratado de amores de Arnalte y Lucenda. Sermn
Castalia
0B1LLLP(2008).13+s 3/6/08 18:40 Pgina 210
Evaluacin 13
215 13. La narrativa medieval
1. Indica a qu parte de la historia de La Celestina corres-
ponde este pasaje y reconoce en l recursos expresivos.
Qu modalidad del discurso dramtico presenta
este fragmento?
Explica las caractersticas de las otras dos modalidades.
2. Qu relacin guarda la intervencin de Pleberio con el
tratamiento del amor en la obra? Comenta otros aspectos
del amor considerados en la unidad.
3. Expn cuanto sepas acerca del gnero y los personajes
principales de La Celestina.
4. En cierta forma, la obra de Fernando de Rojas parodia la
novela sentimental. Explica por qu y compara este tipo
de ficcin narrativa del siglo &V con la de caballeras,
atendiendo a los aspectos que se indican en la tabla.
5. Explica los dos asuntos que determinan la estructura del
Cantar de Mio Cid.
6. Compara el Cantar de Mio Cid con los poemas del mester
de clereca, considerando los aspectos indicados.
7. Menciona obras del mester de clereca que estn escritas
en cuaderna va.
8. Qu son los enxiemplos? Indica cmo se insertan en
El conde Lucanor.
9. Explica la estructura y el sentido del Libro de buen amor.
Qu personaje incorpora esta obra medieval a la tradi-
cin literaria?
10. Redacta una exposicin acerca de los romances que
contemple los aspectos sealados a continuacin.
Caractersticas de los romances viejos.
Forma de transmisin de dichas composiciones.
Temas, estructura narrativa y recursos expresivos
caractersticos.
Planto de Pleberio
PLEBERIO.Oh Fortuna variable, ministra y mayordoma de los
temporales bienes! Por qu no ejecutaste tu cruel ira, tus
mudables ondas, en aquello que a ti es sujeto? Por qu no
destruiste mi patrimonio? Por qu no quemaste mi morada?
Por qu no asolaste mis grandes heredamientos? Dejrasme
aquella florida planta, en quien t poder no tenas. Dirasme,
fortuna fluctuosa, triste la mocedad con vejez alegre; no
pervirtieras el orden. Mejor sufriera persecuciones de tus
engaos en la recia y robusta edad, que no en la flaca postri-
mera. Oh vida de congojas llena, de miserias acompaada!
Oh mundo, mundo! Muchos mucho de ti dijeron, muchos en
tus cualidades metieron la mano; a diversas cosas por odas te
compararon. Yo por triste experiencia lo contar, como a quien
las ventas y compras de tu engaosa feria no prsperamente
sucedieron, como aquel que mucho ha hasta ahora callado
tus falsas propiedades por no encender con odio tu ira, porque
no me secases sin tiempo esta flor que este da echaste de
tu poder. Pues ahora, sin temor, como quien no tiene qu
perder, como aquel a quien tu compaa es ya enojosa, como
caminante pobre que sin temor de los crueles salteadores va
cantando en alta voz. Yo pensaba en mi ms tierna edad que
eras y eran tus hechos regidos por algn orden; ahora, visto
el pro y la contra de tus bienandanzas, me pareces un labe-
rinto de errores, un desierto espantable, una morada de fieras,
juego de hombres que andan en corro, laguna llena de cieno,
regin llena de espinas, monte alto, campo pedregoso, prado
lleno de serpientes, huerto florido y sin fruto, fuente de cuidados,
ro de lgrimas, mal de miserias, trabajo sin provecho, dulce
ponzoa, vana esperanza, falsa alegra, verdadero dolor. Cbasnos,
mundo falso, con el manjar de tus deleites; al mejor sabor nos des-
cubres el anzuelo: no lo podemos huir, que nos tiene ya cazadas
las voluntades. Prometes mucho, nada cumples. chasnos de ti,
porque no te podamos pedir que mantengas vanos prometimientos.
Corremos por los prados de tus viciosos vicios muy descuidados,
a rienda suelta; descbresnos la celada cuando ya no hay lugar
de volver.
Fernando DE ROJAS
La Celestina, Crtica
DE CABALLERAS SENTIMENTAL
AAAA).A, )
AAAAA )A .
AA-.A )AA A.
El suicidio de Melibea, grabado de la edicin de Valencia de 1514.
CANTAR MESTER DE CLERECA
AA( A.
A AA-AAA
AA)AA A A
AAAu. A, )
0B1LLLA.13 1/4/08 09:58 Pgina 215
REFUERZO
1. Obra potica de Garcilaso
de la Vega
En la obra potica de Garcilaso, el amor es el tema ms importante, con el que se relacionan los
de la naturaleza y la mitologa. Estos temas estn presentes en sus sonetos y en las glogas.
Lee el Soneto XXIX y explica qu le ocurre
a Leandro. Qu sentimientos le impulsan
a esforzarse?
A quines se dirige en los tercetos y qu
les pide?
Qu descripcin de la amada realiza
Nemoroso en la Egloga I ?
A qu imagen corresponden los rasgos
que has sealado en la pregunta anterior?
Qu tpico recrean las interrogaciones que
estructuran el fragmento de la gloga I?
Qu rasgos tiene el paisaje descrito en la
gloga II? A qu tpico responde?
Analiza la mtrica y los principales recursos
expresivos de las tres composiciones.
Relaciona los temas y tpicos de los textos
con los de la poesa renacentista.
4 8
7
6
5
3
2
1
237 14. La lrica renacentista
M
A
T
E
R
I
A
L

F
O
T
O
C
O
P
I
A
B
L
E

/


O
x
f
o
r
d

U
n
i
v
e
r
s
i
t
y

P
r
e
s
s

E
s
p
a

a
,
S
.
A
.
ACTI VI DADES
1
4
Soneto XXIX
Pasando el mar Leandro el animoso,
en amoroso fuego todo ardiendo,
esforz el viento
1
y fuese embraveciendo
el agua con un mpetu furioso.
Vencido del trabajo presuroso, 5
contrastar
2
a las ondas no pudiendo,
y ms del bien que all perda muriendo
que de su propia vida congojoso
3
,
como pudo, sforz su voz cansada
y a las ondas habl desta manera, 10
mas nunca fue su voz dellas oda:
Ondas, pues no se escusa que yo muera,
dejadme all llegar, y a la tornada
4
vuestro furor esecut
5
en mi vida.
gloga II
ALBANIO:
En aquel prado all nos reclinamos,
y, del Cfiro
13
fresco recogiendo
el agradable espirtu
14
, respiramos;
las flores, a los ojos ofreciendo
diversidad estraa de pintura
15
, 5
diversamente as estaban oliendo;
y en medio aquesta fuente clara y pura,
que como de cristal resplandeca,
mostrando abiertamente su hondura,
el arena, que doro pareca, 10
de blancas pedrezuelas varada,
por do manaba el agua, se bulla.
gloga I
NEMOROSO:
D
6
estn agora
7
aquellos claros ojos
que llevaban tras s, como colgada,
mi alma, doquier
8
que ellos se volvan?
Do est la blanca mano delicada,
llena de vencimientos
9
y despojos, 5
que de m mis sentidos l`ofrecan?
Los cabellos que van
con gran desprecio al oro
como a menor tesoro
adnde estn, adnde el blanco pecho? 10
D la columna
10
quel dorado techo
11
con proporcin graciosa sostena?
Aquesto todo agora ya sencierra,
por desventura ma,
en la escura
12
, desierta y dura tierra. 15
1
esforz el viento: se desat el vendaval.
2
contrastar: no pudiendo resistir las olas.
3
congojoso: preocupado.
4
tornada: vuelta.
5
esecut: ejecutad.
6
d: dnde
7
agora: ahora.
8
doquier: donde quiera
9
vencimiento: derrota.
10
columna: el cuello.
11
dorado techo: la rubia cabellera.
12
escura: oscura.
13
Cfiro: viento suave y apacible.
14
espirtu: aire.
15
estraa de pintura: diversidad extraordinaria
de colores.
0B1LLLP(2008).14+s 3/6/08 15:56 Pgina 237
REFUERZO
2. Fray Luis de Len
238 14. La lrica renacentista
M
A
T
E
R
I
A
L

F
O
T
O
C
O
P
I
A
B
L
E

/


O
x
f
o
r
d

U
n
i
v
e
r
s
i
t
y

P
r
e
s
s

E
s
p
a

a
,
S
.
A
.
Adems del tema de la envidia, qu otros
temas desarrolla el primer poema?
Qu tpico aparece? Relaciona este texto
con la Oda a la vida retirada (pgina 227
de tu libro de texto).
Qu suceso religioso se canta en el segundo
poema?
Explica a quin se dirige en la primera estrofa
y qu le reprocha.
Cules sern los sentimientos de los
apstoles, despus de la partida de Jess?
A qu pueden referirse las metforas del mar,
el viento fiero, la nave y el puerto de la
siguiente estrofa?
La nube envidiosa tapa la subida de Cristo,
cmo se quedan los apstoles?
Realiza el anlisis mtrico del primer poema.
Cul es la estrofa utilizada en el segundo?
8
7
6
5
4
3
2
1
Fray Luis sufri un proceso inquisitorial que lo llev a la crcel. El agustino se sinti acosado
y crey que la envidia de otros, sobre todo dominicos, haba sido determinante; a esto
se refiere en el primer poema reproducido. La segunda composicin forma parte de sus
poemas religiosos.
ACTI VI DADES
1
4
A la salida de la crcel
Aqu la envidia y mentira
me tuvieron encerrado.
Dichoso el humilde estado
del sabio que se retira
de aqueste mundo malvado,
y con pobre mesa y casa 5
en el campo deleitoso
con solo Dios se compasa
y a solas su vida pasa,
ni envidiado ni envidioso.
En la Ascensin
Y dejas, Pastor santo,
tu grey
1
en este valle hondo, escuro,
con soledad y llanto;
y t, rompiendo el puro
aire, te vas al inmortal seguro
2
? 5
Lo antes bienhadados
y los agora tristes y afligidos,
a tus pechos criados,
de ti desposedos,
a d convertirn
3
ya sus sentidos? 10
Qu mirarn los ojos,
que vieron de tu rostro la hermosura,
que no les sea enojos?
Quien oy tu dulzura,
qu no tendr por sordo
4
y desventura? 15
Aqueste mar turbado
quin le pondr ya freno?, quin concierto
al viento fiero, airado?;
estando t encubierto,
qu norte guiar la nave al puerto? 20
Ay nube envidiosa!:
aun deste breve gozo, qu te aquejas?
D vuelas presurosa?
Cun rica t te alejas!
Cun pobres y cun ciegos, ay, nos dejas! 25
1
grey: conjunto de fieles cristianos.
2
seguro: lugar desprovisto de todo peligro.
3
convertirn: volvern, dirigirn.
4
sordo: sonido desprovisto de armona.
0B1LLLP(2008).14+s 3/6/08 15:56 Pgina 238
REFUERZO
3. San Juan de la Cruz
El Cntico espiritual es una de las obras maestras de la poesa mstica. En l, san Juan utiliza el
lenguaje simblico para referirse a la bsqueda que realiza la Esposa (el alma) de su amado
(Dios), junto con la valoracin de la naturaleza y numerosos recursos expresivos.
De qu se queja la amada? Cmo se siente?
Qu les pide a los pastores?
Qu decide hacer, segn la tercera estrofa
y a qu est dispuesta?
La amada se dirige a la naturaleza,
cmo se presenta sta? Qu le responde?
Qu le reprocha al amado en la sexta estrofa?
Qu rechaza?
En los versos 31-35 ya se ha producido la
unin, explica cul es el ruego de la esposa,
a dnde le sugiere que vayan?
Qu desea la esposa en la ltima estrofa?
Analiza la mtrica del poema.
Localiza y explica los principales recursos
expresivos empleados por san Juan en estos
versos.
8
7
6
5
4
3
2
1
239 14. La lrica renacentista
M
A
T
E
R
I
A
L

F
O
T
O
C
O
P
I
A
B
L
E

/


O
x
f
o
r
d

U
n
i
v
e
r
s
i
t
y

P
r
e
s
s

E
s
p
a

a
,
S
.
A
.
ACTI VI DADES
1
4
ESPOSA:
Adnde te escondiste, Amado,
y me dejaste con gemido?
Como el ciervo huiste,
habindome herido;
sal tras ti clamando, y eras ido. 5
Pastores, los que fuerdes
1
all por las majadas al otero
2
,
si por ventura vierdes
aquel que yo ms quiero,
decildle que adolezco
3
, peno y muero. 10
Buscando mis amores
ir por esos montes y riberas;
ni coger las flores,
ni temer las fieras,
y pasar los fuertes y fronteras. 15
(PREGUNTA A LAS CRIATURAS)
Oh, bosques y espesuras,
plantadas por la mano del Amado!
Oh, prado de verduras,
de flores esmaltado!,
decid si por vosotros ha pasado 20
RESPUESTA DE LAS CRIATURAS:
Mil gracias derramando,
pas por estos sotos con presura
4
,
e, yndolos mirando,
con sola su figura
vestidos los dej de hermosura. 25
ESPOSA:
Ay, quin podr sanarme!
Acaba de entregarte ya de vero
5
;
no quieras enviarme
de hoy ms ya mensajero,
que no saben decirme lo que quiero. [] 30
ESPOSA:
Gocmonos, Amado,
y vmonos a ver en tu hermosura
al monte o al collado,
do mana el agua pura;
entremos ms adentro en la espesura. 35
Y luego a las subidas
cavernas de la piedra nos iremos,
que estn bien escondidas,
y all nos entraremos
y el mosto de granadas gustaremos. 40
All me mostraras
aquello que mi alma pretenda,
y luego me daras
all, t, vida ma,
aquello que me diste el otro da: 45
el aspirar
6
del aire,
el canto de la dulce filomena
7
,
el soto y su donaire
en la noche serena,
con llama que consume y no da pena. 50
San Juan DE LA CRUZ
Cntico espiritual, Crtica
1
fuerdes / vierdes: formas arcaicas de fueres
y vieres.
2
otero: cerro aislado que domina un llano.
3
adolecer: enfermar.
4
presura: prisa, prontitud, ligereza.
5
de vero: de verdad.
6
aspirar: el soplo.
7
filomena: el ruiseor.
Cntico espiritual
0B1LLLP(2008).14+s 3/6/08 15:56 Pgina 239
AMPLIACIN
4. Lrica renacentista
En esta pgina se incluyen poemas de otros autores renacentistas, que siguieron el modelo de Garcilaso
de la Vega: Gutierre de Cetina (1510-1554), Fernando de Herrera (1534-1597), Francisco de Aldana
(1537-1578). Del bellsimo ltimo texto, annimo, no se sabe exactamente su fecha.
En qu situacin se halla el yo potico en el
texto 1? Cules son sus sentimientos?
Cules son sus deseos y sus temores, segn
los tercetos? Qu temas desarrolla este texto?
Qu descripcin se hace de la amada en el
texto 2?
Qu visin de ella nos ofrece la voz potica?
Relaciona esta imagen con las ideas del
neoplatonismo.
En el texto 3, hay una escena de amor
correspondido; sin embargo muestra una
situacin dolorosa, cul es?
Cul es la reaccin y la respuesta del
amante?
Explica las razones del amor a Cristo del yo
potico en el ltimo texto.
Relaciona los temas de estos poemas con los
de la lrica renacentista.
8
7
4
5
6
3
2
1
240 14. La lrica renacentista
M
A
T
E
R
I
A
L

F
O
T
O
C
O
P
I
A
B
L
E

/


O
x
f
o
r
d

U
n
i
v
e
r
s
i
t
y

P
r
e
s
s

E
s
p
a

a
,
S
.
A
.
ACTI VI DADES
1
4
Texto 1
Entre armas, guerra, fuego, ira y furores,
que al soberbio francs tienen opreso
1
,
cuando el aire es ms turbio y ms espeso,
all me aprieta el fiero ardor de amores.
Miro el cielo, los rboles, las flores, 5
y en ellos hallo mi dolor expreso;
que en el tiempo ms fro y ms avieso
2
nacen y reverdecen mis temores.
Digo llorando: Oh dulce primavera!,
cundo ser que a mi esperanza vea, 10
verde, prestar al alma algn sosiego?
Mas temo que mi fin mi suerte fiera
tan lejos de mi bien quiere que sea
entre guerra y furor, ira, armas, fuego.
Gutierre DE CETINA
1
opreso: oprimido.
2
avieso: malo, torcido.
Texto 2
Serena Luz, en quien presente espira
3
divino amor, que enciende y junto enfrena
4
el noble pecho, que en mortal cadena
al alto Olimpo levantarse aspira;
ricos cercos dorados, do se mira 5
tesoro celestial de eterna vena; ,
armona de anglica sirena,
que entre las perlas y el coral respira,
cul nueva maravilla, cul ejemplo
de la inmortal grandeza nos descubre 10
aquesta sombra del hermoso velo?
Que yo en esa belleza que contemplo
(aunque a mi flaca vista ofende y cubre),
la inmensa busco, y voy siguiendo al cielo.
Fernando DE HERRERA
3
espira: sopla.
4
enfrena: frena.
Texto 3
De sus hermosos ojos dulcemente
un tierno llanto Filis despeda
que por el rostro amado pareca
claro y precioso aljfar
5
transparente;
en brazos de Damn, con baja frente, 5
triste, rendida, muerta, helada y fra,
estas palabras breves le deca,
creciendo a su llorar nueva corriente:
Oh pecho duro, oh alma dura y llena
de mil durezas! Dnde vas huyendo? 10
D vas con ala tan ligera y presta?
Y l, soltando de llanto amarga vena,
della las dulces lgrimas bebiendo,
besola, y solo un ay! fue su respuesta.
Francisco DE ALDANA
5
aljfar: perla pequea de forma irregular.
Texto 4
Soneto a Cristo crucificado
No me mueve, mi Dios, para quererte
el cielo que me tienes prometido;
ni me mueve el infierno tan temido
para dejar por eso de ofenderte.
T me mueves, seor; muveme el verte 5
clavado en una cruz y escarnecido;
muveme ver tu cuerpo tan herido;
muvenme tus afrentas y tu muerte.
Muveme, en fin, tu amor, y en tal manera
que aunque no hubiera cielo, yo te amara, 10
y aunque no hubiera infierno, te temiera.
No tienes que me dar porque te quiera,
pues aunque cuanto espero no esperara,
lo mismo que te quiero te quisiera.
ANNIMO
Poesa lrica del Siglo de Oro, Ctedra
0B1LLLP(2008).14+s 3/6/08 15:56 Pgina 240
Evaluacin 1
4
233 14. La lrica renacentista
1. Explica el contenido de este fragmento de la gloga II,
de Garcilaso de la Vega.
2. Menciona el tema del poema y reconoce los tpicos que
aparecen.
3. Justifica el carcter literario y el gnero de este texto.
4. Seala los principales recursos expresivos y realiza el
anlisis mtrico de la composicin.
5. Atendiendo a tus respuestas, relaciona el tema y los
tpicos, los recursos expresivos y la mtrica de estos
versos con los principales rasgos de la lrica renacentista.
6. El fragmento citado pertenece a la gloga II. Explica qu es
una gloga y comenta el contenido de las tres que
es cribi Garcilaso de la Vega. Menciona tambin los
temas principales de su produccin potica.
7. Qu innovaciones de la lrica renacentista proceden de
Italia? Por qu se dio esta influencia?
8. Qu tipos de poesa religiosa se cultivaron en la segunda
mitad del siglo &VI? Cita los ttulos de los poemas ma -
yores de san Juan de la Cruz, explica el asunto principal
abordado en su poesa y el recurso bsico que utiliza.
9. Cules son los temas y tpicos ms importantes de la
Oda a la vida retirada, de fray Luis de Len? Explica qu
mtrica utiliza en la mayora de sus poemas.
10. Redacta un texto con las etapas y las tendencias poticas
de la poesa del siglo &VI. Cita autores y obras.
El lamento de Albanio
En medio del invierno est templada
el agua dulce desta clara fuente,
y en el verano ms que nieve helada.
Oh claras ondas, cmo veo presente,
en vindoos, la memoria da quel da 5
de que el alma temblar y arder se siente!
En vuestra claridad vi mi alegra
oscurecerse toda y enturbiarse;
cuando os cobr, perd mi compaa.
A quin pudiera igual tormento darse, 10
que con lo que descansa otro afligido
venga mi corazn a atormentarse?
El dulce murmurar deste rido,
el mover de los rboles al viento,
el suave olor del prado florecido 15
podrian tornar denfermo y descontento
cualquier pastor del mundo alegre y sano:
yo solo en tanto bien morir me siento.
Oh hermosura sobrel ser humano,
oh claros ojos, oh cabellos doro, 20
oh cuello de marfil, oh blanca mano!
Cmo puede ora ser quen triste lloro
se convirtiese tan alegre vida
y en tal pobreza todo mi tesoro?
Quiero mudar lugar y a la partida 25
quiz me dejar parte del dao
que tiene el alma casi consumida.
Concierto campestre, por Tiziano.
0B1LLLA.14 7/4/08 15:51 Pgina 233
251 15. La narrativa renacentista
M
A
T
E
R
I
A
L

F
O
T
O
C
O
P
I
A
B
L
E

/


O
x
f
o
r
d

U
n
i
v
e
r
s
i
t
y

P
r
e
s
s

E
s
p
a

a
,
S
.
A
.
ACTI VI DADES
1
5
AMPLIACIN
1. Novela bizantina y novela pastoril
Resume el contenido de ambos fragmentos.
Explica por qu constituyen textos narrativos.
Reconoce en los dos pasajes:
Narrador.
Movimientos narrativos.
Tipos de discurso.
Seala los recursos expresivos con los que
se materializa en cada uno de los fragmentos
la funcin potica.
Indica los rasgos por los que los textos
reproducidos en esta pgina se adscriben
a la novela pastoril, el primero, y a la bizantina,
el segundo.
5
4
3
2
1
Los amores de Clareo y Florisea y los trabajos de la sin ventura Isea, de Alonso Nez de Reinoso,
y La Diana, de Jorge de Montemayor, son obras paradigmticas de la novela bizantina y de
la novela pastoril, respectivamente, en la pennsula.
En la fuente de los alisos
Despus que Sireno puso fin a su canto, vio como hacia l vena la hermosa Selvagia, y el pastor
Sylvano, de que no recibi pequeo contentamiento, y despus de haberse recibido, determinaron ir a
la fuente de los alisos, donde el da antes haban estado. Y primero que all llegasen, dijo Sylvano:
Escucha, Selvagia, no oyes cantar? S oigo dijo Selvagia, y aun parece ms de una voz. A dn-
de ser? dijo Sireno. Parceme respondi Selvagia que es en el prado de los laureles, por donde
pasa el arroyo que corre de esta clara fuente. Bien ser que nos lleguemos all, y de manera que no
nos sientan los que cantan, porque no interrumpamos la msica. Vamos dijo Selvagia. Y as su
paso a paso se fueron hacia aquella parte donde las voces se oan, y escondindose entre unos rboles
que estaban junto al arroyo, vieron sobre las doradas flores asentadas tres ninfas, tan hermosas que
pareca haber en ellas dado la naturaleza muy clara muestra de lo que puede.
Venan vestidas de unas ropas blancas, labradas por encima de follajes de oro, sus cabellos que los
rayos del sol oscurecan, revueltos a la cabeza, y tomados con sendos hilos de orientales perlas, con que
encima de la cristalina frente se haca una lazada, y en medio della estaba un guila de oro, que entre
las uas tena un muy hermoso diamante.
Jorge DE MONTEMAYOR
La Diana, Ctedra
La nsula de la crueldad
Pasadas todas estas cosas, yo determin de partirme en Efeso; y mandando a fletar una nao y despi-
dindome de Ibrina y de su marido, nos embarcamos y comenzamos de navegar la va de la ciudad de
Efeso. Y habiendo algunos das que con buen tiempo hacamos nuestro camino, vimos de lejos una isla,
la cual pareca tan oscura que la noche no lo es tanto. Pareca que unos humos negros de azufre salan
della; las casas y arboledas eran todas negras y de negra color, las aguas que por ella corran eran
todas de color de sangre; oanse grandes y dolorosos gritos y grandes alaridos que ponan espanto
a los que los oan. Y ans, nosotros quedamos espantados y con deseo de saber qu aventura fuese
aquella. Y porque ya era de noche, mandamos a los marineros que detuviesen la nao hasta la maana
porque queramos saber qu tierra fuese aquella. Ellos lo hiceron ans y dijeron que era mejor tomar
tierra porque aquella, aunque tan triste pareciese, era segura, a la cual se llamaba la nsula de la
Crueldad, porque en ella estaban sepultados todos aquellos que cruelmente haban sido muertos, y que
ans era el uso de aquellas tierras que estaban cercanas y de ms lejos, que era traer all a sepultallos.
Yo quisiera que pasramos adelante por no quedar all en tan triste tierra, pero Clareo no lo consinti,
antes mand tomar tierra. Y ans lo hicimos; y saliendo al puerto nos quedamos all la noche, la cual fue
tan larga que pensamos que nunca amaneciese, y era la causa porque el sol no pareca en aquella isla,
antes hua della. Pero habiendo entrado el da, aunque oscuro, bien conocimos que era ya de da, y
comenzando a mirar la tierra vimos muchas sepulturas de muchas personas que cruelmente haban
sido muertas.
Alonso NEZ DE REINOSO
Los amores de Clareo y Florisea y los trabajos de la sin ventura Isea, SPUE
0B1LLLP(2008).15+s 3/6/08 15:57 Pgina 251
252 15. La narrativa renacentista
M
A
T
E
R
I
A
L

F
O
T
O
C
O
P
I
A
B
L
E

/


O
x
f
o
r
d

U
n
i
v
e
r
s
i
t
y

P
r
e
s
s

E
s
p
a

a
,
S
.
A
.
ACTI VI DADES
1
5
AMPLIACIN
2. Novela celestinesca
y novela morisca
El regreso de Celestina
ARESA.Ay Jess, que me muero de miedo!
ELICIA.Ay, hermana ma, que mi madre Celestina
parece! Ay, vlame la Virgen Mara, y no sea algn
fantasma que nos quiera matar!
CELESTINA.Ay bobas, y no hayis miedo, que yo soy!
Las mis hijas y los mis amores, venidme abrazar,
y dad las gracias a Dios que ac tornar me dej!
As juntas os quiero abrazar, que no tendr sufri-
miento para ms esperar.
ELICIA.Ay Jess, Jess! Vlame Dios! Ay madre, des-
vate all que me muero de miedo, que pienso que
eres muerta!
CELESTINA.Ay boba, bobita!; y de qu das gritos,
loca? No hayas miedo, mi amor y mi hija y las mis
entraas, que yo soy tu ta, que viva soy y no muer-
ta; abrzame, loca, qu te escandalizas? Osadas,
que si fuera hombre y mozo como soy mujer y vieja
que no te espantaras de me ver ni de me abrazar.
ELICIA.Ay ta, seora, y bien seas venida! Y para
qu dices malicias en mal hora y en mal punto?
Y qu hombre pudiera venir a quien yo ms qui-
siera que a ti?
CELESTINA.Aquel Sempronio, que a osadas, hija, que
no estuviera bien abrazallo segn queda, y yo lo vi,
en los infiernos abrasado.
Feliciano DE SILVA
Segunda Celestina, Ctedra
La victoria de Narvez
Rodrigo de Narvez, barruntando la necesidad en
que sus compaeros estaban, atraves el camino,
y como traa mejor caballo se adelant; y viendo la
valenta del moro, qued espantado, porque de los
cinco escuderos tena los cuatro en el suelo, y el otro,
casi al mismo punto. l le dijo:
Moro, vente a m, y si t me vences, yo te asegu-
ro de lo dems.
Y comenzaron a trabar brava escaramuza, mas
como el alcaide vena de refresco, y el moro y su
caballo estaban heridos, dbale tanta priesa que
no poda mantenerse; mas viendo que en sola esta
batalla le iba la vida y contentamiento, dio una lan-
zada a Rodrigo de Narvez que, a no tomar el golpe
en su darga, le hubiera muerto. l, en recibiendo el
golpe, arremeti a l y diole una herida en el brazo
derecho, y cerrando luego con l, le trab a brazos
y, sacndole de la silla, dio con l en el suelo. Y yendo
sobre l le dijo:
Caballero, date por vencido; si no, matarte he.
Matarme bien podrs dijo el moro que en
tu poder me tienes, mas no podr vencerme sino
quien una vez me venci.
El alcaide no par en el misterio con que se de -
can estas palabras, y usando en aquel punto de su
acostumbrada virtud, le ayud a levantar, porque de
la herida que le dio el escudero en el muslo y de la del
brazo, aunque no eran grandes, y del gran cansancio
y cada, qued quebrantado; y tomando de los escu-
deros aparejo, le lig las heridas.
El Abencerraje (novela y romancero)
Ctedra
Resume el contenido de ambos fragmentos.
Explica la peculiaridad del discurso
del primer texto.
Reconoce en el segundo texto los siguientes
aspectos:
Narrador.
Movimiento narrativo.
Tipos de discurso.
Seala los principales recursos expresivos
con los que se materializa en los fragmentos
de Segunda Celestina y del Abencerraje
la funcin potica.
Indica los rasgos por los que estos textos
pertenecen a la novela celestinesca,
el primero, y a la morisca, el segundo.
No olvides argumentar tu respuesta.
5
4
3
1
2
A continuacin presentamos un par de textos representativos de dos de los modelos narrativos
cultivados en el siglo XVI: la novela celestinesca y la novela morisca. Se trata, respectivamente,
de Segunda Celestina, de Feliciano de Silva, y de El Abencerraje, que apareci incorporado en una
edicin de La Diana.
0B1LLLP(2008).15+s 3/6/08 15:57 Pgina 252
253 15. La narrativa renacentista
M
A
T
E
R
I
A
L

F
O
T
O
C
O
P
I
A
B
L
E

/


O
x
f
o
r
d

U
n
i
v
e
r
s
i
t
y

P
r
e
s
s

E
s
p
a

a
,
S
.
A
.
ACTI VI DADES
1
5
REFUERZO
3. Lazarillo de Tormes
El siguiente fragmento corresponde al Tratado III, que pertenece al primer mdulo de la obra,
donde se cuenta la infancia de Lzaro de Tormes.
Resume el contenido de este fragmento,
correspondiente al Tratado III del Lazarillo
y caracteriza el comportamiento
del escudero.
En el primer prrafo se observa un cambio
en el uso de los tiempos verbales. Qu efecto
produce? Este recurso se reitera ms adelante;
localzalo.
Qu movimiento narrativo se aprecia en la
primera parte del fragmento? Por qu tipo
de discurso es interrumpido?
Indica a qu tipo de discurso pertenece
la exclamacin final.
Seala y explica el empleo del recurso de
la irona en las palabras del escudero.
5
4
3
2
1
La maana venida, levantmonos, y comienza
a limpiar y sacudir sus calzas
1
y jubn
2
y sayo
3
y
capa; y yo que le serva de pelillo
4
. Y vsteseme muy
a su placer, despacio. Echele aguamanos
5
, peinose
y psose su espada en el talabarte
6
y, al tiempo que
la pona, djome:
Oh, si supieses, mozo, qu pieza es esta! No
hay marco de oro en el mundo por que yo la diese.
Mas ans ninguna de cuantas Antonio hizo no
acert a ponelle
7
los aceros tan prestos como esta
los tiene.
Y sacola de la vaina y tentola con los dedos,
diciendo:
Vesla aqu? Yo me obligo con ella cercenar
un copo de lana.
Y yo dije entre m: Y yo con mis dientes, aunque
no son de acero, un pan de cuatro libras.
Tornola a meter y cisela, y un sartal
8
de cuen-
tas gruesas del talabarte. Y con un paso sosegado
y el cuerpo derecho, haciendo con l y con la cabeza
muy gentiles meneos, echando el cabo de la capa
sobre el hombro y a veces so el brazo, y poniendo la
mano derecha en el costado, sali por la puerta,
diciendo:
Lzaro, mira por la casa en tanto que yo voy a
or misa, y haz la cama y ve por la vasija de agua al
ro, que aqu bajo est, y cierra la puerta con llave,
no nos hurten algo, y ponla aqu al quicio, porque si
yo viniere en tanto pueda entrar.
Y sbese por la calle arriba con tal gentil sem-
blante y continente, que quien no lo conociera pen-
sara ser muy cercano pariente al conde de Arcos,
o al menos camarero que le daba de vestir.
Bendito seis Vos, Seor qued yo dicien-
do, que dais la enfermedad y ponis el remedio!
Quin encontrar a aquel mi seor que no piense,
segn l contento de s lleva, haber anoche bien
cenado y bien dormido en buena cama, y, aun
agora es de maana, no le cuenten por muy bien
almorzado? Grandes secretos son, Seor, los que
Vos hacis y las gentes ignoran! A quin no empe-
ar aquella buena disposicin y razonable capa
y sayo? Y quin pensar que aquel gentil hombre
se pas todo el da sin comer, con aquel mendrugo
de pan que su criado Lzaro trujo
9
un da y una
noche en el arca de su seno, do no se le poda
pegar mucha limpieza, y hoy, lavndose las
manos y cara, a falta de pao de manos se haca
servir de la halda
10
del sayo? Nadie, por cierto, lo
sospechar. Oh, Seor, y cuntos de aquestos
debis Vos tener por el mundo derramados, que
padecen por la negra que llaman honra lo que por
Vos no sufriran!
Lazarillo de Tormes
Ctedra
Al servicio del escudero
1
calza: prenda que cubra, cindolos, el muslo
y la pierna.
2
jubn: vestidura que cubra desde los hombros hasta
la cintura, ceida y ajustada al cuerpo.
3
sayo: prenda de vestir holgada y sin botones que
cubra el cuerpo hasta las rodillas.
4
servir de pelillo: hacer servicios de poca importancia.
5
aguamanos: aguamanil; agua que sirve para lavar las
manos.
6
talabarte: cinturn de cuero del que cuelga la espada.
7
ponelle: ponerle.
8
sartal: sarta de cosas metidas en un hilo o cuerda.
9
trujo: trajo (de traer).
10
halda: falda.
0B1LLLP(2008).15+s 3/6/08 15:57 Pgina 253
254 15. La narrativa renacentista
M
A
T
E
R
I
A
L

F
O
T
O
C
O
P
I
A
B
L
E

/


O
x
f
o
r
d

U
n
i
v
e
r
s
i
t
y

P
r
e
s
s

E
s
p
a

a
,
S
.
A
.
ACTI VI DADES
1
5
Comenta los temas que se desarrollan
en el dilogo entre don Quijote y los duques
y relacinalos con las caractersticas del
personaje protagonista y con lo que conoces
del argumento de la novela.
Repasa las principales caractersticas de las
novelas de caballeras y del concepto del amor
corts y relacinalas con lo que expresa
el caballero.
Qu responde el protagonista respecto
al comentario de que Dulcinea es una dama
fantstica?
Explica las caractersticas de la locura
de don Quijote.
Analiza los recursos expresivos utilizados
en la primera y la ltima intervencin de
don Quijote en el fragmento que acabas
de leer.
4
5
3
2
1
REFUERZO
4. Don Quijote de la Mancha
El siguiente texto pertenece a la segunda parte del Quijote. Los protagonistas han llegado al palacio
de los duques, quienes, habiendo ledo la primera parte del libro, deciden invitarlos para divertirse
a su costa. La duquesa le pide al caballero que describa a Dulcinea, y este comienza su respuesta
aludiendo al encantamiento de que ha sido objeto (recuerda el engao de Sancho).
S hiciera, por cierto respondi
don Quijote, si no me la hubiera borrado
de la idea la desgracia que poco ha que
le sucedi, que es tal, que ms estoy
para llorarla que para describirla, por-
que habrn de saber vuestras gran dezas
que yendo los das pasados a besarle las
manos, y a recebir su bendicin, benepl -
cito y licencia para esta tercera salida, hall
otra de la que buscaba: hallela encantada
y convertida de princesa en labradora, de
hermosa en fea, de ngel en diablo, de olorosa
en pestfera, de bien hablada en rstica, de repo sada
en brincadora, de luz en tinieblas, y, finalmente, de
Dulcinea del Toboso en una villana de Sayago.
Vlame Dios! dando una gran voz, dijo a este instante
el duque. Quin ha sido el que tanto mal ha hecho al mun-
do? Quin ha quitado dl la belleza que le alegraba, el donaire
que le entretena y la honestidad que le acreditaba?
Quin? respondi don Quijote. Quin puede ser sino
algn maligno encantador de los muchos invidiosos que me
persiguen? Esta raza maldita, nacida en el mundo para escure-
cer y aniquilar las hazaas de los buenos, y para dar luz y levan-
tar los fechos de los malos. Perseguido me han encantadores,
encantadores me persiguen, y encantadores me perseguirn
hasta dar conmigo y con mil altas caballeras en el profundo
abismo del olvido, y en aquella parte que me daan y hieren
donde ven que ms lo siento, porque quitarle a un caballero
andante su dama es quitarle los ojos con que mira, y el sol con
que se alumbra, y el sustento con que se mantiene. Otras
muchas veces lo he dicho, y ahora lo vuelvo a decir; que el caba-
llero andante sin dama es como el rbol sin hojas, el edificio sin
cimiento, y la sombra sin cuerpo de quien se cause.
No hay ms que decir dijo la duquesa; pero si, con
todo eso, hemos de dar crdito a la historia que del seor don
Quijote de pocos das a esta parte ha salido a la luz del mundo,
con aplauso de las gentes, della se colige, si mal no me acuerdo,
que nunca vuesa merced ha visto a la seora Dulcinea.
Y que esta tal seora no es en el mundo, sino que es dama
fantstica, que vuesa merced la engendr y pari en su entendi-
miento, y la pint con todas aquellas gracias y perfecciones que
quiso.
En eso hay mucho que decir respondi don Quijote.
Dios sabe si hay Dulcinea o no en el mundo, o si es fantstica, o
no es fantstica; y estas no son de las cosas cuya averiguacin
se ha de llevar hasta el cabo. Ni yo engendr ni par a mi seora,
puesto que la contemplo como conviene que sea una dama
que contenga en s las partes que puedan hacerla famosa en
todas las del mundo, como son: hermosa sin tacha, grave sin
soberbia, amorosa con honestidad, agradecida por corts, cor-
ts por bien criada y, finalmente, alta por linaje, a causa que
sobre la buena sangre resplandece y campea la hermosura con
ms grados de perfeccin que en las hermosas humildemente
nacidas.
Miguel DE CERVANTES
Don Quijote de la Mancha
Planeta
El encantamiento de Dulcinea
0B1LLLP(2008).15+s 3/6/08 15:58 Pgina 254
255 15. La narrativa renacentista
M
A
T
E
R
I
A
L

F
O
T
O
C
O
P
I
A
B
L
E

/


O
x
f
o
r
d

U
n
i
v
e
r
s
i
t
y

P
r
e
s
s

E
s
p
a

a
,
S
.
A
.
ACTI VI DADES
1
5
AMPLIACIN
5. Una imagen de mujer
en el Quijote
A continuacin se presentan dos textos del Quijote. El primero corresponde al discurso que pro-
nuncia el caballero sobre la Edad de Oro. El segundo fragmento pertenece a la historia pastoril
intercalada de Marcela y Grisstomo. Ambos jvenes hermosos y ricos campesinosdeci-
den hacerse pastores: Marcela porque prefiere este tipo de vida; Grisstomo porque pretende
conquistarla. Marcela lo rechaza, pues no desea casarse, y Grisstomo se quita la vida. En este
texto, la pastora hace su alegato.
Redactad en grupos un informe en el que
se relacionen los siguientes aspectos.
Los pensamientos de don Quijote sobre
la Edad de Oro.
El alegato de Marcela.
Las ideas de Gelasia plasmadas en el soneto
de la pgina 244 del Libro del alumno.
Escribe un texto en el que reflexiones acerca
de la vinculacin de la imagen de la mujer
independiente planteada en estos textos
con las concepciones de la mujer vigentes
en la sociedad actual.
Analiza los principales recursos expresivos
utilizados en el texto 2.
3
2 1
Texto 1
Dichosa edad y siglos dichosos aquellos a quien los antiguos pusieron nombre de dora-
dos, y no porque en ellos el oro, que en esta nuestra edad de hierro tanto se estima, se alcan-
zase en aquella venturosa sin fatiga alguna, sino porque entonces los que en ella vivan
ignoraban estas dos palabras de tuyo y mo []. Las doncellas y la honestidad andaban,
como tengo dicho, por dondequiera, sola y seora, sin temor que la ajena desenvoltura y lasci-
vo intento la menoscabasen, y su perdicin naca de su gusto y propia voluntad.
Miguel DE CERVANTES
Don Quijote de la Mancha
Planeta
Texto 2
Yo nac libre, y para poder vivir libre escog la soledad de los campos. Los rboles de estas
montaas son mi compaa, las claras aguas de estos arroyos mis espejos; con los rboles
y con las aguas comunico mis pensamientos y hermosura. Fuego soy apartado y espada
puesta lejos. A los que he enamorado con la vista he desengaado con las palabras. Y si los
deseos se sustentan con esperanzas, no habiendo yo dado alguna a Grisstomo ni a otro
alguno, en fin de ninguno de ellos, bien se puede decir que antes le mat su porfa que mi
crueldad. Y si se me hace cargo que eran honestos sus pensamientos, y que por esto esta-
ba obligada a corresponder a ellos, digo que cuando en este mismo lugar donde ahora se
cava su sepultura me descubri la bondad de su intencin, le dije yo que la ma era vivir en
perpetua soledad, y que sola la tierra gozase el fruto de mi recogimiento y los despojos de
mi hermosura; y si l, con todo este desengao, quiso porfiar contra la esperanza y navegar
contra el viento, qu mucho que se anegase en la mitad del golfo de su desatino? [] Si yo
conservo mi limpieza con la compaa de los rboles, por qu ha de querer que la pierda el
que quiere que la tenga con los hombres? Yo, como sabis, tengo riquezas propias y no
codicio las ajenas; tengo libre condicin y no gusto de sujetarme; ni quiero ni aborrezco
a nadie. No engao a este, ni solicito a aquel; ni burlo con uno, ni me entretengo con el otro.
La conversacin honesta de las zagalas destas aldeas y el cuidado de mis cabras me
entretiene. Tienen mis deseos por trmino estas montaas, y si de aqu salen, es a contemplar
la hermosura del cielo, pasos con que camina el alma a su morada primera.
Miguel DE CERVANTES
Don Quijote de la Mancha
Planeta
0B1LLLP(2008).15+s 3/6/08 15:58 Pgina 255
Evaluacin 1
5
255 15. La narrativa renacentista
1. Resume el contenido de este fragmento del Lazarillo
de Tormes.
2. Caracteriza los elementos del nivel de la historia presen-
tes en el fragmento.
Personajes. Acciones.
Tiempo. Espacio.
3. Explica qu descubre Lzaro en este episodio. De qu
mdulo o mdulos de la estructura de la obra es una
constante ese descubrimiento?
4. Identifica el movimiento narrativo presente en el frag-
mento, extrae ejemplos de discurso valorativo y univer-
sal, y seala el recurso de la irona.
5. Caracteriza al protagonista del Lazarillo de Tormes y
compralo con los protagonistas de las novelas de caba-
lleras, sentimental, morisca y bizantina.
6. Qu estilos propios de la narrativa anterior estn
presentes en el Quijote? Argumenta por qu razn se la
considera la primera novela moderna.
7. Explica la estructura narrativa de esta clebre obra
cervantina, caracteriza a sus protagonistas y enuncia
sus temas principales.
8. Menciona las caractersticas de las Novelas ejemplares de
Miguel de Cervantes atendiendo a los aspectos que se
indican a continuacin.
Caractersticas temticas
Caractersticas estructurales
9. Qu otras obras narrativas escribi Cervantes? Indica a
qu tipo de novela renacentista pertenecen.
10. Redacta una exposicin sobre el tratamiento del amor
en la narrativa renacentista.
Encuentro con el escudero
T, mozo, has comido?
No, seor dije yo , que an no eran dadas las ocho
cuando con Vuestra Merced encontr.
Pues, aunque de maana, yo haba almorzado, y cuando
ans como algo, hgote saber que hasta la noche me estoy ans.
Por eso, psate como pudieres, que despus cenaremos.
Vuestra Merced crea, cuando esto o, que estuve en poco de caer
de mi estado, no tanto de hambre como por conocer de todo en
todo la fortuna serme adversa. All se me presentaron de nuevo
mis fatigas y torn a llorar de mis trabajos. All se me vino a la
memoria la consideracin que haca cuando me pensaba ir del
clrigo, diciendo que, aunque aquel era desventurado y msero,
por ventura topara con otro peor. Finalmente, all llor mi trabajosa
vida pasada y mi muerte venidera. Y con todo ello, disimulando lo
mejor que pude:
Seor, mozo soy, que no me fatigo mucho por comer, bendito
Dios. De eso me podr yo alabar entre todos mis iguales, por de
mejor garganta, y ans fui yo loado de ella hasta hoy da de los
amos que yo he tenido.
Virtud es esa dijo l , y por eso te querr yo ms. Porque
el hartar es de los puercos y el comer regaladamente es de los
hombres de bien.
[ ] Pseme a un cabo del portal y saqu unos pedazos de pan
del seno, que me haban quedado de los de por Dios. l que vio
esto, djome:
Ven ac, mozo. Qu comes?
Yo llegueme a l y mostrele el pan. Tomome l un pedazo de
tres que eran, el mejor y el ms grande. Y djome:
Por mi vida, que parece este buen pan.
Y cmo, agora dije yo, seor, es bueno?
S, a fe dijo l . Adnde lo hubiste? Si es amasado de
manos limpias?
N o s yo eso l e dije, mas a m no me pone asco el sabor
de ello.
As plega a Dios dijo el pobre de mi amo.
Y llevndolo a la boca, comenz a dar en l tan fieros bocados
como yo en lo otro.
Lazarillo de Tormes
Ctedra
Retrato doble de dos jvenes, por Giorgione.
0B1LLLA.15 1/4/08 10:08 Pgina 255
AMPLIACIN
1. La apariencia barroca
La ideologa del barroco, condicionada por el contexto histrico, social y religioso, presenta una
serie de rasgos que la diferencian de la de otros perodos y que se manifiestan en todas las pro-
ducciones culturales. En el siguiente texto se trata uno de sus componentes esenciales.
La apariencia
No hace falta recordar que la imaginacin fue el gran componente del desarro-
llo cultural de la pennsula ibrica en el siglo XVII. Una sociedad de cuyo acervo cul-
tural forman parte los milagros tiene que ser, a la fuerza, una sociedad imaginativa;
incluso la cantidad de plagios, copias, versiones, variaciones, amontonamiento de
citas y rplicas no hacen ms que atestiguar una actividad incesante, una reflexin
minimalista, detallista, barroca, que descubre y respira en el cmulo de las infinitas
posibilidades de lo que puede ser y lo que es.
La gran pasin del siglo es la apariencia: la vida es sueo, desconfiemos de los
sentidos, he aqu la reflexin inicial del pensamiento barroco. Esta desconfianza de
los sentidos no tiene tanto que ver con la desconfianza en la percepcin, sino, al
contrario, con una fe en el poder de seduccin y de comunicacin de la imagen, que
imposibilita discernir entre la apariencia y la realidad. Mis ojos no me engaan res-
pecto de lo que veo, sino respecto de la naturaleza de lo que veo. Por lo tanto el
engao lo es del valor de las cosas: el oro resulta finalmente ser oropel. Precisamente
por ser esta una poca transida de smbolos, el engao y el equvoco fluctan
siempre en torno a un error, provocado o fortuito, en la interpretacin. El drama Del
Rey abajo ninguno se construye en torno a la confusin creada por una banda que
debera llevar al rey y lleva a otra persona; los consejos del hidalgo de El Buscn
giran en torno a cmo engaar en la apariencia, es decir, a cmo sostener la dife-
rencia entre lo que el espectador ve y lo que la capa sabe [].
Encontramos este mismo inters en la poltica, mientras que en el mbito terico
y cientfico esta inquietud por los reflejos de una realidad evasiva, se manifestar de
dos formas distintas. La primera, acumulando maravillas, todo aquello que al ojo
pueda, por distintos motivos, sorprender, agradar o enfadar; dicho de otro modo,
haciendo recuento y acopio de todos los casos, de todos los objetos, de todas las for-
mas que presentan una combinacin original, una excrecencia, una composicin
excelsa. La segunda, intentando establecer un orden en esa multitud de aparien-
cias, intentando crear modos de medir, de fijar y comprender las relaciones pticas
con la realidad: desde la naturaleza de la luz y el funcionamiento del ojo, a la medi-
cin de las distancias y la estimacin de las alturas. Refraccin, reflexin, sombra,
proyeccin, movimientos aparentes, puntos fijos, perspectiva, gnomnica son solo
una parte de los intereses de este siglo.
Nuria VALVERDE
El prodigio en su medida,
en Madrid, ciencia y Corte, Consejera de Educacin y Cultura
Cul es la gran pasin del siglo XVII?
En qu creencia se basa?
Relaciona la afirmacin anterior con
el poema de Argensola de la pgina 260
del Libro del alumno.
Cmo se manifiesta este inters por la
apariencia en el mbito tcnico y cientfico?
Cita algunos ejemplos.
La confianza en el poder de persuasin
y fascinacin de las imgenes se manifiesta
en la proliferacin de otros tipos
de actividades. De cules se trata?
Tomando en cuenta las respuestas anteriores,
realiza un resumen del texto.
Cita algunos poemas ledos en esta unidad
en los que se trate el tema del engao.
6
5
4
3
2
1
275 16. La poesa y la prosa barrocas
M
A
T
E
R
I
A
L

F
O
T
O
C
O
P
I
A
B
L
E

/


O
x
f
o
r
d

U
n
i
v
e
r
s
i
t
y

P
r
e
s
s

E
s
p
a

a
,
S
.
A
.
ACTI VI DADES
16
0B1LLLP(2008).16+s 3/6/08 15:54 Pgina 275
REFUERZO
2. Sonetos de Luis de Gngora
276 16. La poesa y la prosa barrocas
M
A
T
E
R
I
A
L

F
O
T
O
C
O
P
I
A
B
L
E

/


O
x
f
o
r
d

U
n
i
v
e
r
s
i
t
y

P
r
e
s
s

E
s
p
a

a
,
S
.
A
.
Qu elementos de su ciudad natal, alaba
Gngora en los cuartetos de A Crdoba?
Explica cmo expresa la aoranza y el amor
a su tierra en los tercetos.
Qu recurso aparece en el ltimo terceto?
Segn los cuartetos del segundo texto,
cmo se siente la voz potica?
Qu relacin se establece entre los suspiros
y las lgrimas y los elementos de la naturaleza?
Indica las correlaciones que hay en cada verso.
Quin es el ngel fieramente humano? Explica
cul es la conclusin expresada en los tercetos.
Menciona el tema del soneto Suspiros tristes,
lgrimas cansadas.
Qu se describe en el tercer texto?
Haz una relacin de los tipos humanos que
componen la enumeracin de los cuartetos.
Qu tpico desarrolla el primer terceto?
Cul es la imagen final? En qu tipo
de sonetos incluiras este poema?
10
11
9
8
6
5
7
4
3
2
1
Gngora escribi sonetos de diversos temas: amorosos, de influencia petrarquista; morales, satricos y burles-
cos. Tambin escribi los llamados heroicos, destinados a la alabanza de amigos, grandes o reyes.
ACTI VI DADES
16
1
gran ro: traduccin literal de Guadalquivir.
2
doradas: las distingue del Dauro, que llevaba oro.
3
por plumas espadas: se refiere a los cordobeses ilustres:
Sneca, Juan de Mena y el Gran Capitn.
4
Genil y Dauro: se refiere a los dos ros de Granada.
5
plantas: lamos.
6
grandes: ttulo nobiliario.
7
abada: rinoceronte.
8
ttulos: nobles.
9
liberales: prdigos, generosos.
10
ilustri cavaglier: en italiano, ilustres caballeros.
11
llaves doradas: las de los del servicio de la cmara real.
12
hbito: se refiere a los caballeros de las rdenes militares.
13
damas tocas: infieles y viudas alegres.
14
carrozas: las mejores solan tener cuatro caballos.
15
catarriberas: abogados, alcaldes y corregidores.
16
Brtulos y Abades: jurisconsultos del derecho civil y cannico.
17
casas malicia: las construidas para librarse de la obligacin
de alojar a los servidores del rey.
18
pechos: voluntades.
19
perejil: excrementos, inmundicias que se arrojaban a la calle.
A Crdoba
Oh excelso muro, oh torres coronadas
de honor, de majestad, de gallarda!
Oh gran ro
1
, gran rey de Andaluca,
de arenas nobles, ya que no doradas
2
!
Oh frtil llano, oh sierras levantadas, 5
que privilegia el cielo y dora el da!
Oh siempre glorosa patria ma,
tanto por plumas cuanto por espadas
3
!
Si entre aquellas rinas y despojos
que enriquece Genil y Dauro
4
baa 10
tu memoria no fue alimento mo,
nunca merezcan mis ausentes ojos
ver tu muro, tus torres y tu ro,
tu llano y sierra, oh patria, oh flor de Espaa!
Suspiros tristes, lgrimas cansadas
Suspiros tristes, lgrimas cansadas,
que lanza el corazn, los ojos llueven,
los troncos baan y las ramas mueven
de estas plantas
5
a Alcides consagradas;
mas del viento las fuerzas conjuradas 5
los suspiros desatan y remueven,
y los troncos las lgrimas se beben,
mal ellos y peor ellas derramadas.
Hasta en mi tierno rostro aquel tributo
que dan mis ojos, invisible mano 10
de sombra o de aire me le deja enjuto,
porque aquel ngel fieramente humano
no crea mi dolor, y as es mi fruto
llorar sin premio y suspirar en vano.
Grandes ms que elefantes y que abadas
Grandes
6
, ms que elefantes y que abadas
7
,
ttulos
8
liberales
9
como rocas,
gentiles hombres, solo de sus bocas,
ilustri cavaglier
10
, llaves doradas
11
;
hbitos
12
, capas digo remendadas, 5
damas de haz y envs, viudas sin tocas
13
,
carrozas
14
de ocho bestias, y aun son pocas
con las que tiran y que son tiradas;
catarriberas
15
, nimas en pena,
con Brtulos y Abades
16
la milicia, 10
y los derechos con espada y daga;
casas
17
y pechos
18
, todo a la malicia;
lodos con perejil
19
y yerbabuena:
esto es la Corte. Buena pro les haga!
0B1LLLP(2008).16+s 3/6/08 15:55 Pgina 276
REFUERZO
3. Sonetos de Francisco de Quevedo
Se ha hecho una clasificacin temtica de la poesa de Quevedo, que incluye los poemas amo-
rosos, los metafsicos y morales, los religiosos y los satricos y burlescos. Aqu se reproducen
tres sonetos que tratan los temas ms importantes.
Cmo define la vida el yo potico en el primer
cuarteto de Vivir es caminar breve jornada?
Qu tpico literario aparece?
Segn el segundo cuarteto, qu desea,
sin embargo, el ser humano?
Qu idea desarrollan los tercetos?
Menciona los temas del texto y reconoce
los principales recursos expresivos.
A qu herida se refiere la voz potica
en el segundo texto? Qu efectos tiene
y en qu convierte su vida?
Cmo se comporta el yo potico?
Cules son sus sentimientos?
Qu motivos se desarrollan en el segundo
soneto? Reconoce las metforas referidas
al amor.
Cul es el consejo que le da la voz potica
a Apolo en el tercer texto?
Qu ejemplos de historias mitolgicas utiliza
como argumentos? Cul es la conclusin?
Qu visin del amor aparece en el tercer
poema?
Analiza los principales recursos expresivos
utilizados en A Apolo siguiendo a Dafne.
Adscribe los tres sonetos en la clasificacin
temtica de la poesa de Francisco Quevedo.
12
11
10
9
8
7
6
5
4
3
2
1
277 16. La poesa y la prosa barrocas
M
A
T
E
R
I
A
L

F
O
T
O
C
O
P
I
A
B
L
E

/


O
x
f
o
r
d

U
n
i
v
e
r
s
i
t
y

P
r
e
s
s

E
s
p
a

a
,
S
.
A
.
ACTI VI DADES
16
Vivir es caminar breve jornada
Vivir es caminar breve jornada
y muerte viva es, Lico, nuestra vida,
ayer al frgil cuerpo amanecida,
cada instante en el cuerpo sepultada.
Nada, que siendo, es poco y ser nada 5
en poco tiempo, que ambiciosa olvida,
pues de la vanidad mal persuadida
anhela duracin tierra animada.
Llevada de engaoso pensamiento
y de esperanza burladora y ciega, 10
tropezar en el mismo monumento,
como el que divertido el mar navega
y sin moverse vuela con el viento,
y antes que piense en acercarse llega.
A Apolo siguiendo a Dafne
Bermejazo
3
platero de las cumbres,
a cuya luz se espulga
4
la canalla,
la ninfa
5
Dafne, que se afufa
6
y calla,
si la quieres gozar, paga y no alumbres.
Si quieres ahorrar de pesadumbres, 5
ojo del cielo, trata de compralla:
en confites gast Marte
7
la malla
8
,
y la espada en pasteles y en azumbres
9
.
Volvise en bolsa
10
Jpiter severo;
levantse las faldas la doncella 10
por recogerle en lluvia de dinero.
Astucia fue de alguna duea
11
estrella
12
,
que de estrella sin duea no lo infiero:
Febo
13
, pues eres sol, srvete de ella.
En los claustros de lalma la herida
En los claustros de lalma la herida
yace callada, mas consume hambrienta
la vida, que en mis venas alimenta
llama por las medulas extendida.
Bebe el ardor hidrpica
1
mi vida 5
que ya, ceniza amante y macilenta,
cadver del incendio hermoso ostenta
su luz en humo y noche fallecida.
La gente esquivo y me es horror el da;
dilato en largas noches negro llanto, 10
que a sordo mar mi ardiente pena enva.
A los suspiros di la voz de el canto;
la confusin inunda lalma ma;
mi corazn es reino del espanto
2
.
1
hidrpica: sedienta.
2
reino del espanto: el Infierno.
3
bermejazo: rojizo, pelirrojo.
4
espulgar: limpiar de pulgas.
5
ninfa: diosa, y en germana (jerga de ladrones
y malhechores), prostituta.
6
afufar: huir, escapar. Voz de germana.
7
Marte: dios romano de la guerra. Quevedo da una
versin burlesca de los amores de Marte y Venus.
8
malla: parte de la armadura parecida a una red.
9
azumbre: medida de capacidad.
10
bolsa: odre, y tambin, bolsa para el dinero. Jpiter
se convirti en lluvia de oro para seducir a Dnae.
11
duea: alcahueta; mujer que no es doncella.
12
estrella: embustera y astro.
13
Febo: nombre romano del dios Apolo.
0B1LLLP(2008).16+s 3/6/08 15:55 Pgina 277
AMPLIACIN
4. Lope y otros poetas barrocos
Adems de la obra potica de Lope de Vega, destacan en este perodo la de otros autores como Fran-
cisco de Rioja (Sevilla, h. 1583-1639) y Gabriel Bocngel (Madrid, 1603-1658), en cuyas composiciones
aparecen tratados los temas ms importantes de la lrica del barroco.
Qu concepcin del amor desarrolla el soneto
de Lope?
Qu recursos expresivos utiliza para ello?
Segn el segundo poema, a qu est unida
la vida desde su nacimiento?
Cul es la concepcin del tiempo y de la vida
que se expone en los tercetos?
Qu descripcin de la rosa aparece
en el poema Pura, encendida rosa,
de Francisco de Rioja?
De qu tema importante del barroco
se convirti en un smbolo la rosa?
Relaciona los temas de estos tres poemas
con los de la lrica barroca.
6
5
3
7 4
2
1
278 16. La poesa y la prosa barrocas
M
A
T
E
R
I
A
L

F
O
T
O
C
O
P
I
A
B
L
E

/


O
x
f
o
r
d

U
n
i
v
e
r
s
i
t
y

P
r
e
s
s

E
s
p
a

a
,
S
.
A
.
ACTI VI DADES
16
Definicin de amor
Desmayarse, atreverse, estar furioso,
spero, tierno, liberal, esquivo,
alentado, mortal, difunto, vivo,
leal, traidor, cobarde y animoso;
no hallar fuera del bien centro y reposo, 5
mostrarse alegre, triste, humilde, altivo,
enojado, valiente, fugitivo,
satisfecho, ofendido, receloso;
huir el rostro al claro desengao,
beber veneno por licor save, 10
olvidar el provecho, amar el dao;
creer que un cielo en un infierno cabe,
dar la vida y el alma a un desengao:
esto es amor: quien lo prob lo sabe.
Lope DE VEGA
Poesa completa, Castalia
Pura, encendida rosa
Huye del sol el sol, y se deshace
Huye del sol el sol, y se deshace
la vida a manos de la propia vida;
del tiempo que, a sus partos homicida,
en mies de siglos las edades pace,
nace la vida, y con la vida nace 5
del cadver la fbrica temida.
Qu teme, pues, el hombre en la partida,
si vivo estriba en lo que muerto yace?
Lo que pas ya falta; lo futuro
an no se vive; lo que est presente 10
no est, porque es su esencia el movimiento.
Lo que se ignora es solo lo seguro;
este mundo, repblica de viento
que tiene por monarca un accidente.
Gabriel BOCNGEL
Poesa lrica del Siglo de Oro, Ctedra
1
de la deidad: Venus, que naci de la espuma del mar, se pinch con una espina y, al teir las rosas
blancas con su sangre se volvieron rojas.
Pura, encendida rosa,
mula de la llama
que sale con el da,
cmo naces tan llena de alegra
si sabes que la edad que te da el cielo 5
es apenas un breve y veloz vuelo?
Y ni valdrn las puntas de tu rama,
ni prpura hermosa
a detener un punto
la ejecucin del hado presurosa. 10
El mismo cerco alado
que estoy viendo riente,
ya temo, amortiguado,
presto despojo de la llama ardiente.
Para las hojas de tu crespo seno 15
te dio Amor de sus alas blandas plumas,
y oro de su cabello dio a tu frente.
Oh fiel imagen suya peregrina!
Bate en su color sangre divina
de la deidad
1
que dieron las espumas. 20
Y esto, purprea flor, esto no pudo
hacer menos violento el rayo agudo?
Rbate en una hora,
rbate licencioso su ardimiento
el color y el aliento: 25
tiendes an no las alas abrasadas
y ya vuelan al suelo desmayadas:
tan cerca, tan unida
est al morir tu vida,
que dudo si en sus lgrimas la aurora 30
mustia, tu nacimiento o muerte llora.
Francisco DE RIOJA
Poesa lrica del Siglo de Oro
Ctedra
0B1LLLP(2008).16+s 3/6/08 15:55 Pgina 278
AMPLIACIN
5. La fuerza del amor,
de Mara de Zayas
279 16. La poesa y la prosa barrocas
M
A
T
E
R
I
A
L

F
O
T
O
C
O
P
I
A
B
L
E

/


O
x
f
o
r
d

U
n
i
v
e
r
s
i
t
y

P
r
e
s
s

E
s
p
a

a
,
S
.
A
.
Mara de Zayas y Sotomayor es una de las principales representantes de la evolucin
de la novela corta, dentro de la variedad de la novela cortesana, a la que aport la ori-
ginalidad de un punto de vista femenino. En 1637 public sus Novelas amorosas y
ejemplares, y en 1647, la Parte segunda del sarao y entretenimiento honesto, conocida
tambin con el nombre de Desengaos.
Las Novelas constan de un marco narrativo (las reuniones en casa de Lisis) en el que,
a su vez, se desarrolla una intriga amorosa, y de una serie de relatos narrados por los
personajes del marco. Los temas principales de estas historias son el amor apasionado
y firme, la deshonra y la venganza que esta desencadena, los celos, el sometimiento
de la mujer y la violencia que se ejerce sobre ella, as como la bsqueda de indepen-
dencia y libertad por parte de las mujeres. La autora rechaza la superioridad del hom-
bre y denuncia la marginacin y subordinacin de la mujer, a quien se le vedaba la
educacin y la cultura. Cuestiona, adems, el sistema patriarcal, que impona el matri-
monio como nico destino de la mujer en general y como solucin en caso de violacin.
La fuerza del amor es una de las narraciones de las Novelas amorosas y ejemplares.
En este relato, Laura, la protagonista, se casa con Diego, un galn muy enamorado al
principio, pero que, poco despus de la boda, reanuda sus relaciones con una novia
anterior y empieza a despreciar a su esposa, llegando hasta el maltrato fsico. En el
siguiente fragmento la protagonista habla consigo misma.
A quin increpa Laura al principio
del texto?
Explica qu reproche le hace la joven
a Diego, su marido, en el segundo prrafo
y, por extensin, a todos los hombres.
A quines se dirige Laura a continuacin?
Explica de qu los acusa.
Indica de qu recurso expresivo se vale la
autora para formular los reproches de Laura.
Distingue en el fragmento las partes
que corresponden al discurso universal
y al discurso valorativo.
Explica la denuncia de la situacin de la mujer
que se expresa al final del texto.
6
5
4
3
2
1
ACTI VI DADES
16
La fuerza del amor
Desdichada de ti, Laura, y cmo fueras ms venturosa si
como le cost tu nacimiento la vida a tu madre, fuera tambin la
tuya sacrificio de la muerte! Oh amor, enemigo mortal de las
gentes! Y qu de males han venido por ti al mundo, y ms a
las mujeres que como en todo somos las ms perdidosas y las
ms fciles de engaar, parece que solo contra ellas tienes
el poder, o por mejor decir, el enojo. No s para qu el cielo me
cre hermosa, noble y rica, si todo haba de tener tan poco valor
contra la desdicha, sin que tantos dotes de naturaleza y fortuna
me quitasen la mala estrella en que nac. O, ya que lo soy, para
qu me guarda la vida?, pues tenerla un desdichado ms es
agravio que ventura. A quin contar mis penas que me las
remedie? Quin oir mis quejas que se enternezca? Y quin
ver mis lgrimas que me las enjugue? Nadie, por cierto, pues mi
padre y hermanos, por no orlas me han desamparado, y hasta el
cielo, consuelo de los afligidos, se hace sordo por no drmele.
Ay don Diego, y quin lo pensara! Mas s debiera pensar, si
mirara que eres hombre, cuyos engaos quitan el poder a los
mismos demonios y hacen ellos lo que los ministros de malda-
des dejan de hacer. Dnde se hallar un hombre verdadero?
En cul dura la voluntad un da, y ms si se ven queridos?, que
parece que al paso que conocen el amor, crece su libertad
y aborrecimiento. Malhaya la mujer que en ellos cree, pues al
cabo hallar el pago de su amor, como yo le hallo! Quin es la
necia que desea casarse, viendo tantos y tan lastimosos ejem-
plos?, pues la que ms piensa que acierta, ms yerra.
Cmo es mi nimo tan poco, mi valor tan afeminado y mi
cobarda tanta que no quito la vida, no solo a la enemiga de
mi sosiego, sino al ingrato que me trata con tanto rigor? Mas,
ay, que tengo amor! Y en lo uno temo perderle, y en lo otro eno-
jarle. Por qu, vanos legisladores del mundo, atis nuestras
manos para las venganzas, imposibilitando nuestras fuerzas
con vuestras falsas opiniones, pues nos negis letras y armas?
El alma no es la misma que la de los hombres? Pues si ella es la
que da valor al cuerpo, quin obliga a los nuestros a tanta
cobarda? Yo os aseguro que si entendierais que tambin haba
en nosotros valor y fortaleza, no os burlarais como os burlis.
Y as, por tenernos sujetas desde que nacemos, vais enflaque-
ciendo nuestras fuerzas con los temores de la honra, y el enten-
dimiento con el recato de la vergenza, dndonos por espadas,
ruecas, y por libros, almohadillas. Mas triste de m! De qu me
sirven estos pensamientos, pues ya no sirven para remediar
cosas tan sin remedio?
Mara DE ZAYAS Y SOTOMAYOR
Novelas amorosas y ejemplares
Ctedra
0B1LLLP(2008).16+s 3/6/08 15:55 Pgina 279
REFUERZO
6. Guzmn de Alfarache,
de Mateo Alemn
El clebre fragmento que reproducimos a continuacin corresponde al principio del libro,
cuando Guzmanillo, solo y todava muy inocente, deja su casa y sale a recorrer el mundo.
Ha andado mucho, est cansado y hambriento, llega a una venta y pide de comer; el carcter
repulsivo del episodio ilustra claramente la existencia del engao, del que es vctima por pri-
mera vez.
De lo que le sucedi a Guzmanillo en una venta
Djele [a la ventera] que iba a la corte, que me diese de comer. Hzome sentar en un ban-
quillo cojo y encima de un poyo me puso un barredero de horno, con un salero hecho de un
suelo de cntaro, un tiesto de gallinas lleno de agua y una media hogaza ms negra que los
manteles. Luego me sac en un plato una tortilla de huevos, que pudiera llamarse mejor
emplasto de huevos.
Ellos, el pan, jarro, agua, salero, sal, manteles y la huspeda, todo era de lo mismo. Halleme
bozal, el estmago apurado, las tripas de posta, que se daban unas con otras de vacas. Com,
como el puerco la bellota, todo a hecho; aunque verdaderamente senta crujir entre los dientes
los tiernecitos huesos de los sin ventura pollos, que era como hacerme cosquillas en las encas.
Bien es verdad que se me hizo novedad y aun en el gusto, que no era como el de los otros hue-
vos que sola comer en casa de mi madre; mas dej pasar aquel pensamiento con la hambre y
el cansancio, parecindome que la distancia de la tierra lo causaba y que no eran todos de un
sabor ni calidad. Yo estaba de manera que aquello tuve por buena suerte.
Tan propio es al hambriento no reparar en salsas, como al necesitado salir a cualquier
partido. Era poco, paselo presto con las buenas ganas. En el pan me detuve algo ms. Comilo
a pausas, porque siendo muy malo, fue forzoso llevarlo de espacio, dando lugar unos boca-
dos a otros, que bajasen al estmago por su orden. Comencelo por las cortezas y acabelo en
el migajn, que estaba hecho engrudo; mas tal cual, no le perdon letra ni les hice a las hor-
migas migaja de cortesa ms que si fuera poco y bueno. As acontece si se juntan buenos
comedores en un plato de fruta, que picando primero en la ms madura, se comen despus la
verde, sin dejar memoria de lo que all estuvo. []
As prosegu mi camino, y no con poco cuidado de saber qu pudiera ser aquel taerme
castaetas los huevos en la boca. Fui dando y tomando en esta imaginacin, que, cuanto
ms la segua, ms gnero de desventuras me representaba y el estmago se me alteraba;
porque nunca sospech cosa menos que asquerosa, vindolos tan mal guisados, el aceite
negro, que pareca de suelos de candiles, la sartn negra y la ventera legaosa.
Entre unas y otras imaginaciones, encontr con la verdad y, teniendo andada otra legua,
con solo aquel pensamiento, fue imposible resistirme. Porque, como a mujer preada, me
iban y venan eruptaciones del estmago a la boca, hasta que de todo punto no me qued
cosa en el cuerpo. Y aun el da de hoy me parece que siento los pobreticos pollos pindome
ac dentro. As estaba sentado en la falda del vallado de unas vias, considerando mis infor-
tunios, harto arrepentido de mi mal considerada partida, que siempre se despean los mozos
tras el gusto presente, sin respetar ni mirar el dao venidero.
Mateo ALEMN
Guzmn de Alfarache, Planeta
Qu condiciones presenta la mesa
que le sirven en la venta al protagonista?
Qu plato le sirve la ventera? Explica
qu come en primer lugar y qu piensa
el protagonista del estado del ingrediente
principal.
El protagonista sigue su camino:
Qu efectos le causan los alimentos?
Qu verdad descubre?
Explica la reflexin final del narrador.
Seala en este fragmento enunciados de
discurso valorativo y de discurso universal.
5
4
3
2
1
280 16. La poesa y la prosa barrocas
M
A
T
E
R
I
A
L

F
O
T
O
C
O
P
I
A
B
L
E

/


O
x
f
o
r
d

U
n
i
v
e
r
s
i
t
y

P
r
e
s
s

E
s
p
a

a
,
S
.
A
.
ACTI VI DADES
16
0B1LLLP(2008).16+s 3/6/08 15:55 Pgina 280
REFUERZO
7. Una descripcin mordaz
Divide el texto en partes segn los aspectos
que abarca la descripcin: partes del cuerpo,
vestimenta, habitacin y hbitos.
Que rasgo principal se destaca de la conducta
del licenciado Cabra?
Qu personaje de una novela picaresca
anterior tiene caractersticas similares
a las del licenciado?
Analiza los recursos expresivos y determina
cules predominan.
Realiza una parfrasis del texto.
Relaciona el modo en que se describe
al personaje con los rasgos temticos
y formales de la obra.
Reconoce algunas de las caractersticas
propias de un texto descriptivo.
7
6
5
4
3
2
1
281 16. La poesa y la prosa barrocas
M
A
T
E
R
I
A
L

F
O
T
O
C
O
P
I
A
B
L
E

/


O
x
f
o
r
d

U
n
i
v
e
r
s
i
t
y

P
r
e
s
s

E
s
p
a

a
,
S
.
A
.
ACTI VI DADES
16
El siguiente texto corresponde a la famosa descripcin del licenciado Cabra, en la que Queve-
do muestra su ingenio y su capacidad para la agudeza. Pablos, el protagonista, marcha a Segovia
como criado de don Diego Coronel, a quien su padre ha puesto bajo la tutela del licenciado.
El licenciado Cabra
Entramos, primero domingo despus de Cuaresma, en poder de la hambre viva,
porque tal laceria
1
no admite encarecimiento. l era un clrigo cerbatana, largo
2
solo
en el talle; una cabeza pequea; los ojos, avecindados en el cogote, que pareca que
miraba por cuvanos
3
, tan hundidos y escuros, que era buen sitio el suyo para tiendas
de mercaderes; la nariz, de cuerpo de santo, comido el pico, entre Roma y Francia
4
,
porque se le haba comido de unas bas
5
de resfriado, que aun no fueron de vicio por-
que cuestan dinero; las barbas, descoloridas de miedo de la boca vecina, que, de pura
hambre, pareca que amenazaba a comrselas; los dientes, le faltaban no s cuntos,
y pienso que por holgazanes y vagamundos se los haban desterrado; el gaznate,
largo como de avestruz, con una nuez tan salida, que pareca se iba a buscar de comer
forzada de la necesidad; los brazos, secos; las manos, como un manojo de sarmientos
cada una; mirado de medio abajo, pareca tenedor o comps, con dos piernas largas
y flacas; su andar, muy espacioso: si se descompona algo, le sonaban los gesos como
tablillas de San Lzaro
6
; la habla, tica
7
; la barba, grande, que nunca se la cortaba por
no gastar, y l deca que era tanto el asco que le daba ver la mano del barbero por su
cara, que antes se dejara matar que tal permitiese: cortbale los cabellos un mucha-
cho de nosotros. Traa un bonete los das de sol, ratonado con mil gateras, y guarni-
ciones de grasa; era de cosa que fue pao, con los fondos en caspa. La sotana, segn
decan algunos, era milagrosa, porque no se saba de qu color era. Unos, vindola tan
sin pelo, la tenan por de cuero de rana; otros decan que era ilusin: desde cerca pareca
negra, y desde lejos entreazul. Llevbala sin ceidor; no traa cuello ni puos. Pareca,
con esto y los cabellos largos y la sotana y el bonetn, teatino lanudo
8
. Cada zapato
poda ser tumba de un filisteo
9
. Pues su aposento? Aun araas no haba en l. Con-
juraba los ratones de miedo que no le royesen algunos mendrugos que guardaba. La
cama tena en el suelo, y dorma siempre de un lado por no gastar las sbanas. Al fin,
l era archipobre y protomiseria.
Francisco DE QUEVEDO
La vida del Buscn, Crtica
1
laceria: miseria.
2
largo: alto y, tambin, generoso (diloga).
3
cuvanos: cestos de mimbre usados en la
vendimia.
4
entre Roma y Francia: alude a que la nariz era roma
(pequea y poco puntiaguda) y deforme, como atacada
por la sfilis (llamada el mal francs).
5
bas: bubas, tumores blandos.
6
tablillas de san Lzaro: las utilizaban las personas
enfermas de lepra para avisar de su presencia y pedir
limosna.
7
tica: un tipo de fiebre, que afectaba al habla.
8
teatino lanudo: religioso de una orden que sola
acompaar a los condenados a muerte.
9
filisteo: hombre de gran estatura y corpulencia.
0B1LLLP(2008).16+s 3/6/08 15:55 Pgina 281
AMPLIACIN
8. Los Sueos, de Quevedo
Menciona todos los tipos humanos y sociales
que se nombran en el primer prrafo.
Cmo se ubica en el Infierno a los que van
llegando? Explica alguna de las distribuciones
que se realizan.
Menciona cul es la finalidad de Quevedo
al determinar la agrupacin de los condenados.
Reconoce los recursos expresivos que le sirven
para tal objetivo.
Seala los tipos de enamorados. Qu significa
la frase todos lo son de s mismos?
Por qu hay pocos enamorados de las mujeres
en el Infierno?
Relaciona el contenido del texto con los temas
de los Sueos.
En qu otras obras de Quevedo aparecen
temas semejantes?
7
6
5
4
3
2
1
282 16. La poesa y la prosa barrocas
M
A
T
E
R
I
A
L

F
O
T
O
C
O
P
I
A
B
L
E

/


O
x
f
o
r
d

U
n
i
v
e
r
s
i
t
y

P
r
e
s
s

E
s
p
a

a
,
S
.
A
.
ACTI VI DADES
16
En El alguacil endemoniado el narrador habla con un demonio que se ha apoderado del
alma de un alguacil, personaje que pertenece a un sector social habitualmente criticado
por Quevedo; el diablo advierte que estos golpes que le doy y lo que le aporreo, no es sino
que yo y l reimos ac sobre quin ha de estar en mejor lugar y andamos a ms diablo es
l. El narrador le pregunta sobre quines estn en el Infierno y sobre sus penas.
Del sitio que a cada cual le corresponde en el Infierno
Y en el Infierno estn todos aposentados as.
Que un artillero que baj all el otro da, que-
riendo que le pusiesen entre la gente de guerra,
como al preguntarle del oficio que haba tenido
dijese que hacer tiros
1
en el mundo, fue remitido
al cuartel de los escribanos, pues son los que
hacen tiros en el mundo. Un sastre, porque dijo
que haba vivido de cortar de vestir
2
, fue apo-
sentado con los maldicientes. Un ciego, que
quiso encajarse con los poetas, fue llevado a los
enamorados, por serlo todos. Los que venan por
el camino de los locos, ponemos con los astr-
logos, y a los por mente catos, con los alquimis-
tas. Uno vino por unas muertes, y est con los
mdicos. Los mercaderes que se condenan por vender, estn con Judas. Los malos
ministros, por lo que han tomado, alojan con el mal ladrn. Los necios estn con
los verdugos. Y un aguador, que dijo haba vendido agua fra, fue llevado con los
taberneros. Lleg un moha trero
3
tres das ha, y dijo que l se condenaba por
haber vendido gato por liebre, y pusmoslo de pies con los venteros, que dan lo
mismo. Al fin, el Infierno est repar tido en estas partes.
Ote decir antes de los enamorados, y por ser cosa que a m me toca,
gustara saber si hay muchos.
Mancha es la de los enamorados respondi que lo toma todo, porque
todos lo son de s mismos: algunos, de sus dineros; otros de sus palabras; otros, de
sus obras, y algunos, de las mujeres. Y destos postreros hay menos que de todos
en el Infierno, porque las mujeres son tales, que, con ruindades, con malos tratos
y peores correspondencias les dan ocasiones de arrepentimiento cada da a los
hombres. [] Algunos hay que en celos y esperanzas amortajados y en deseos,
se van por la posta
4
al Infierno, sin saber cmo ni cundo ni de qu manera.
Francisco DE QUEVEDO
Sueos, Espasa Calpe
1
hacer tiro: engaar.
2
cortar de vestir: murmurar, hablar mal de alguien.
3
mohatrero: persona que engaa, que comete fraudes.
4
por la posta: de prisa, velozmente.
0B1LLLP(2008).16+s 3/6/08 15:55 Pgina 282
AMPLIACIN
9. Orculo manual y arte
de prudencia, de Baltasar Gracin
283 16. La poesa y la prosa barrocas
M
A
T
E
R
I
A
L

F
O
T
O
C
O
P
I
A
B
L
E

/


O
x
f
o
r
d

U
n
i
v
e
r
s
i
t
y

P
r
e
s
s

E
s
p
a

a
,
S
.
A
.
ACTI VI DADES
16
Este libro, compuesto por trescientos aforismos, ofrece un arte de prudencia,
unas normas de vida constituidas por afirmaciones caracterizadas por su laconismo.
A continuacin reproducimos algunos de ellos.
El primer aforismo contiene un enunciado
muy difundido, cul es? A qu situacin
lo aplica el texto? Pon ejemplos de otros casos
en que tambin podra utilizarse esta
expresin.
Establece relaciones temticas entre
los aforismos 105 y 299 y localiza en ambos
una expresin casi idntica.
Cules son los argumentos que fundamentan
los consejos de No vivir aprisa y Saber estimar?
Por qu conviene no excusarse de lo que uno
no ha sido acusado? Qu consecuencias trae?
Redacta un texto sobre el vivir deprisa
en la sociedad actual (cmo se manifiesta
y en quines, cules son sus motivos y
justificaciones, qu juicios te merecen, etc.).
5
4
3
2
1
1
negocio: asunto, ocupacin.
2
postilln: mozo que iba a caballo, delante de las postas
para guiar y ensear el camino.
3
hacer del ojo: avisar, llamar la atencin.
4
es grande la baja de la segunda vez: la segunda vez
que algo se aprecia baja en estimacin.
105. No cansar. Suele ser pesado el hombre de un negocio
1
y el de un verbo. La brevedad es lisonjera, y ms negociante:
gana por lo corts lo que pierde por lo corto. Lo bueno, si breve, dos veces bueno; y aun lo malo, si poco, no tan malo. Ms
obran quintas esencias que frragos; y es verdad comn que hombre largo raras veces es entendido, no tanto en lo material
de la disposicin, cuanto en lo formal del discurso. Hay hombres que sirven ms de embarazo que de adorno del universo,
alhajas perdidas que todos las desvan. Excuse el discreto el embarazar, y mucho menos a grandes personajes, que viven
muy ocupados; y sera peor desazonar uno de ellos que todo lo restante del mundo. Lo bien dicho se dice presto.
299. Dejar con hambre.
Hase de dejar en los la-
bios aun con el nctar. Es
el deseo medida de la esti-
macin. Hasta la material
sed es treta de buen gusto
picarla, pero no acabarla:
lo bueno, si poco, dos ve-
ces bueno. Es grande la
baja de la segunda vez
4
.
Hartazgos de agrado son
peligrosos, que ocasionan
desprecios a la ms eter-
na eminencia. nica regla
de agradar, coger el ape -
tito picado con el hambre
con que se qued. Si se ha
de irritar, sea antes por im-
paciencia del deseo que
por enfado de la fruicin;
gstase al doble de la feli-
cidad penada.
Baltasar GRACIN
Orculo manual y arte de prudencia
Planeta
246. Nunca dar satisfaccin a quien no la peda. Y aunque se pida, es especie de
delito si es sobrada. El excusarse antes de ocasin es culparse, y el sangrarse en salud
es hacer del ojo
3
al mal y a la malicia. La excusa anticipada despierta al recelo que
dorma. Ni se ha de dar el cuerdo por entendido de la sospecha ajena, que es salir a buscar
el agravio; entonces le ha de procurar desmentir con la entereza de su proceder.
174. No vivir aprisa. El saber repartir las cosas es saberlas gozar. A muchos les sobra la
vida y se les acaba la felicidad; malogran los contentos, que no los gozan, y querran
despus volver atrs cuando se hallan tan adelante; postillones
2
del vivir, que, a ms del
comn correr del tiempo, aaden ellos su atropellamiento genial; querran devorar en un
da lo que apenas podran digerir en toda la vida; viven adelantados en las felicidades,
cmense los aos por venir y, como van con tanta prisa, acaban presto con todo. Aun en
el querer saber ha de haber modo, para no saber las cosas mal sabidas. Son ms los das
que las dichas. En el gozar, a espacio; en el obrar, aprisa. Las hazaas, bien estn hechas;
los contentos, mal acabados.
195. Saber estimar. Ninguno hay que no pueda ser maestro de otro en algo; ni hay quien
no exceda al que excede. Saber disfrutar a cada uno es til saber. El sabio estima a todos,
porque reconoce lo bueno en cada uno, y sabe lo que cuestan las cosas de hacerse bien.
El necio desprecia a todos por ignorancia de lo bueno y por eleccin de lo peor.
0B1LLLP(2008).16+s 3/6/08 15:55 Pgina 283
Evaluacin 16
275 16. La poesa y la prosa barrocas
1. Reconoce el gnero al que pertenece el texto y justifica
los motivos de esa adscripcin.
2. Explica el contenido del poema y determina el o los
temas que desarrolla.
3. Qu motivo propio de la poesa de Quevedo aparece
en este texto? En qu tipos de obras lo trata?
4. Realiza el anlisis mtrico de la composicin. Qu otras
formas mtricas se cultivaron en el barroco?
5. Analiza los recursos expresivos ms destacados del texto.
6. Enuncia y explica otros tipos de poesa que haya cultivado
Quevedo y menciona las caractersticas de su estilo.
7. Realiza una sntesis de la obra potica de Gngora y
de las caractersticas estilsticas ms importantes.
8. Menciona las obras principales de Lope de Vega, los
temas tratados en general y la particularidad de su estilo.
9. Relaciona los temas de la poesa y la prosa barrocas con
el contexto histrico, social y literario de su produccin.
10. Redacta un texto en que reflexiones, en general, sobre
la brevedad de la vida y su final inevitable (por ejemplo,
si has pensado en ello alguna vez o es una realidad
percibida como muy lejana, cmo afrontarla, si compartes
la actitud de los poetas barrocos ).
Alegora de una poca fugaz, por Antoni da Pereda.
1
secreta: ignorada, escondida.
2
despreciada: la juventud (la edad lozana).
3
postrer invierno: la vejez.
4
me deben la vida mis engaEos: los engaos le han quitado
la vida.
Huye sin percibirse lento el da
Huye sin percibirse lento el da,
y la hora secreta
1
y recatada
con silencio se acerca, y despreciada
2
lleva tras s la edad lozana ma.
La vida nueva, que en niez arda, 5
la juventud robusta y engaada,
en el postrer invierno
3
sepultada,
yace entre negra sombra y nieve fra.
No sent resbalar mudos los aos;
hoy los lloro pasados y los veo 10
riendo de mis lgrimas y daos.
Mi penitencia deba a mi deseo,
pues me deben la vida mis engaos
4
y espero el mal que paso y no le creo.
Francisco DE QUEVEDO
Poesa completa, Planeta
0B1LLLA.16 7/4/08 17:02 Pgina 275
AMPLIACIN
1. Auto de los Reyes Magos
El Auto o Representacin de los Reyes Magos, perteneciente al ciclo del Ordo Stellae (escenificado
en las fiestas de la Epifana), es el ms antiguo drama sacro en romance que se conserva. Consta
de los monlogos de los Reyes, su encuentro y la decisin de peregrinar juntos, la visita a Hero-
des y el dilogo de este con sus consejeros.
Explica por qu el fragmento del Auto
de los Reyes Magos pertenece a un texto
dramtico.
Resume el contenido del dilogo que
mantienen los personajes. Cul es el conflicto
que presenta? Explcalo brevemente.
Caracteriza los personajes que intervienen
en este pasaje atendiendo especialmente
a sus palabras.
Las acotaciones fueron incorporadas por el
profesor Fernando Lzaro Carreter, en la
edicin que prepar del Auto. Seala a qu
aspectos de los cdigos visuales y auditivos
de la representacin hacen referencia.
Reconoce en las intervenciones de los
personajes que participan en el dilogo,
ejemplos de las funciones del lenguaje
expresiva, apelativa y referencial.
5
4
3
2
1
297 17. El teatro: de los orgenes a la comedia nacional
M
A
T
E
R
I
A
L

F
O
T
O
C
O
P
I
A
B
L
E

/


O
x
f
o
r
d

U
n
i
v
e
r
s
i
t
y

P
r
e
s
s

E
s
p
a

a
,
S
.
A
.
ACTI VI DADES
17
Encuentro de los Magos con Herodes
GASPAR.(Dirigindose a HERODES, que se acaricia la negra
y breve barba con gesto impaciente.)
Gurdete el Creador. Dios te libre de mal.
Te entretendremos poco Un instante no ms.
(HERODES, apresurado, le estimula a continuar. GASPAR
considera prudente repetir el saludo.)
Dios te d larga vida, y te libre de mal.
Vamos en romera, para a un rey adorar,
que ha nacido en la tierra No logrmosle hallar.
HERODES.(Sbitamente interesado.)
Qu decs? Dnde vais? A quin vais a buscar? De
qu tierra vens? Dnde intentis llegar? Decidme
vuestros nombres, no los queris celar.
GASPAR.Yo me llamo Gaspar;
este que ves, Melchor; y aquel es Baltasar.
(GASPAR no interpreta bien el inters de HERODES. Atro -
pellado e ingenuo le comunica la nueva.)
Rey, un rey ha nacido que es seor de la tierra, que
mandar en el mundo con gran paz y sin guerra!
HERODES.Es as en verdad?
GASPAR.S, rey, por caridad.
HERODES.Y cmo lo sabis?
Probado lo tenis?
GASPAR.Rey, verdad te diremos:
probado lo tenemos.
MELCHOR.(Sin poder contenerse, seala hacia el cielo.)
Esto es gran maravilla:
un astro nuevo brilla!
BALTASAR.Seal de que ha nacido
y que en carne ha venido!
(Nueva pausa. La voz de HERODES tiembla levemente,
mientras mira a lo alto.)
HERODES.Cunto hace que le visteis
y que lo percibisteis?
GASPAR.Trece das son ya.
(Volvindose a sus compaeros.)
Creo que ms no har
desde que est advertido
y muy bien percibido.
HERODES.(Con tono que sera amable si no le delatara
el temblor de la voz.) Pues andad y buscad,
y a ese rey adorad,
y por aqu tornad
(Ligera pausa. El REY no mira ya a los MAGOS.)
Si lo encontris, ir,
y yo le adorar.
(Se apaga la luz del espacio central. GASPAR sale, tras
haberse inclinado ante HERODES. Este, solo, se pone en
pie. Ya no tiene por qu disimular su clera.)
Quin vio nunca tal mal?
Sobre un rey, otro tal!
An no estoy yo muerto
ni bajo tierra puesto.
Otro rey sobre m?
Nunca tal cosa vi!
El mundo va hacia atrs:
no s qu me haga ya.
Juro que no lo crea
hasta que no lo vea.
Teatro medieval
Castalia
0B1LLLP(2008).17+s-def 3/6/08 18:48 Pgina 297
AMPLIACIN
2. Representacin del nacimiento
de Nuestro Seor
298 17. El teatro: de los orgenes a la comedia nacional
M
A
T
E
R
I
A
L

F
O
T
O
C
O
P
I
A
B
L
E

/


O
x
f
o
r
d

U
n
i
v
e
r
s
i
t
y

P
r
e
s
s

E
s
p
a

a
,
S
.
A
.
Explica en qu consiste el conflicto
que se plantea en el inicio de este fragmento
de la Representacin de nacimiento
de Nuestro Seor.
Cmo caracteriza a Jos la Virgen Mara
en su primera intervencin? Qu le reprocha
el ngel?
Identifica el tipo de estrofa empleado
en este fragmento dramtico.
Incorpora al texto algunas acotaciones
que se refieran a distintos elementos
de los cdigos que intervienen en una
representacin teatral (escenografa,
gestos, movimientos).
4
3
2
1
ACTI VI DADES
17
El escritor Gmez Manrique compuso hacia 1460 la Representacin del nacimiento de Nuestro
Seor, para su hermana doa Mara Manrique, vicaria en el convento de clarisas de Calabazanos
(Palencia). Para esta composicin dramtica, el poeta se basa en los evangelios cannicos
y emplea formas mtricas de la poesa cancioneril del siglo XV.
La desconfianza de Jos
LO QUE DICE JOS, SOSPECHANDO DE NTRA. SEORA:
Oh viejo desventurado!
Negra dicha fue la ma,
en casarme con Mara
por quien fuese deshonrado.
Yo la veo bien preada; 5
no s de quin ni de cunto.
Dicen que d Espritu Santo,
mas yo d esto no s nada.
LA ORACIN QUE HACE LA GLORIOSA:
Mi solo Dios verdadero,
cuyo ser es inmovible, 10
a quien es todo posible,
fcil y bien facedero
1
!
T que sabes la pureza
de la mi virginidad,
alumbra la ceguedad 15
de Jos y su simpleza.
EL NGEL A JOS:
Oh viejo de muchos das,
en el seso de muy pocos,
el principal de los locos!
T no sabes que Isaas 20
dijo: Virgen parir,
lo cual escribi por esta
doncella gentil, honesta,
cuyo par nunca ser?
LA QUE REPRESENTA A LA GLORIOSA, CUANDO LE DIEREN EL NIO:
Adrote, rey del cielo, 25
verdadero Dios y hombre;
adoro tu santo nombre,
mi salvacin y consuelo.
Adrote, hijo y padre,
a quien sin dolor par, 30
porque quisiste de m
hacer de sierva tu madre.
Bien podr decir aqu
aquel salmo gloroso
que dije, hijo precioso, 35
cuando yo te conceb:
que mi nima
2
engrandece
a ti, mi solo seor,
y en ti, mi salvador,
mi espritu florece. 40
Mas este mi gran placer
en dolor ser tornado,
pues t eres enviado
para muerte padecer
por salvar los pecadores 45
en la cual yo pasar,
no mengundome la fe,
innumerables dolores.
Gmez MANRIQUE
Representacin del nacimiento de Nuestro Seor
en Ronald E. SURTZ, Teatro castellano de la Edad Media
Taurus (actualizacin)
1
facedero: hacedero; que se puede hacer.
2
nima: alma.
0B1LLLP(2008).17+s-def 3/6/08 18:48 Pgina 298
AMPLIACIN
3. Una gloga de Juan del Encina
En el siguiente fragmento corresponde al comienzo de la gloga en la que Mengo y un escu-
dero pugnan por el amor de la pastorcilla Pascuala.
Por qu puede adscribirse este texto
de Juan del Encina al gnero dramtico?
Resume el contenido del fragmento.
Caracteriza los personajes y sus actitudes.
Analiza la mtrica empleada.
Caracteriza el habla de los tres personajes
de esta gloga.
Reconoce en el texto un recurso caracterstico
de la poesa cancioneril; quin lo emplea?
Compara estos pastores con los de las glogas
de Garcilaso de la Vega.
7
6
5
4
3
2
1
299 17. El teatro: de los orgenes a la comedia nacional
M
A
T
E
R
I
A
L

F
O
T
O
C
O
P
I
A
B
L
E

/


O
x
f
o
r
d

U
n
i
v
e
r
s
i
t
y

P
r
e
s
s

E
s
p
a

a
,
S
.
A
.
ACTI VI DADES
17
gloga representada en requesta
1
de unos amores
MINGO.Pascuala, Dios te mantenga.
PASCUALA.Nora buena vengas, Mingo.
Oy, qu es da de domingo,
no ests con tu esposa Menga?
MINGO.No ay quien all me detenga, 5
quel cario que te tengo
me pone un quejo
2
tan luengo
que me acossa que me venga.
PASCUALA.Y no praga
3
a Dios contigo,
y aun con tu esposa Menguilla! 10
Cmo dexas tu esposilla
por venirte ac comigo?
MINGO.Soncas
4
, soncas, no te digo
que eres, zagala, tan bella
que te quiero ms que a ella? 15
Dios lo sabe, qu es testigo.
PASCUALA.Miaf
5
, Mingo, no te creo
que de m ests namorado.
Pues eres ya desposado,
tu querer no lo desseo. 20
MINGO.Ay, Pascuala, que te veo
tan lozana y tan garrida,
que yo te juro a mi vida
que deslumbro si te oteo!
Y porque eres tan hermosa 25
te quiero; mira, vers,
quireme, quireme ms,
pues por ti dejo a mi esposa.
Y toma, toma esta rosa
que para ti la cog, 30
aunque no curas de m
ni por m se te da cosa.
PASCUALA.O, qu chapados olores!
Mingo, Dios te d salud,
y gozes la juventud 35
ms que todos los pastores.
MINGO.Y t dasme mil dolores.
Dame, dame una manija,
o siquiera essa sortija,
que traya por tus amores. 40
PASCUALA.Tirte
6
, tirte all, Minguillo,
no te quellotres
7
de vero!
Hete
8
, viene un escudero;
vea que eres pastorcillo.
Sacude tu caramillo
9
, 45
tu hondijo
10
y tu cayado;
haz que aballas
11
el ganado,
silva, hurria
12
, da gritillo.
ESCUDERO.Pastora, slvete Dios.
PASCUALA.Dios os d, seor, buen da. 50
ESCUDERO.Guarde Dios tu galana.
PASCUALA.Escudero, ass haga a vos.
ESCUDERO.Tienes ms gala que dos
de las de mayor beldad.
PASCUALA.Essos que sois de ciudad 55
perchufis
13
huerte
14
de nos.
ESCUDERO.Desso no tengas temor.
Por mi vida, pastorcica,
que te haga presto rica
PASCUALA.Essas trnicas
15
, seor, 60
all para las de villa.
ESCUDERO.Vete comigo, carilla
16
.
Dexa, dexa esse pastor.
Dxalo, que Dios te vala;
no te pene su penar, 65
que no te sabe tratar
segn requiere tu gala.
MINGO.Estate queda, Pascuala,
no te engae este traidor,
palaciego, burlador, 70
que ha burlado otra zagala.
Juan DEL ENCINA
Teatro y poesa, Taurus
1
requesta: desafo.
2
quejo: dolor, pena.
3
praga: agrade.
4
soncas: en verdad, a fe.
5
miaf: por mi fe, s.
6
tirte: aprtate.
7
quellotres: lamentes.
8
hete: mira.
9
caramillo: flauta de caa.
10
hondijo: honda.
11
aballas: arreas.
12
hurria: arrea.
13
perchufis: burlis.
14
huerte: fuerte, mucho.
15
trnicas: retricas.
16
carilla: compaera.
0B1LLLP(2008).17+s-def 3/6/08 18:48 Pgina 299
AMPLIACIN
4. Un paso de Lope de Rueda
Lope de Rueda fue el principal creador de entremeses, escenas cmicas con autonoma dra-
mtica, conocidas en su poca como pasos. Estas obras breves se escribieron para ser repre-
sentadas en un espectculo ms amplio, como una comedia. De Lope de Rueda se conservan
ms de veinte pasos, que se suelen clasificar en cuatro grupos, segn el tipo de protagonista:
los de simple, los de negra, los de lacayo fanfarrn y los de otras figuras. El siguiente es un frag-
mento del paso Las aceitunas.
Justifica por qu el fragmento de Lope
de Rueda pertenece a un texto dramtico.
Identifica los personajes que intervienen en
este dilogo sealando la relacin que existe
entre ellos.
Por qu acude Aloja? Explica cul es el
conflicto.
Clasifica el paso Las aceitunas en uno
de los grupos sealados en la explicacin
que aparece ms arriba.
Atendiendo a las palabras finales de Aloja,
sobre qu crees que tratar la comedia
que se puede representar despus de este
paso? Razona tu respuesta.
5
4
3
2
1
300 17. El teatro: de los orgenes a la comedia nacional
M
A
T
E
R
I
A
L

F
O
T
O
C
O
P
I
A
B
L
E

/


O
x
f
o
r
d

U
n
i
v
e
r
s
i
t
y

P
r
e
s
s

E
s
p
a

a
,
S
.
A
.
ACTI VI DADES
17
Las aceitunas
ALOJA.Qus esto, vecinos? Por qu me maltra-
tis ans la mochacha?
GUEDA.Ay, seor! Este mal hombre que me quiere
dar las cosas a menos precio y quiere echar a
perder mi casa. Unas aceitunas que son como
nueces!
TORUVIO.Yo juro a los huesos de mi linaje que no
son ni aun como piones.
GUEDA.S son!
TORUVIO.No son!
ALOJA.Ora, seora vecina, haceme tamao placer
que os entris all dentro, que yo lo averiguar
todo.
GUEDA.Averige o pngase todo del quebranto
1
.
ALOJA.Seor vecino, qu son de las aceitunas?
Sacaldas ac fuera, que yo las comprar, aun-
que sean veinte hanegas
2
.
TORUVIO.Que no, seor, que no es desa manera
que vuesa merced se piensa; que no estn las
aceitunas aqu en casa, sino en la heredad
3
.
ALOJA.Pues traeldas aqu, que yos las comprar
todas al precio que justo fuere.
MENCIGELA.A dos reales quiere mi madre que se
venda el celemn
4
.
ALOJA.Cara cosa es esa.
TORUVIO.No le paresce a vuesa merced?
MENCIGELA.Y mi padre a quince dineros.
ALOJA.Tenga yo una muestra dellas.
TORUVIO.Vlame Dios, seor! Vuesa merced no
me quiere entender. Hoy he yo plantado un
renuevo
5
de aceitunas y dice mi mujer que de
aqu a seis o siete aos llevar cuatro o cinco
hanegas de aceituna y quella la cogera y que
yo la acarrease y la mochacha la vendiese.
Y que, a la fuerza de drecho
6
, haba de pedir a
dos reales por cada celemn. Yo, que no, y ella,
que s. Y sobre esto ha sido la cuistin.
ALOJA.Oh, qu graciosa cuistin! Nunca tal se ha
visto. Las aceitunas no estn plantadas y ha
llevado la mochacha tarea sobre ellas.
MENCIGELA.Qu le paresce, seor?
TORUVIO.No llores, rapaza; la mochacha, seor,
es como un oro. Ora andad, hija, y ponedme la
mesa, que yos prometo de hacer un ayuelo de
las primeras aceitunas que se vendieren.
ALOJA.Ora, andad, vecino; entraos all dentro
y ten paz con vuestra mujer.
TORUVIO.Adis, seor.
ALOJA.Ora, por cierto, qu cosas vemos en esta
vida que ponen espanto! Las aceitunas no estn
plantadas, ya las habemos visto reidas. Razn
ser que d fin a mi embajada
7
.
Lope DE RUEDA
Pasos completos, Espasa Calpe
1
averige del quebranto: esta frase se ha
interpretado como averige o djese todo
a favor de la prdida econmica.
2
hanegas: fanegas, medida de capacidad equivalente
a 55 litros y medio.
3
heredad: posesin en el campo.
4
celemn: medida de algo ms de cuatro litros
y medio.
5
renuevo: vstago que echa una planta podada.
6
a la fuerza de drecho: por derecho.
7
embajada: se llamaba as a la presentacin
y embajador al presentador.
0B1LLLP(2008).17+s-def 3/6/08 18:48 Pgina 300
301 17. El teatro: de los orgenes a la comedia nacional
M
A
T
E
R
I
A
L

F
O
T
O
C
O
P
I
A
B
L
E

/


O
x
f
o
r
d

U
n
i
v
e
r
s
i
t
y

P
r
e
s
s

E
s
p
a

a
,
S
.
A
.
ACTI VI DADES
17
REFUERZO
5. Rasgos de la comedia nacional
Repasa los rasgos de la comedia nacional y luego lee atentamente el siguiente fragmento de
La dama duende, de Caldern de la Barca.
LUIS.No hay accin que me suceda
bien, Rodrigo. Si una dama
veo airosa, y conocerla
solicito, me detienen
un necio y una pendencia; 5
que no s cual es peor;
si rio, y mi hermano llega,
es mi enemigo su amigo;
si por disculpa me deja
de una dama, es una dama 10
que mil pesares me cuesta:
de suerte que una tapada
me huye, un necio me atormenta,
un forastero me mata,
y un hermano me le lleva 15
a ser mi husped a casa,
y otra dama me desprecia
De mal anda mi fortuna!
RODRIGO.Qu de todas esas penas
que s la que sientes ms? 20
LUIS.No sabes.
RODRIGO. Que la que llegas
a sentir ms, son los celos
de tu hermana y Beatriz bella?
LUIS.Engaste.
RODRIGO. Pues, cul es?
LUIS.Si tengo de hablar de veras 25
de ti solo me fiara,
lo que ms siento es que sea
mi hermano tan poco atento,
que llevar a casa quiera
un hombre mozo, teniendo, 30
Rodrigo, una hermana en ella,
viuda y moza; y, como sabes,
tan de secreto, que apenas
sabe el sol que vive en casa;
porque Beatriz, por ser deuda, 35
solamente la visita.
RODRIGO.Ya s que su esposo era
administrador de puertos
de mar en unas reales rentas,
y qued debiendo al rey 40
grande cantidad de hacienda;
y ella a la corte se vino
de secreto, donde intenta,
escondida y retirada,
componer mejor sus deudas. 45
Y esto disculpa a tu hermano,
pues, si mejor consideras
que su estado no le da
ni permisin ni licencia
de que nadie la visite, 50
y que, aunque tu husped sea
don Manuel, no ha de saber
que en casa, seor, se encierra
tal mujer, qu inconveniente
hay en admitirle en ella? 55
Y ms habiendo tenido
tal recato y advertencia,
que para su cuarto ha dado
por otra calle la puerta,
y la que sala a la casa, 60
por desmentir la sospecha,
de que el cuidado la haba
cerrado, o porque pudiera
otra vez, fabric en ella
una alacena de vidrios, 65
labrada de tal manera,
que parece que jams
en tal parte ha habido puerta.
LUIS.Ves con lo que me aseguras?
Pues con eso mismo intentas 70
darme muerte, pues ya dices
que no ha puesto por defensa
de su honor ms que unos vidrios,
que al primer golpe se quiebran.
(Vanse y salen DOA NGELA e ISABEL.)
DOA NGELA.Vulveme a dar, Isabel, 75
esas tocas; pena esquiva!,
vuelve a amortajarme viva,
ya que mi suerte crel
lo quiere as.
ISABEL. Toma presto;
porque si tu hermano viene 80
y alguna sospecha tiene,
no la confirme con esto,
de hallarte desta manera,
que hoy en palacio te vio.
Pedro CALDERN DE LA BARCA
La dama duende
Planeta
Resume la situacin planteada en la escena.
Analiza la mtrica empleada y caracterzala.
Explica cmo son los personajes.
Qu rasgos de la comedia nacional estn
presentes en estos versos de Caldern?
Explica en qu consisten aquellos otros rasgos
que no se evidencian en el fragmento.
5
4
3
2
1
El honor familiar
0B1LLLP(2008).17+s-def 3/6/08 18:48 Pgina 301
302 17. El teatro: de los orgenes a la comedia nacional
M
A
T
E
R
I
A
L

F
O
T
O
C
O
P
I
A
B
L
E

/


O
x
f
o
r
d

U
n
i
v
e
r
s
i
t
y

P
r
e
s
s

E
s
p
a

a
,
S
.
A
.
ACTI VI DADES
17
AMPLIACIN
6. Acotaciones de una comedia
cortesana
En 1651, Baccio del Bianco realiz en el parque del Coliseo del Buen Retiro de Madrid
un excepcional montaje de La fiera, el rayo y la piedra, de Caldern de la Barca. Las siguientes
son algunas de las acotaciones incluidas en esta comedia cortesana.
Todas estas acotaciones estn incluidas
en La fiera, el rayo y la piedra, comedia
cortesana de Caldern de la Barca. Seala
aquellas posturas referidas a cuestiones
relacionadas con los personajes (gestos,
movimientos, vestuario).
Cules de estas acotaciones hacen mencin
de elementos relacionados con la utilera de
los personajes? Y con la utilera de escena?
Seala qu indicaciones se ofrecen acerca
de la ambientacin de la obra. Diferencia
entre:
Elementos visuales.
Elementos auditivos.
Despus de analizar estas acotaciones,
relaciona su informacin con lo que sabes
acerca del teatro barroco, especialmente
el teatro cortesano.
4
3
2
1
Texto 2
(Sale IRIFILE, vestida de
pieles, suelto el cabello.)
Texto 3
(Atraviesan varios
pe ces por la marina.)
Texto 4
(Pasan algunas sirenas,
cantando.)
Texto 5
(Atraviesan algunos baje -
lillos por la marina.)
(Empieza a aclarar.)
Texto 6
(Al entrarse por otra
parte huyendo, salen PIG-
MALEN y LEBRN.)
Texto 7
(Mdase el teatro en
el bosque, y en el foro la
gruta de LAS PARCAS.)
Texto 9
(Cantando, en tono muy
triste.)
(Cirrase la gruta.)
Texto 1
(Oscurcese el teatro, que
ser de peascos con el foro de
marina; y mientras se dicen los
primeros versos, se descubre la
perspectiva del mar, y habr
truenos y relmpagos.)
Texto 8
(brese la gruta y vese en lo
ms lejos de ella a LAS TRES PARCAS; la
primera con una rueca, cuyo hilo
va a dar a la tercera, que lo devana,
dejando en medio a la segunda
con unas tijeras en la mano.)
Texto 11
(Pnense las cuatro a
las cuatro puntas del ta-
blado, retranse ANA JARTE y
las otras damas, y mien-
tras cantan, sale IRIFILE,
acechando.)
(Suenan dentro los
martillos de la fragua.)
Texto 10
(Vuela CUPIDO, mdase el teatro
en el de monte, y en el foro, la fra-
gua de VULCANO; y salen por una
parte LISI, CLORI, LAURA e ISABELLA, con
arcos y flechas y varios instru-
mentos en las manos; y por la
otra ANAJARTE, en traje de cazadora,
con venablo, y OTRAS.)
Texto 12
(Descbrese la fragua, y los
C CLO PES cantan al son de los mar-
tillos.)
(Desaparece la fragua y pasa
en una nube ANTEROS, atrave-
sando el teatro con un venablo
en la mano.)
Pedro CALDERN DE LA BARCA
La fiera, el rayo y la piedra
Ibero-Americana de Publicaciones
0B1LLLP(2008).17+s-def 3/6/08 18:48 Pgina 302
Recuerda las caractersticas de los autos sacramentales y luego lee el siguiente fragmento,
que corresponde al final de El gran teatro del mundo, de Pedro Caldern de laBarca.
Explica el contenido de este fragmento.
Qu caracteriza a los personajes?
Analiza la mtrica empleada en estos versos
de El gran teatro del mundo.
Identifica las figuras estilsticas del texto
y explica su funcin.
Qu caractersticas de los autos
sacramentales observas en el fragmento?
5
4
3
2
1
AMPLIACIN
7. Los autos sacramentales
303 17. El teatro: de los orgenes a la comedia nacional
M
A
T
E
R
I
A
L

F
O
T
O
C
O
P
I
A
B
L
E

/


O
x
f
o
r
d

U
n
i
v
e
r
s
i
t
y

P
r
e
s
s

E
s
p
a

a
,
S
.
A
.
ACTI VI DADES
17
RICO.Si el poder y la hermosura
por aquella vanagloria
que tuvieron, con haber
llorado, tanto te asombran,
y el labrador que a gemidos 5
enterneciera una roca
est temblando de ver
la presencia poderosa
de la vista del Autor,
cmo oso mirarla ahora? 10
Mas es preciso llegar,
pues no hay adonde me esconda
de su riguroso juicio.
Autor!
AUTOR. Cmo as me nombras?
Que aunque soy tu Autor, es bien 15
que de decirlo te corras,
pues que ya en mi compaa
no has de estar. De ella te arroja
mi poder. Desciende adonde
te atormente tu ambiciosa 20
condicin eternamente
entre penas y congojas.
RICO.Ay de m! Que envuelto en fuego
caigo arrastrando mi sombra
donde ya que no me vea 25
yo a m mismo, duras rocas
sepultarn mis entraas
en tenebrosas alcobas.
DISCRECIN.Infinita gloria tengo.
HERMOSURA.Tenerla espero dichosa. 30
LABRADOR.Hermosura, por deseos
no me llevars la joya.
RICO.No la espero eternamente.
NIO.No tengo, para m, gloria.
AUTOR.Las cuatro postrimeras 35
son las que presentes notan
vuestros ojos, y porque
destas cuatro se conozca
que se ha de acabar la una,
suba la Hermosura ahora 40
con el Labrador, alegres,
a esta mesa misteriosa,
pues que ya por sus fatigas
merecen grados de gloria.
(Suben los dos.)
HERMOSURA.Qu ventura!
LABRADOR. Qu consuelo! 45
RICO.Qu desdicha!
REY. Qu victoria!
RICO.Qu sentimiento!
DISCRECIN. Qu alivio!
POBRE.Qu dulzura!
RICO. Qu ponzoa!
NIO.Gloria y pena hay, pero yo
no tengo pena ni gloria. 50
AUTOR.Pues el ngel en el cielo,
en el mundo las personas
y en el infierno el demonio
todos a este Pan se postran;
en el infierno, en el cielo 55
y mundo a un tiempo se oigan
dulces voces que le alaben
acordadas y sonoras.
(Tocan chirimas, cantando el Tantum
ergo muchas veces.)
MUNDO.Y pues representaciones
en aquesta vida toda, 60
merezca alcanzar perdn
de las unas y las otras.
Pedro CALDERN DE LA BARCA
El gran teatro del mundo
Ctedra
El autor y sus personajes
0B1LLLP(2008).17+s-def 3/6/08 18:48 Pgina 303
AMPLIACIN
8. El burlador de Sevilla
El burlador de Sevilla, atribuida a Tirso de Molina, constituye una creacin dramtica original
que, aunque utiliza componentes previos, crea uno de los personajes ms importantes de
la literatura: el don Juan, que gener una importante descendencia. En este fragmento, tras
engaar a una mujer noble, don Juan seduce a Tisbea, una pescadora, prometindole casa-
miento; pero en verdad piensa huir una vez consumados sus propsitos.
Resume el contenido de estos versos
de El burlador de Sevilla y relacinalo con lo
que sabes sobre el personaje de don Juan.
Explica la actitud que mantiene el criado
Catalinn ante su amo y la respuesta
que este le da.
Qu mtrica emplea en estos versos Tirso
de Molina? A qu tipo de estructura
estrfica corresponde?
Analiza los principales recursos estilsticos
presentes en este fragmento de El burlador
de Sevilla.
4
3
2
1
304 17. El teatro: de los orgenes a la comedia nacional
M
A
T
E
R
I
A
L

F
O
T
O
C
O
P
I
A
B
L
E

/


O
x
f
o
r
d

U
n
i
v
e
r
s
i
t
y

P
r
e
s
s

E
s
p
a

a
,
S
.
A
.
ACTI VI DADES
17
El engao de don Juan
DON JUAN.Mientras que los pescadores
van de regocijo y fiesta,
t las dos yeguas apresta
1
,
que de sus pies voladores
solo nuestro engao fo. 5
CATALINN.Al fin pretendes gozar a Tisbea?
DON JUAN.Si el burlar
es hbito antiguo mo,
qu me preguntas, sabiendo
mi condicin?
CATALINN. Ya s que eres 10
castigo de las mujeres.
DON JUAN.Por Tisbea estoy muriendo,
que es buena moza.
CATALINN. Buen pago
a su hospedaje deseas.
DON JUAN.Necio, lo mismo hizo Eneas
2
15
con la reina de Cartago.
CATALINN.Los que fings, y engais
las mujeres de esa suerte,
lo pagaris en la muerte.
DON JUAN.Qu largo me lo fiis! 20
Catalinn
3
con razn
te llaman.
CATALINN. Tus pareceres
sigue, que en burlar mujeres
quiero ser Catalinn.
Ya viene la desdichada. 25
DON JUAN.Vete, y las yeguas prevn.
CATALINN.Pobre mujer, harto bien
te pagamos la posada.
(Vase CATALINN y sale TISBEA.)
TISBEA.El rato que sin ti estoy
estoy ajena de m. 30
DON JUAN.Por lo que finges as
ningn crdito te doy.
TISBEA.Por qu?
DON JUAN. Porque si me amaras
mi alma favorecieras.
TISBEA.Tuya soy.
DON JUAN. Pues di, qu esperas, 35
o en qu, seora, reparas?
TISBEA.Reparo en que fue castigo
de amor el que he hallado en ti.
DON JUAN.Si vivo, mi bien, en ti
a cualquier cosa me obligo, 40
aunque yo sepa perder
en tu servicio la vida,
la diera por bien perdida,
y te prometo de ser
tu esposo. 45
Tirso DE MOLINA
El burlador de Sevilla
Ctedra
1
aprestar: aparejar, preparar, disponer lo necesario
para algo.
3
Eneas: legendario hroe de la guerra de Troya
que abandon a la reina Dido para seguir rumbo
a Italia, dejndola enamorada y desesperada.
3
Catalinn: probablemente, cobarde.
0B1LLLP(2008).17+s-def 3/6/08 18:48 Pgina 304
Evaluacin 17
295 17. El teatro: de los orgenes a la comedia nacional
1. En esta escena, Juan le propone a su padre Pedro Crespo
que compre una ejecutoria carta que exima de pagar
tributos al rey , para evitar alojar a don lvaro. Resume
los argumentos que esgrime el alcalde para negarse.
2. Qu tema del teatro barroco observas en este dilogo?
3. Analiza la mtrica empleada en este fragmento y reco-
noce los principales recursos expresivos utilizados.
4. Enuncia los rasgos de la comedia nacional y explcalos.
Cules de ellos reconoces en el fragmento?
5. Explica las diferencias entre el teatro cortesano y las
representaciones de los corrales de comedia.
6. Cul era el orden que se segua normalmente en las
representaciones de los corrales de comedia? Caracteriza
las piezas cortas que se representaban.
7. Seala los rasgos propios de los autos sacramentales.
8. Establece diferencias entre el teatro de Lope de Vega
y el de Caldern. Menciona obras de uno y otro.
9. Cules son los principales hitos del teatro renacentista
que influyeron en el teatro barroco? Recuerda autores
y obras del siglo &VI.
10. Redacta una exposicin sobre las manifestaciones
dramticas en la Edad Media.
JUAN.Que quieras, siendo t rico,
vivir a estos hospedajes
sujeto!
PEDRO CRESPO. Pues cmo puedo
excusarlos ni excusarme?
JUAN.Comprando una ejecutoria. 5
PEDRO CRESPO.Dime por tu vida, hay alguien
que no sepa, que yo soy,
si bien de limpio linaje,
hombre llano? No por cierto.
Pues, qu gano yo en comprarle 10
una ejecutoria al Rey,
si no le compro la sangre?
Dirn entonces, que soy
mejor que ahora? No, es dislate.
Pues qu dirn? Que soy noble 15
por cinco o seis mil reales;
y esto es dinero y no es honra;
que honra no la compra nadie.
Quieres, aunque sea trivial,
un ejemplillo escucharme? 20
Es calvo un hombre mil aos,
y al cabo de ellos se hace
una cabellera. Este,
en opiniones vulgares,
deja de ser calvo? No. 25
Pues qu dicen al mirarle?
Bien puesta la cabellera
trae Fulano. Pues qu hace,
si, aunque no le vean la calva,
todos que la tiene saben? 30
JUAN.Enmendar su vejacin,
remediarse de su parte,
y redimir vejaciones
del sol, del hielo y del aire.
PEDRO CRESPO.Yo no quiero honor postizo, 35
que el defecto ha de dejarme
en casa. Villanos fueron
mis abuelos y mis padres;
sean villanos mis hijos.
Pedro CALDERN DE LA BARCA
El alcalde de Zalamea, Ctedra
La dignidad del villano
Representacin de El alcalde de Zalamea, dirigida por Gustavo Prez Puig y Mara Recatero (2003).
0B1LLLA.17 7/4/08 17:16 Pgina 295
323 18. La literatura del siglo XVIII
M
A
T
E
R
I
A
L

F
O
T
O
C
O
P
I
A
B
L
E

/


O
x
f
o
r
d

U
n
i
v
e
r
s
i
t
y

P
r
e
s
s

E
s
p
a

a
,
S
.
A
.
ACTI VI DADES
18
REFUERZO
1. Poesa dieciochesca
En las primeras dcadas del siglo XVIII se sigui cultivando una poesa de rasgos barrocos, pero despus esta
prctica evolucion en distintas direcciones. A continuacin, puedes leer algunas composiciones represen-
tativas de las distintas manifestaciones que adopt la poesa en el denominado Siglo de las Luces.
De mi gusto
Retrico molesto,
deja de persuadirme
que ocupe bien el tiempo
y a mi Dorila olvide.
Ni t tampoco quieras 5
con rplicas sutiles.
del nctar de Leo
1
hacer que me desve.
Ni t, que al feroz Marte
muy ms errado sigues, 10
me angusties con pintarme
lo horrendo de sus lides.
Empero habladme todos
de bailes y de brindis,
de juegos y de amores, 15
de olores y convites,
que tras la edad florida
corre la vejez triste,
y antes que llegue quiero
holgarme y divertirme. 20
Juan MELNDEZ VALDS
Poesa y prosa, Planeta
A la invencin de la imprenta
Lleg, pues, el gran da
en que un mortal divino, sacudiendo
de entre la mengua universal la frente,
con voz omnipotente
dijo a la faz del mundo: EL HOMBRE
[ES LIBRE. 5
Y esta sagrada aclamacin saliendo,
no en los estrechos lmites hundida
se vio de una regin: el eco grande
que invent GUTENBERG la alza
[en sus alas;
y en ellas conducido 10
se mira en un momento
salvar los montes, recorrer los mares,
ocupar la extensin del vago viento,
y sin que el trono o su furor la asombre,
por todas partes el valiente grito 15
sonar de la razn: LIBRE ES EL
[HOMBRE.
Manuel Jos QUINTANA
A la muerte de Filis
En lgubres cipreses
he visto convertidos
los pmpanos de Baco
y de Venus los mirtos
2
;
cual ronca voz del cuervo 5
hiere mi triste odo
el siempre dulce tono
del tierno jilguerillo;
ni murmura el arroyo
con delicioso trino; 10
resuena cual peasco
con olas combatido.
En vez de los corderos
de los montes vecinos
rebaos de leones 15
bajar con furia he visto;
del sol y de la luna
los carros fugitivos
esparcen negras sombras
mientras dura su giro; 20
las pastoriles flautas,
que taen mis amigos,
resuenan como truenos
del que reina en Olimpo.
Pues Baco, Venus, aves, 25
arroyos, pastorcillos,
sol, luna, todos juntos
miradme compasivos,
y a la ninfa
3
que amaba
al infeliz Narciso, 30
mandad que diga al orbe
la pena de Dalmiro.
Jos CADALSO
El len y el ratn
Estaba un ratoncillo aprisionado
en las garras de un len; el desdichado
en tal ratonera no fue preso
por ladrn de tocino ni de queso,
sino porque con otros molestaba 5
al len, que en su retiro descansaba.
Pide perdn, llorando su insolencia;
al or implorar la real clemencia,
responde el Rey en majestuoso tono
(no dijera ms Tito): Te perdono. 10
Poco despus, cazando, el len tropieza
en una red oculta en la maleza;
quiere salir, mas queda prisionero;
atronando la selva ruge fiero.
El libre ratoncillo, que lo siente, 15
corriendo llega; roe diligente
los nudos de la red de tal manera
que al fin rompi los grillos de la fiera.
Conviene al poderoso
para los infelices ser piadoso; 20
tal vez se pueda ver necesitado
del auxilio de aquel ms desdichado.
Flix Mara SAMANIEGO
en Poesa del siglo XVIII, Castalia
1
nctar de Leo: referencia al vino; Leo
(el que relaja) es uno de los apelativos que
reciba el dios griego Dioniso.
2
mirto: arrayn. Arbusto de dos o tres metros
de altura, oloroso, con ramas flexibles y hojas
opuestas de color verde vivo.
3
ninfa: se refiere a la ninfa Eco.
Justifica la adscripcin
del texto De mi gusto
a la poesa rococ.
Analiza la mtrica
y la rima del texto de Juan
Melndez Valds
(caractersticas de la
anacrentica) y explica
el proceso comunicativo
interno del poema.
Compara el tono
de la anacrentica de
Cadalso con la de
Melndez Valds.
3
2
1
Por qu se puede
considerar prerromntico
el texto de Jos Cadalso?
Explica su proceso
comunicativo interno
y el contenido
del mensaje.
A qu tipo de poesa
del siglo XVIII pertenecen
los versos de Quintana?
Justifica tu respuesta.
Razona a qu gnero
y subgnero pertenece
el texto de Samaniego.
En qu grupo
de la poesa del siglo XVIII
se incluye?
Analiza la mtrica y la rima
de El len y el ratn
y divdelo en partes.
Seala los recursos
expresivos que observes
en los cuatro textos.
4
8
7
6
5
0B1LLLP(2008).18+s 3/6/08 15:52 Pgina 323
AMPLIACIN
2. La tragedia neoclsica: Raquel
Raquel, de Vicente Garca de la Huerta, se estren con gran xito en 1778 en el teatro Prncipe
de Madrid, pero ya se haba representado en Orn en 1772, donde estaba desterrado su autor.
La obra, que trata de los amores de Alfonso VIII con una juda de Toledo, sirve a Garca de la
Huerta para expresar su opinin acerca de cmo deba ser una monarqua en su poca.
Raquel responde a las caractersticas de la tragedia neoclsica: respeta las unidades de tiempo,
espacio y accin, adems del principio de verosimilitud, posee un nmero reducido de perso-
najes y pretende brindar una enseanza ejemplar.
324 18. La literatura del siglo XVIII
M
A
T
E
R
I
A
L

F
O
T
O
C
O
P
I
A
B
L
E

/


O
x
f
o
r
d

U
n
i
v
e
r
s
i
t
y

P
r
e
s
s

E
s
p
a

a
,
S
.
A
.
ACTI VI DADES
18
Despreciada
RAQUEL.Ay de m! qu he escuchado? T,
[Alvar Fez,
explcame este arcano.
ALVAR FEZ. Pues te avisan
que eres la ocasin de tantos males,
la respuesta te puedes dar t misma
RAQUEL.(A Rubn.) Estoy despierta o sueo
[por ventura? 5
RUBN.No s, Raquel: la misma duda agita
mi discurso y razn, imaginando
qu es cuanto he visto, sueo o fantasa.
RAQUEL.Qu especie de dolor tan inhumano
es este, oh corazn, que por primicias 10
de los males y sustos que me aguardan,
me ofrece la tirana suerte ma?
Quin de tanto favor se prometiera
tan no esperada, tan mortal cada?
Y quin hecha, fortuna, a tus halagos 15
pudiera recelarse tal desdicha?
Alfonso me aborrece; sus desvos
de mis temores la verdad confirman:
pues cmo podr ser ya venturosa,
la que se ve de Alfonso aborrecida? 20
Qu necio quien se fa de la suerte,
sin advertir que el tiempo y que los das,
que ciudades destruyen y edificios,
favores y privanzas aniquilan!
Qu causa puede haber, amado Alfonso, 25
para tanto desvo? Mis caricias
en qu te han ofendido, que por premio
solo odio y desagrado se concilian?
Mas ay de m! que en vano me desvelo
en buscar la ocasin de mis fatigas; 30
pues la suerte que empieza a perseguirme,
por doblarme el dolor, querr encubrirla.
RUBN.As, Raquel, tu corazn desmaya
en tan fuerte ocasin, donde es precisa
la constancia mayor? En los principios 35
si un mal, aunque sea leve, se descuida,
fuerzas del abandono va cobrando,
que el remedio despus inutilizan.
Reciente es este mal; an se est en tiempo
de poderle acudir; quien averigua 40
la causa de un dolor, con ms acierto
aplicarle podr la medicina.
Inquirase, Raquel, de esta desgracia
la ocasin; que despus de conocida,
si no cede a remedios ordinarios, 45
buscar los extremos mi malicia.
RAQUEL.Bien, Rubn, me aconsejas: en qu dudas?
Al yugo vuelva la cerviz altiva,
segunda vez Alfonso: el fin se logre,
y el medio sea cualquiera, que t elijas. 50
Lcito es cuanto sea conveniente:
propia moral de la venganza ma (Ruido dentro.)
Mas, ay de m! Qu estrpito confuso
orse deja? Al alma pronostica
el corazn, latiendo apresurado, 55
algn cercano mal.
RUBN. Ya ms distintas
se perciben las voces: nunca pruebas
mayores dio de s la cobarda,
que al escuchar rumor tan temeroso.
(Voz dentro.) Muera Raquel, para que Alfonso viva. 60
RAQUEL.No es delirio: verdad es la que toco:
y esto sufre mi enojo?, esto mis iras?
Espera, vulgo brbaro, atrevido,
que si mi sangre a derramar conspiras,
vers que a costa de la tuya sabe 65
defender y guardar Raquel su vida.
Vicente GARCA DE LA HUERTA
Raquel, Compaa Ibero-Americana de Publicaciones
Lee el fragmento y explica qu ocurre en l.
Cules son los sentimientos que embargan
a Raquel? Para responder, atiende
especialmente a su tercera intervencin.
Cmo reacciona Rubn? Qu le aconseja?
Se produce un cambio en la actitud de la
protagonista hacia el final del fragmento?
Argumenta tu respuesta.
Analiza la mtrica y la rima empleada:
es la que se emplea en toda la obra.
Encuentra en el texto ejemplos de discurso
valorativo y de discurso universal, y de las
funciones del lenguaje.
Reconoce en el fragmento recursos expresivos.
Establece diferencias entre la tragedia
y la comedia neoclsicas.
8
7
6
5
4
3
2
1
0B1LLLP(2008).18+s 3/6/08 15:52 Pgina 324
325 18. La literatura del siglo XVIII
M
A
T
E
R
I
A
L

F
O
T
O
C
O
P
I
A
B
L
E

/


O
x
f
o
r
d

U
n
i
v
e
r
s
i
t
y

P
r
e
s
s

E
s
p
a

a
,
S
.
A
.
ACTI VI DADES
18
AMPLIACIN
3. La comedia sentimental:
El delincuente honrado
En 1787 se estrena El delincuente honrado, de Gaspar Melchor de Jovellanos, obra cumbre
de la comedia sentimental. La obra desarrolla una historia melodramtica centrada en
un personaje, Torcuato, que aos atrs haba participado en un duelo en el que haba
matado al anterior marido de su esposa. El fragmento que tienes a continuacin corres-
ponde a la parte inicial de la comedia.
En qu estado se encuentra Torcuato al inicio
de la escena?
A qu atribuye Anselmo lo que le ocurre
a su amigo?
Explica por qu cree Anselmo que no hay que
preocuparse por el suceso acaecido y por qu,
por del contrario, Torcuato cree que s.
Encuentra en el fragmento ejemplos de las
funciones expresiva, apelativa y referencial.
Busca en las palabras de Torcuato un ejemplo
de discurso universal y explica su sentido.
A qu hace referencia el texto secundario?
Establece diferencias entre la comedia
sentimental y la comedia neoclsica.
7
6
5
4
3
2
1
Errores antiguos
ANSELMO.A fe, amigo mo, que me has hecho bien mala obra.
Dejar la cama a las siete de la maana! Hombre, no lo
hara ni por una duquesa; mas tu recado fue tan ejecutivo
(Despus de alguna pausa.) Pero, Torcuato, t ests triste
Tus ojos Vaya, apostemos a que has llorado?
TORCUATO.En mi dolor apenas he tenido ese pequeo desahogo.
ANSELMO.Desahogo las lgrimas? No lo entiendo. Pues qu,
un hombre como t no se correra
1
?
TORCUATO.Si las lgrimas son efecto de la sensibilidad del cora-
zn, desdichado de aquel que no es capaz de derramarlas!
ANSELMO.Como quiera que sea, yo no te comprendo. Torcuato,
tus ojos estn hinchados, tu semblante triste, y de algunos
das a esta parte noto que has perdido tu natural alegra.
Qu es esto? Cuando debieras Hombre, vamos claros;
quieres que te diga lo que he pensado? T acabas de casarte
con Laura, y por ms que la quieras, tener una mujer para
toda la vida, sufrir a un suegro viejo e impertinente, empezar a
sentir la falta de la dulce libertad y el peso de las obligaciones
del matrimonio, son sin duda para un joven graves motivos de
tristeza, y ve aqu a lo que atribuyo la tuya. Pero si esta es la
causa, t no tienes disculpa, amigo mo, porque te la has bus-
cado por tu mano. Por otra parte, Laura es virtuosa, es linda,
tiene un genio dcil y amable, te quiere mucho; y t, que has
sido siempre derretido, creo que no la vas en zaga. Sobre todo
(viendo que no le responde), Torcuato, t no debes afligirte
por frioleras; goza, con sosiego, de las dulzuras del matri -
monio, que ya llegar el da en que cada cual tome su partido.
TORCUATO.Ay, Anselmo! Esas dulzuras, que pudieran hacerme
tan dichoso, se van a cambiar en pena y desconsuelo; yo las
voy a perder para siempre.
ANSELMO.A perderlas? Pues qu? Ah! (Dndose una pal-
mada en la frente.) Ahora me acuerdo de que tu criada me
dijo no s qu de un viaje Pero yo estaba tan dormido
TORCUATO.T eres mi amigo, Anselmo, y voy a darte ahora la
ltima prueba de mi confianza.
ANSELMO.Pues sea sin prembulos, porque los aborrezco. Puedo
servirte en algo? Mi caudal, mis fuerzas, mi vida, todo es tuyo;
di lo que quieras, y si es preciso
TORCUATO.Ya sabes que fui autor de la muerte del marqus de
Montilla, y que este funesto
2
secreto, que hoy llena mi vida
de amargura, se conserva entre los dos.
ANSELMO.Es verdad; pero en cuanto al secreto no hay que rece-
lar. T sabes tambin cunto hice con Juanillo, el criado del
marqus, para alejar toda sospecha; pues aunque solo tena
algunos antecedentes del desafo, yo le gratifiqu, le traspuse
a Madrid, donde nadie le conoce, y mi amigo, el marqus de la
Fuente, est encargado de observar sus pasos. No; lejos de
pensar en ti ese bribn, tal vez creer Pero no hablemos
de eso, porque no es posible
TORCUATO.Ay, Anselmo, cunto te engaas! Ese criado est ya
en las crceles de Segovia.
Gaspar Melchor DE JOVELLANOS
Poesa. Teatro. Prosa
Taurus
1
no se correra: no se avergonzara.
2
funesto: aciago, que es origen de pesares o de ruina.
0B1LLLP(2008).18+s 3/6/08 15:52 Pgina 325
AMPLIACIN
4. La prosa dieciochesca: Vida
Diego de Torres Villarroel (Salamanca, 1694-1770) aprovecha la estructura de la novela picares-
ca, gnero que diverta a los lectores, para narrar su vida. Pero Villarroel cuenta, a diferencia del
pcaro que medra para mejorar su condicin, cmo progresa gracias a su esfuerzo y mritos.
Adems, sus orgenes no son miserables, sino que sus familiares, honrados, se ocupan de su ins-
truccin. En su relato emplear los recursos de la burla y la irona, tpicamente quevedescos.
Los sucesos narrados son pocos y su finalidad es aportar datos para ir configurando la persona-
lidad de su autor, objetivo de la obra, como puedes leer en el siguiente fragmento que corres-
ponde a la Introduccin.
Qu objetivo manifiesta el yo narrador
al comienzo? Qu visin presenta
de su persona? A qu la atribuye?
Justifica la presencia de los nombres propios
citados.
Explica la finalidad del tercer prrafo
en relacin con lo dicho en el segundo.
Encuentra en el texto ejemplos de discurso
universal.
Seala los recursos expresivos empleados. 5
4
3
2
1
326 18. La literatura del siglo XVIII
M
A
T
E
R
I
A
L

F
O
T
O
C
O
P
I
A
B
L
E

/


O
x
f
o
r
d

U
n
i
v
e
r
s
i
t
y

P
r
e
s
s

E
s
p
a

a
,
S
.
A
.
ACTI VI DADES
18
Un hombre de novela
Por lo mismo que ha tardado mi muerte, ya no pue-
de tardar; y quiero, antes de morirme, desvanecer con
mis confesiones y verdades, los enredos y mentiras que
me han abultado los crticos y los embusteros. La
pobreza, la mocedad, lo desentonado de mi aprensin,
lo ridculo de mi estudio, mis almanaques, mis coplas y
mis enemigos me han hecho hombre de novela, un
estudiantn extravagante y un escolar
1
entre brujo y
astrlogo, con visos
2
de diablo y perspectivas
3
de hechi-
cero. Los tontos que pican en eruditos me sacan y me
meten en sus conversaciones, y en los estrados y las
cocinas, detrs de un aforismo
4
del calendario, me
injieren una ridcula quijotada y me pegan un par de
aventuras descomunales; y, por mi desgracia y por su
gusto, ando entre las gentes hecho un mamarracho,
cubierto con el sayo que se les antoja, y con los parches
e hisopadas
5
de sus negras noticias. Paso, entre los que
me conocen y me ignoran, me abominan y me saludan,
por un Guzmn de Alfarache, un Gregorio de Guadaa
6
y un Lzaro de Tormes; y ni soy este, ni aquel, ni el otro; y
por vida ma, que se ha de saber quin soy. Yo quiero
meterme en corro; y ya que cualquier monigote presu-
mido se toma de mi murmuracin, murmuremos a
medias, que yo lo puedo hacer con ms verdad y con
menos injusticia y escndalo que todos. Sgase la con-
versacin, y crea despus el mundo a quien quisiere.
No me mueve a confesar en el pblico mis verdade-
ras liviandades el deseo de sosegar los chismes y las
parleras con que anda alborotado mi nombre y fora-
jida mi opinin
7
, porque el espritu no se altera con el
aire de las alabanzas, ni con el ruido de los vituperios.
A todo el mundo le dejo garlar
9
y decidir sobre lo que
sabe o lo que ignora, sobre m o sobre quien agarra el
vuelo su voluntad, su rabia o su costumbre.
Desde muy nio conoc que de las gentes no se
puede pretender ni esperar ms justicia ni ms miseri-
cordia que la que no le haga falta a su amor propio. En
los empeos de poca o mucha consideracin, cada
uno sigue su comodidad y sus ideas. Al que me alaba,
no se lo agradezco, porque, si me alaba, es porque le
conviene a su modestia o su hipocresa, y a ellas puede
pedir las gracias que yo no debo darle. Al que me corrige,
le oigo y lo dejo descabezar
10
; rome mucho de ver
cmo presume de consejero muy repotente
11
y gusto-
so de sus propias satisfacciones.
Diego DE TORRES VILLARROEL
Vida, ascendencia, nacimiento, crianza y aventuras
DeBolsillo
1
escolar: posible juego de palabras entre la acepcin
de atribuirse tanto a profesores como estudiantes que
convivan en una escuela y a la de nigromntico y
embustero que finge ser un pobre estudiante y mendiga.
2
viso: apariencia de las cosas.
3
perspectivas: falsas apariencias.
4
aforismo: sentencia breve y doctrinal que se propone
como regla.
5
hisopadas: rociadas de agua hechas con el hisopo.
6
Gregorio de Guadaa: protagonista de la novela
picaresca El siglo pitagrico y vida de Don Gregorio
Guadaa (1644), de Antonio Enrquez Gmez.
7
opinin: aqu fama, honra.
8
vituperio: baldn u oprobio que se dice a uno.
9
garlar: hablar mucho y con poca discrecin.
10
descabezar: (coloq.) empezar a vencer la dificultad
o tropiezo que se encuentre en algo.
11
repotente: engredo, prepotente.
0B1LLLP(2008).18+s 3/6/08 15:52 Pgina 326
327 18. La literatura del siglo XVIII
M
A
T
E
R
I
A
L

F
O
T
O
C
O
P
I
A
B
L
E

/


O
x
f
o
r
d

U
n
i
v
e
r
s
i
t
y

P
r
e
s
s

E
s
p
a

a
,
S
.
A
.
ACTI VI DADES
18
AMPLIACIN
5. Ensayo e Ilustracin
El ensayo espaol est estrechamente vinculado a la difusin de las ideas de la Ilustracin. El
autor del siguiente texto reflexiona sobre este fenmeno y brinda algunos de los rasgos ms
importantes que caracterizaron este gnero en el siglo XVIII.
Explica la importancia que tuvo, segn el texto
de Francisco Snchez Blanco, la irrupcin
del ensayo en el siglo XVIII.
Por qu con este gnero dejaron de tener
relevancia las cuestiones formales?
En qu se fijaron, sobre todo, los escritores?
Indica qu circunstancias favorecieron
el ambiente cultural de la poca
que nos ocupa.
Qu tiene siempre en cuenta el escritor
ilustrado?
Seala qu consecuencias tuvo en el
pensamiento dieciochesco el escepticismo
reinante y a qu campos se aplic el mtodo
emprico.
Por qu en el perodo ilustrado se publicaron
numerosos tratados de determinadas ciencias?
Qu se buscaba finalmente?
Desde qu perspectiva se abordaron
las reformas?
Escribe un resumen del texto de Francisco
Snchez Blanco.
8
7
6
5
4
3
2
1
Sin necesidad de establecer una ecuacin perfecta, se pue-
de afirmar que el gnero ensayo est ntimamente vinculado
al fenmeno cultural que se conoce con el nombre de Ilustra-
cin. Gran parte del saber erudito, antes expresado en latn, se
vierte ahora a la lengua verncula
1
: ideas, antes solo asequi-
bles a los que haban aprendido una lengua culta o un idioma
extranjero, llegan a un amplio pblico, que se arroga, adems,
la facultad de discurrir y sentenciar sobre todo lo divino y lo
humano. Este fenmeno histrico condiciona esencialmente
la escritura y modifica el lenguaje, hacindolo ms gil y fami-
liar. Aquellos modelos textuales que se adaptan mejor al esp-
ritu y al contenido de mensajes dirigidos al conjunto de los
ciudadanos atraen especialmente la atencin de los escritores.
Pero la tenaz curiosidad por conocer novedades, tanto a nivel
cientfico como poltico, desplaza a segundo plano cuestiones
meramente formales o retricas. Cuando sobre el tapete se
ponen temas como la libertad de pensamiento, los privilegios
estamentales
2
del clero y la nobleza, el derecho a opinar en
cuestiones acadmicas o gubernamentales, la reforma univer-
sitaria y multitud de otros asuntos de inters pblico, da hasta
cierto punto igual si sobre ello se escribe en forma de dilogo
amistoso, de apasionada oracin o de sosegada exposicin
didctica. Los autores se concentran ms en el fondo que en la
forma.
Indudablemente, lo que el ambiente cultural de la poca no
favorece es que alguien prescinda de la tertulia de amigos
o que simplemente haga abstraccin del pblico opinante y se
encierre en su biblioteca privada a elucubrar
3
sobre el ms all
o a departir con autores difuntos. El hombre dieciochesco se
considera ante todo miembro de una colectividad contempo-
rnea y, por tanto, siente responsabilidad frente a ella. No
muestra, pues, demasiada propensin a rumiar solitariamente
sobre la brevedad de la vida o sobre consuelos que alivien la
aciaga
4
fortuna individual, sino que prefiere pensar en plural
afrontando sus deberes de patriota y de ciudadano y propo-
niendo soluciones tiles para el comn. []
Los pensadores del siglo XVIII meditan sobre intereses colecti-
vos, basndose en una concepcin cosmopolita de la razn.
[] El ilustrado no pierde nunca de vista el horizonte de la hu-
manidad aun cuando proponga reformas especficas para la
propia patria. Ciertamente, muchos ilustrados retienen todava
el principio barroco de contribuir al esplendor de la monarqua
y del trono, pero no todos creen que el gobierno monrquico
sea el nico legtimo o necesariamente el mejor. En este punto,
como en otros, no suelen hacer profesin de fe.
Un escepticismo generalizado domina la actitud intelectual
y hace que todo se someta, primero, a examen y, despus, a la
prueba de la experiencia. Por eso se tratan preferentemente
aquellos temas que admiten solucin inmediata: tcnicas agr-
colas, mejoras de las manufacturas o reformas administrativas.
Se confa en el progreso del conocimiento y en el creciente pre-
dominio de la naturaleza. El mtodo emprico se aplica a todos
los campos del saber. En consecuencia surge una extraordina-
ria demanda de tratados de fsica, botnica, qumica e incluso
zoologa, es decir, de todas las ciencias que pueden contribuir
al progreso de la ganadera, de la agricultura y de la industria.
Pero ese inters cognoscitivo est subordinado a aumentar la
felicidad pblica y privada, lo cual implica tambin mejorar el
ordenamiento de la sociedad. Aunque la discusin recuerda las
causas de la decadencia de Espaa, el planteamiento de las re-
formas se enfoca desde una perspectiva cosmopolita, es decir,
alzando la mirada por encima de la barrera de los Pirineos
y observando lo que hacen las dems naciones.
Francisco SNCHEZ BLANCO
Prlogo a El ensayo espaol: el siglo XVIII, Crtica
1
vernculo: domstico, nativo, de nuestra casa o pas.
2
estamental: perteneciente o relativo a un estrato social
determinado.
3
elucubrar: elaborar una divagacin complicada y con apariencia
profunda.
4
aciago: infeliz, desgraciado.
Autores ilustrados
0B1LLLP(2008).18+s 3/6/08 15:52 Pgina 327
Evaluacin 18
309 18. La literatura del siglo XVIII
1. Resume el contenido de este texto de Benito Jernimo
Feijoo.
2. Indica cul es el tema que trata el autor en este frag-
mento y menciona qu otros asuntos suele abordar en
sus escritos.
3. Reconoce en el texto las expresiones valorativas y los
principales recursos estilsticos.
4. Seala la diferencia que existe entre Benito Jernimo
Feijoo y Gaspar Melchor de Jovellanos en cuanto a los
destinatarios de sus escritos. Qu temas aborda este
ltimo autor? Menciona obras de ambos escritores.
5. En qu obra analiza Cadalso la situacin de Espaa?
Explica cmo la estructura.
6. Comenta las principales vertientes poticas del siglo &VIII
y nombra los principales autores.
7. Cul es la finalidad del teatro neoclsico? Explica las
caractersticas de sus dos principales manifestaciones.
Ejemplifica cada una de ellas con una obra y su autor.
8. Seala otras orientaciones dramticas del siglo &VIII y
nombra los dramaturgos ms destacados.
9. Menciona algunas de las principales instituciones ilus-
tradas.
10. Escribe un texto sobre la literatura predominante en la
primera mitad del siglo &VIII y la que comienza a cultivar-
se en la segunda mitad de la centuria. No olvides citar
las obras y los autores ms importantes.
Levntase el rumor de algn portento
1
sucedido en un pueblo,
a que dio principio o un embustero o un alucinado, y no pocos,
que tienen igual ligereza en la creencia que en la pluma, despa-
chan por la estafeta
2
la noticia a otras partes. Aclrase despus la
verdad y viene el desengao para aquel pueblo, mas no para los
otros donde se comunic la especie, porque los que la escribieron
o no se acuerdan (especialmente si medi espacio de tiempo con-
siderable) de escribir el desengao, o no quieren hacerlo, porque
no se les atribuya el primer asenso
3
a ligereza, con que queda
estampada en los otros pueblos la patraa
4
, porque no lleg el caso
de ratificacin a los testigos y deshacer en la plenaria
5
el engao
padecido en la sumaria
6
.
En todo el pueblo de Llanes (distante de esta capital dieciocho
leguas) corri uno de estos aos pasados por indubitable la
existencia de un duende, gran enredador, que se deca infestaba
continuamente una de las casas de aquella villa.
Llegaron aqu repetidas noticias del caso, tan circunstanciadas
y citando tantos testigos de vista que aun los ms incrdulos de
duendes llegaron a dar asenso, y de m confieso que estuve harto
inclinado a lo mismo. Sin embargo, despus, por muchos y segur-
simos informes se supo que el duende haba salido fingido, y que
dos muchachas, con un enredillo bien poco artificioso, haban
puesto a todo el pueblo en aquella creencia. Pero, quin duda
que el engao que con facilidad pudo venir aqu por el continuo
comercio que hay entre los dos lugares no llegara a otros muchos
adonde se haba escrito el embuste?
Benito Jernimo FEIJOO
Obras (Seleccin), Taurus
1
portento: cosa, accin o suceso singular que por su extraeza o novedad
causa admiracin o terror.
2
estafeta: casa u oficina del correo.
3
asenso: asentimiento; aquello a lo que se da crdito.
4
patraEa: mentira o noticia fabulosa, de pura invencin.
5
plenaria: general, plena.
6
sumaria: reducida; parte de algo.
Falsas creencias
Las brujas, por Francisco de Goya.
0B1LLLA.18 1/4/08 10:19 Pgina 309
REFUERZO
1. Las acotaciones en el teatro
romntico
La escenografa tuvo un desarrollo muy importante en el siglo XIX. Al levantarse el teln, la escena
se converta en un lugar de ilusin que se procuraba que reprodujese fielmente los espacios
reales y los imaginados por los dramaturgos, y en los cuales tuvo gran relevancia la iluminacin.
Reconoce en las cuatro acotaciones
reproducidas en esta pgina la presentacin
de cuadros costumbristas, sealando
sus elementos.
Cmo se presenta el protagonista
en la escena III de la jornada primera?
Ten en cuenta que es la primera vez
que aparece en escena.
Seala en los textos elementos relacionados
con los cdigos visuales, detallando
a qu elementos de la representacin
hacen referencia.
Localiza ahora los elementos referidos
a los cdigos auditivos.
Qu tipo de atmsfera se pretende crear
con la ltima acotacin?
5
4
3
2
1
339 19. La literatura del romanticismo
M
A
T
E
R
I
A
L

F
O
T
O
C
O
P
I
A
B
L
E

/


O
x
f
o
r
d

U
n
i
v
e
r
s
i
t
y

P
r
e
s
s

E
s
p
a

a
,
S
.
A
.
ACTI VI DADES
1
9
1
aguaducho: puesto donde se vende agua.
2
anafre: hornillo.
3
flmula: gallardete muy corto.
4
gallardete: tira volante que disminuye hasta acabar en punta
colocada en los mstiles.
5
Hornachuelos: villa de la provincia de Crdoba.
6
candiln: candil grande.
7
cribar: limpiar el trigo u otra semilla por medio de la criba,
del polvo, tierra y dems impurezas.
8
jalma: aparejo que se emplea en las bestias de carga.
Jornada primera
Escena I
La escena representa la entrada del antiguo puente
de barcas de Triana, el que estar practicable a la dere-
cha. En primer trmino, al mismo lado, un aguaducho
1
o barraca de tablas y lonas, con un letrero que diga:
Agua de Tomares; dentro habr un mostrador rstico
con cuatro grandes cntaros, macetas de flores, vasos, un
anafre
2
con una cafetera de hojalata y una bandeja con
azucarillos. Delante del aguaducho habr bancos de
pino. Al fondo se descubrir de lejos, parte del arrabal de
Triana, la huerta de los Remedios con sus altos cipreses,
el ro y varios barcos en l, con flmulas
3
y gallardetes
4
. A
la izquierda se ver en lontananza la Alameda. Varios
habitantes de Sevilla cruzarn en todas direcciones
durante la escena. El cielo demostrar el ponerse del sol
en una tarde de julio [].
Jornada primera
Escena III
Empieza a anochecer, y se va oscureciendo el teatro.
DON LVARO sale embozado en una capa de seda, con un
gran sombrero blanco, botines y espuelas; cruza lenta-
mente la escena, mirando con dignidad y melancola a
todos los lados, y se va por el puente. Todos lo observan
en gran silencio.
Jornada segunda
Escena I
Es de noche, y el teatro representa la cocina de
un mesn de la villa de Hornachuelos
5
. Al frente
estar la chimenea y el hogar. A la izquierda, la
puerta de entrada; a la derecha, dos puertas practi-
cables. A un lado, una mesa larga de pino, rodeada
de asientos toscos, y alumbrado todo por un gran
candiln
6
. El MESONERO y el ALCALDE aparecern
sentados gravemente al fuego. La MESONERA, de
rodillas, guisando. Junto a la mesa, el ESTUDIANTE,
cantando y tocando la guitarra. El ARRIERO que
habla, cribando
7
cebada en el fondo del teatro. El
TO TRABUCO, tendido en primer trmino sobre sus
jalmas
7
. Los dos LUGAREOS, las dos LUGAREAS, la
MOZA y uno de los ARRIEROS, que no habla, estarn
bailando seguidillas. []
Jornada quinta
Escena X
El teatro representa un valle rodeado de riscos
inaccesibles y de malezas, atravesado por un arro-
yuelo. Sobre un peasco accesible con dificultad,
y colocado al fondo, habr una medio gruta, medio
ermita, con puerta practicable, y una campana que
pueda sonar y tocarse desde dentro; el cielo repre-
sentar el ponerse del sol de un da borrascoso, se
ir oscureciendo lentamente la escena y aumen-
tndose los truenos y los relmpagos. []
Duque DE RIVAS
Don lvaro o la fuerza del sino, Ctedra
0B1LLLP(2008).19+s 3/6/08 18:50 Pgina 339
REFUERZO
2. Don lvaro o la fuerza del sino,
del duque de Rivas
340 19. La literatura del romanticismo
M
A
T
E
R
I
A
L

F
O
T
O
C
O
P
I
A
B
L
E

/


O
x
f
o
r
d

U
n
i
v
e
r
s
i
t
y

P
r
e
s
s

E
s
p
a

a
,
S
.
A
.
Explica por qu podemos decir que estamos
en presencia de un texto literario y adscrbelo
al gnero al que pertenece.
Cmo reacciona don lvaro al ver abatida
a doa Leonor? Qu responde ella?
Reconoce en sus palabras caractersticas
tpicas de la mujer en el teatro romntico.
Qu peculiaridad presenta el texto principal?
Reconoce en l enunciados caractersticos
del drama romntico y algunos recursos
expresivos.
A qu elementos de la representacin hacen
referencia las acotaciones?
Realiza un resumen del fragmento. 5
4
3
2
1
ACTI VI DADES
1
9
El siguiente fragmento corresponde a la escena VII de la jornada primera de una obra cumbre
del teatro romntico, Don lvaro o la fuerza del sino, de ngel de Saavedra, duque de Rivas.
El protagonista se encuentra con su amada: estn dispuestos a huir juntos, pero Leonor se
encuentra muy abatida.
La huida
DOA LEONOR. Don lvaro!
DON LVARO. Leonor! (Pausa.)
Fuerza bastante
hay para todo en m Desventurado!
La conmocin conozco que te agita,
inocente Leonor. Dios no permita
que por debilidad en tal momento
sigas mis pasos y mi esposa seas.
Renuncio a tu palabra y juramento;
hachas de muerte las nupciales teas
fueran para los dos Si no me amas
como te amo yo a ti Si arrepentida
DOA LEONOR.Mi dulce esposo, con el alma y vida
es tuya tu Leonor; mi dicha fundo
en seguirte hasta el fin del ancho mundo.
Vamos; resuelta estoy, fij mi suerte,
separarnos podr solo la muerte.
(Van hacia el balcn, cuando de repente se oye ruido,
ladridos y abrir y cerrar de puertas.)
DOA LEONOR.Dios mo! Qu ruido es ese? Don lvaro!
CURRA.Parece que han abierto las puertas del patio y la
de la escalera
DOA LEONOR.Se habr puesto malo mi padre?
CURRA.Qu! No, seora; el ruido viene de otra parte.
DOA LEONOR.Habr llegado alguno de mis hermanos?
DON LVARO.Vamos, vamos, Leonor; no perdamos ni un ins-
tante.
(Vuelven hacia el balcn y de repente se ve por l el
resplandor de hachones de viento y se oye el galopar de
caballos.)
DOA LEONOR.Somos perdidos! Estamos descubiertos Im-
posible es la fuga.
DON LVARO.Serenidad es necesario en todo caso.
CURRA.La Virgen del Rosario nos valga y las nimas bendi-
tas! Qu ser de mi pobre Antonio? (Se asoma al bal-
cn y grita.) Antonio! Antonio!
DON LVARO.Calla, maldita! No llames la atencin hacia es-
te lado; entorna el balcn. (Se acerca el ruido de puertas
y pisadas.)
DOA LEONOR.Ay, desdichada de m! Don lvaro, escn -
dete aqu en mi alcoba
DON LVARO.(Resuelto.) No, yo no me escondo No te
abandono en tal conflicto. (Prepara una pistola.) Defen-
derte y salvarte es mi obligacin.
DOA LEONOR.(Asustadsima.) Qu intentas? Ay! Retira esa
pistola que me hiela la sangre Por Dios, sultala! La
disparars contra mi buen padre? Contra alguno de
mis hermanos? Para matar a alguno de los fieles y an-
tiguos criados de esta casa?
DON LVARO.(Profundamente confundido.) No, no, amor
mo La emplear en dar fin a mi desventurada vida.
DOA LEONOR. Qu horror! Don lvaro!
Duque DE RIVAS
Don lvaro o la fuerza del sino, Ctedra
0B1LLLP(2008).19+s 3/6/08 18:50 Pgina 340
341 19. La literatura del romanticismo
M
A
T
E
R
I
A
L

F
O
T
O
C
O
P
I
A
B
L
E

/


O
x
f
o
r
d

U
n
i
v
e
r
s
i
t
y

P
r
e
s
s

E
s
p
a

a
,
S
.
A
.
ACTI VI DADES
1
9
AMPLIACIN
3. El seor de Bembibre,
de Gil y Carrasco
Enrique Gil y Carrasco (1815, Villafranca del Bierzo provincia de Len; 1846, Berln) realiz
estudios en Ponferrada con los monjes agustinos, en el monasterio benedictino de San Andrs,
de Vega, y en el seminario de Astorga. En 1832 se instal en Valladolid y en 1836 se traslad
a Madrid, donde entr en contacto con literatos de la poca, como Espronceda, Larra, Zorrilla
y Rivas. En 1844 public su novela El seor de Bembibre y viaj a Berln como secretario de
Legacin, donde falleci dos aos despus.
Gil y Carrasco fue autor tambin de poesa, artculos periodsticos y relatos de viajes. El seor de
Bembibre narra los trgicos amores de don lvaro y doa Beatriz vinculados a la desaparicin
de la Orden del Temple.
Justifica por qu el fragmento corresponde
a un texto literario puede adscribirse
a la narrativa.
Analiza en el texto los elementos
correspondientes al nivel de la historia.
Personajes Accin
Tiempo Espacio
Atendiendo al tiempo de la historia y al tiempo
del discurso, qu movimiento narrativo
se observa?
Caracteriza al narrador y seala los tipos
de discurso empleados.
Comenta la adjetivacin y los recursos
expresivos en el fragmento.
5
4
3
2
1
En edad de casarse
Don Alonso Ossorio, seor de Arganza, haba tenido
dos hijos y una hija; pero de los primeros muri uno antes
de salir de la infancia, y el otro muri peleando como bueno
en su primera campaa contra los moros de Andaluca.
As, pues, todas sus esperanzas haban venido a cifrarse en
su hija doa Beatriz, que entonces tena pocos aos, pero
que ya prometa tanta belleza como talento y generosa
ndole. Haba en su carcter una mezcla de la energa que
distingua a su padre y de la dulzura y melancola de doa
Blanca de Balboa, su madre, santa seora cuya vida haba
sido un vivo y constante ejemplo de bondad, de resigna-
cin y de piedad cristiana. Aunque con la prdida tempra-
na de sus dos hijos su complexin, harto delicada por des-
gracia, se haba arruinado enteramente, no fue esto
obstculo para que en la crianza esmerada de su hija
emplease una instruccin poco comn en aquella poca,
y fecundase las felices disposiciones de que la haba dota-
do prdigamente la naturaleza. Sin ms esperanza que
aquella criatura tan querida y hermosa, sobre ella amon-
tonaba su ternura, todas las ilusiones del deseo y los sueos
del porvenir. As creca doa Beatriz como una azucena
gentil y fragante al calor del cario maternal, defendida
por el nombre y poder de su padre y cercada por todas
partes del respeto y amor de sus vasallos, que contempla-
ban en ella una medianera segura para aliviar sus males
y una constante dispensadora de beneficios.
Los aos pasaban en tanto rpidos como suelen, y con
ellos vol la infancia de aquella joven tan noble, agraciada
y rica, a quien por lo mismo pens buscar su padre un
esposo digno de su clase y elevadas prendas. En el Bierzo
no haba entonces ms de dos casas cuyos estados y vasa-
llos estuviesen al nivel: una la de Arganza, otra la de
la antigua familia de los Yez, cuyos dominios compren-
dan la frtil ribera de Bembibre y la mayor parte de las
montaas comarcanas. Este linaje haba dado dos maes-
tres al orden del Temple y era muy honrado y acatado en el
pas. Por una rara coincidencia a la manera que el apellido
Ossorio penda de la frgil existencia de una mujer, el de
Yez estaba vinculado en la de un solo hombre no
menos frgil y deleznable en aquellos tiempos de desdicha
y turbulencias. Don lvaro Yez y su to don Rodrigo
1
,
maestre del Temple en Castilla, eran los dos nicos miem-
bros que quedaban de aquella raza ilustre y numerosa;
rama seca y estril el uno, por su edad y sus votos, y vsta-
go el otro, lleno de savia y lozana, que prometa larga vida
y sonados frutos.
Enrique GIL Y CARRASCO
El seor de Bembibre
Ctedra
1
Don Rodrigo: personaje histrico real, ltimo maestre del
Temple en Castilla y Len.
0B1LLLP(2008).19+s 3/6/08 18:50 Pgina 341
342 19. La literatura del romanticismo
M
A
T
E
R
I
A
L

F
O
T
O
C
O
P
I
A
B
L
E

/


O
x
f
o
r
d

U
n
i
v
e
r
s
i
t
y

P
r
e
s
s

E
s
p
a

a
,
S
.
A
.
ACTI VI DADES
1
9
REFUERZO
4. Artculos, de Larra
En su empeo por la reforma cultural y social del pas, Larra critic actitudes y comporta-
mientos de sus contemporneos y, a la vez, se opuso a su tendencia a la visin negativa
y denigrante de Espaa. En este artculo reflexiona sobre la frase hecha que le da ttulo y
que la mayora repite, y exhorta a sus connacionales a cambiar ciertos hbitos y comprome-
terse con el bien comn.
Qu preguntas se hace Larra sobre el origen
de la frase?
Qu respuesta aventura? Con qu
comparacin la desarrolla?
En qu estado considera que se encuentra
el pas en su poca?
Qu consejos da Mariano Jos de Larra
a modo de conclusin?
Realiza un resumen del texto.
Reconoce las modalidades textuales
presentes en este fragmento
de En este pas.
6
5
4
3
2
1
En este pas
En este pas Esta es la frase que todos repetimos a
porfa, frase que sirve de clave para toda clase de explica-
ciones, cualquiera que sea la cosa que a nuestros ojos cho-
que en mal sentido.
Qu quiere usted? decimos, en este pas!.
Cualquier acontecimiento desagradable que nos suceda,
creemos explicarle perfectamente con la frasecilla: Cosas
de este pas!, que con vanidad pronunciamos y sin pudor
alguno repetimos.
Nace esta frase de un atraso reconocido en toda la
nacin? No creo que pueda ser este su origen, porque solo
puede conocer la carencia de una cosa el que la misma
cosa conoce: de donde se infiere que si todos los individuos
de un pueblo conociesen su atraso, no estaran realmente
atrasados. Es la pereza de imaginacin o de raciocinio,
que nos impide investigar la verdadera razn de cuanto
nos sucede, y que se goza en tener una muletilla siempre a
mano con que responderse a sus propios argumentos,
hacindose cada uno la ilusin de no creerse cmplice de
un mal, cuya responsabilidad descarga sobre el estado del
pas en general? Esto parecera ms ingenioso que cierto.
Creo entrever la causa verdadera de esta humillante
expresin. Cuando se halla un pas en aquel crtico momen-
to en que se acerca a una transicin, y en que, saliendo de
las tinieblas, comienza a brillar a sus ojos un ligero resplan-
dor, no conoce todava el bien, empero ya conoce el mal, de
donde pretende salir para probar cualquiera otra cosa que
no sea lo que hasta entonces ha tenido. Sucdele lo que a
una joven bella que sale de la adolescencia; no conoce el
amor todava ni sus goces; su corazn, sin embargo, o la
naturaleza, por mejor decir, le empieza a revelar una nece -
sidad que pronto ser urgente para ella, y cuyo germen
y cuyos medios de satisfaccin tiene en s misma, si bien los
desconoce todava; la vaga inquietud de su alma, que bus-
ca y ansa, sin saber qu, la atormenta y la disgusta de su
estado actual y del anterior en que viva; y vsela despreciar
y romper aquellos mismos sencillos juguetes que formaban
poco antes el encanto de su ignorante existencia.
Este es acaso nuestro estado, y este, a nuestro entender,
el origen de la fatuidad que en nuestra juventud se observa:
el medio saber reina entre nosotros; no conocemos el bien,
pero sabemos que existe y que podemos llegar a poseerle, si
bien sin imaginar an el cmo. Afectamos, pues, hacer
ascos de lo que tenemos para dar a entender a los que nos
oyeron que conocemos cosas mejores, y nos queremos
engaar miserablemente unos a otros, estando todos en el
mismo caso.
Este medio saber nos impide gozar de lo bueno que
realmente tenemos, y aun nuestra ansia de obtenerlo todo
de una vez nos ciega sobre los mismos progresos que
vamos insensiblemente haciendo. []
Olvidemos, lo repetimos, esa funesta expresin que con-
tribuye a aumentar la injusta desconfianza que de nuestras
propias fuerzas tenemos. Hagamos ms favor o justicia a
nuestro pas, y cremosle capaz de esfuerzos y felicidades.
Cumpla cada espaol con sus deberes de buen patricio, y
en vez de alimentar nuestra inaccin con la expresin de
desaliento: Cosas de Espaa! contribuya cada cual a las
mejoras posibles. Entonces este pas dejar de ser tan mal
tratado de los extranjeros, a cuyo desprecio nada podemos
oponer, si de l les damos nosotros mismos el vergonzoso
ejemplo.
Mariano Jos DE LARRA
Artculos, Castalia
0B1LLLP(2008).19+s 3/6/08 18:50 Pgina 342
343 19. La literatura del romanticismo
M
A
T
E
R
I
A
L

F
O
T
O
C
O
P
I
A
B
L
E

/


O
x
f
o
r
d

U
n
i
v
e
r
s
i
t
y

P
r
e
s
s

E
s
p
a

a
,
S
.
A
.
ACTI VI DADES
1
9
AMPLIACIN
5. El reo de muerte,
de Espronceda
En sus canciones Espronceda cuestiona y denuncia realidades y actitudes de su poca: la hipo-
cresa, el egosmo, la indiferencia frente al dolor, la impiedad. Estas crticas estn presentes,
sobre todo, en El reo de muerte, El verdugo y El mendigo.
En la cancin que presentamos aqu, el joven reo, en la noche, desesperado, vive sus ltimas
horas; desde la celda oye el bullicio de la calle y la frase que se cita; luego reina el silencio: todos
duermen, indiferentes ante su muerte.
El reo de muerte
[] Loca y confusa la encendida mente,
sueos de angustia y fiebre y devaneo,
el alma envuelven del confuso reo,
que inclina al pecho la abatida frente.
Y en sueos 5
confunde
la muerte,
la vida.
Recuerda
y olvida, 10
suspira,
respira
con hrrido afn.
Y en un mundo de tinieblas
vaga y siente miedo y fro, 15
y en su horrible desvaro
palpa en su cuello el dogal
1
;
y cuanto ms forcejea,
cuanto ms lucha y porfa,
tanto ms en su agona 20
aprieta el nudo fatal.
Y oye ruidos, voces, gentes,
y aquella voz que dir:
Para hacer bien por el alma
del que van a ajusticiar!
2
25
O ya libre se contempla,
y el aire puro respira,
y oye de amor que suspira
la mujer que a un tiempo am,
bella y dulce cual sola, 30
tierna flor de primavera,
el amor de la pradera
que el abril galn mim.
Y gozoso a verla vuela,
y alcanzarla intenta en vano, 35
que al tender la ansiosa mano
su esperanza a realizar,
su ilusin la desvanece
de repente el sueo impo,
y halla un cuerpo mudo y fro 40
y un cadalso en su lugar.
Y oye a su lado en son triste
lgubre voz resonar:
Para hacer bien por el alma
del que van a ajusticiar! 45
Jos DE ESPRONCEDA
Poesas lricas y fragmentos picos, Castalia
1
dogal: cuerda para ahorcar a un reo.
2
Para hacer [] ajusticiar!: frase que decan los
cofrades de Paz y Caridad, que asistan a los
condenados, cuando pedan limosnas para el reo.
Qu situacin se describe hasta el verso 25?
Y del verso 26 hasta el final?
Explica el contraste que se establece
entre las dos partes.
En qu se insiste al final del texto?
Analiza la mtrica y los principales recursos
estilsticos empleados por Espronceda
en estos versos.
4
3
2
1
0B1LLLP(2008).19+s 3/6/08 18:50 Pgina 343
344 19. La literatura del romanticismo
M
A
T
E
R
I
A
L

F
O
T
O
C
O
P
I
A
B
L
E

/


O
x
f
o
r
d

U
n
i
v
e
r
s
i
t
y

P
r
e
s
s

E
s
p
a

a
,
S
.
A
.
ACTI VI DADES
1
9
REFUERZO
6. Rimas, de Bcquer
En las Rimas encontramos textos que definen la interioridad, la conciencia de la voz potica
y tambin otros que definen el yo y el t femenino, claves en la obra del autor, y las relaciones
imposibles entre ambos.
Identifica los elementos que se nombran
en las cuatro primeras estrofas de la Rima 15
y menciona:
Qu movimientos realizan.
Quin o qu los causa.
Cules son sus destinos.
La ltima estrofa aclara que los elementos que
has nombrado en la actividad 1 se identifican
con el yo potico, qu similitudes
se establecen?
Determina el tema del poema y analiza
los recursos expresivos que lo estructuran.
Qu elementos se identifican con el t
en la Rima 60? A qu mbito pertenecen?
Consigue comunicarse el yo potico
con la destinataria?
En la tercera estrofa se describe el yo potico,
qu elementos aparecen y qu rasgos suyos
transmiten?
Explica el contenido de la ltima estrofa;
qu persigue la voz potica?
Reconoce los recursos expresivos principales
con que se desarrolla el contenido.
Realiza el anlisis mtrico de ambas
composiciones.
Relaciona los temas de estos textos
con los de la obra potica del autor.
10
9
8
7
6
5
4
3
2
1
Rima 15 (II)
Saeta que voladora
cruza, arrojada al azar,
y que no se sabe dnde
temblando se clavar;
hoja que del rbol seca 5
arrebata el vendaval,
y que no hay quien diga el surco
donde el polvo volver.
Gigante ola que el viento
riza y empuja en el mar 10
y rueda y pasa y se ignora
qu playa buscando va.
Luz que en cercos temblorosos
brilla prxima a expirar,
y que no se sabe de ellos 15
cul el ltimo ser.
Eso soy yo que al acaso
cruzo el mundo sin pensar
de dnde vengo ni a dnde
mis pasos me llevarn. 20
Gustavo Adolfo BCQUER
Libro de los gorriones, Planeta
Rima 60 (XV)
Cendal flotante de leve bruma,
rizada cinta de blanca espuma,
rumor sonoro de arpa de oro,
beso del aura, onda de luz,
eso eres t. 5
T, sombra area, que cuantas veces
voy a tocarte te desvaneces.
Como la llama, como el sonido,
como la niebla, como el gemido
del lago azul! 10
En mar sin playas onda sonante,
en el vaco cometa errante,
largo lamento
del ronco viento,
ansia perpetua de algo mejor, 15
eso soy yo.
Yo, que a tus ojos en mi agona
los ojos vuelvo de noche y da;
yo, que incansable corro y demente
tras una sombra, tras la hija ardiente 20
de una visin!
Gustavo Adolfo BCQUER
Libro de los gorriones, Planeta
0B1LLLP(2008).19+s 3/6/08 18:50 Pgina 344
345 19. La literatura del romanticismo
M
A
T
E
R
I
A
L

F
O
T
O
C
O
P
I
A
B
L
E

/


O
x
f
o
r
d

U
n
i
v
e
r
s
i
t
y

P
r
e
s
s

E
s
p
a

a
,
S
.
A
.
ACTI VI DADES
1
9
AMPLIACIN
7. Las Leyendas, de Bcquer
Las Leyendas contienen narraciones muy diversas de las que se han destacado sus valores esti-
lsticos (se ha hablado de poemas en prosa) en relacin con la prosa de la poca, especial-
mente sus descripciones pictricas y el hincapi que Bcquer pone en la expresin de las
sensaciones.
En El rayo de luna, Manrique, un noble imaginativo y poeta, ama la soledad y suea con el amor;
una noche de luna se interna en las desiertas ruinas de los Templarios, ve agitarse una cosa
blanca y cree que es el traje de una mujer misteriosa. Al hallarla a esa hora y en ese lugar, se
convierte en la dama de sus sueos e inicia su bsqueda. Dos meses despus de buscarla en
vano, se produce el descubrimiento.
Justifica la adscripcin del texto al gnero
narrativo.
Caracteriza al narrador y explica el contenido
de sus intervenciones inicial y final.
Resume el contenido del segundo fragmento.
Explica los elementos caractersticos de la
narracin presentes en l.
Personajes Accin
Tiempo Espacio
Justifica el cambio en el tiempo verbal que
se aprecia en el relato.
Analiza los recursos expresivos y la
adjetivacin empleada en este texto.
Reconoce en ambos fragmentos el empleo
de los discursos referencial, descriptivo,
dramtico, valorativo y universal.
Qu caractersticas del romanticismo
observas en el texto?
8
7
6
5
4
3
2
1
El rayo de luna
Yo no s si esto es una historia que parece cuento o un cuen-
to que parece historia; lo que puedo decir es que en su fondo
hay una verdad, una verdad muy triste, de la que acaso yo ser
uno de los ltimos en aprovecharme, dadas mis condiciones de
imaginacin.
Otro, con esta idea, tal vez hubiera hecho un tomo de filoso-
fa lacrimosa; yo he escrito esta leyenda, que a los que nada
vean en su fondo, al menos podr entretener un rato. []
La noche estaba serena y hermosa; la luna brillaba en toda
su plenitud, en lo ms alto del cielo, y el viento suspiraba con un
rumor dulcsimo entre las hojas de los rboles.
Manrique lleg al claustro, tendi la vista por su recinto, y
mir a travs de las macizas columnas de sus arcadas Esta-
ba desierto.
Sali de l, encamin sus pasos hacia la oscura alameda
que conduce al Duero, y an no haba penetrado en ella cuan-
do de sus labios se escap un grito de jbilo.
Haba visto flotar un instante y desaparecer el extremo del
traje blanco, del traje blanco de la mujer de sus sueos, de la
mujer que ya amaba como un loco.
Corre, corre en su busca; llega al sitio en que la ha visto desa -
parecer; pero al llegar se detiene, fija los espantados ojos en el
suelo, permanece un rato inmvil; un ligero temblor nervioso
agita sus miembros, un temblor que va creciendo, que va cre-
ciendo y ofrece los sntomas de una verdadera convulsin, y
prorrumpe al fin en una carcajada sonora, estridente, horrible.
Aquella cosa blanca, ligera, flotante, haba vuelto a brillar
ante sus ojos; pero haba brillado a sus pies un instante, no ms
que un instante.
Era un rayo de luna, un rayo de luna que penetraba a inter-
valos por entre la bveda de los rboles cuando el viento
mova las ramas.
***
Queris que os diga una cantiga, la ltima que ha com-
puesto mosn
1
Arnaldo, el trovador provenzal?
No! No! exclam el joven, incorporndose colrico en
su sitial. No, no quiero nada es decir, s quiero quiero
que me dejis solo Cantigas, mujeres, gloria, felici-
dad; mentira todo, fantasmas vanos que formamos en nues-
tra imaginacin y vestimos a nuestro antojo, y los amamos
y corremos tras ellos, para qu?, para qu? Para encontrar un
rayo de luna.
Manrique estaba loco; por lo menos, todo el mundo lo crea
as. A m, por el contrario, se me figuraba que lo que haba
hecho era recuperar el juicio.
Gustavo Adolfo BCQUER
Rimas y leyendas
Espasa Calpe
1
mosn: ttulo que se daba a los clrigos y a los nobles de segunda
clase en el antiguo reino de Aragn.
0B1LLLP(2008).19+s 3/6/08 18:50 Pgina 345
Evaluacin 1
9
329 19. La literatura del romanticismo
1. Enumera los elementos que se mencionan en las cua tro
primeras estrofas del poema. Qu les ocurre? En qu
se trans forman?
2. En qu se distingue de las anteriores la estructura de la
ltima estrofa? Indica qu informacin aporta acerca del
emisor y del contenido del texto.
3. Enuncia el tema que trata Gustavo Adolfo Bcquer en
esta composicin y relacinalo con otros motivos carac-
tersticos de su obra lrica.
4. Qu recurso estilstico propio de las Rimas aparece en
la nmero 33? Seala los principales recursos expresivos
y realiza su anlisis mtrico.
5. Menciona otros temas que Bcquer haya tratado con
frecuencia en las Rimas y explica su vinculacin con el
romanticismo.
6. En qu etapa del romanticismo se incluyen los poemas
de Bcquer? En qu se diferencia esta poesa de la de la
primera mitad del siglo? Cita el poeta ms importante
de este ltimo perodo y sus principales obras.
7. Explica los gneros de la prosa romntica y sita la obra
de Larra en ese contexto. Enuncia las caractersticas
fundamentales de sus artculos.
8. Menciona los rasgos principales del drama romntico
y ejemplifcalos con Don lvaro o la fuerza del sino.
9. Qu innovaciones introduce Don Juan Tenorio? Comenta
los aspectos ms relevantes de esta pieza.
10. Redacta un texto sobre la pervivencia actual del roman-
ticismo: ten en cuenta rasgos generales, temas y motivos
tratados, esttica, huella en distintos medios de expresin
(literatura, cine, series televisivas ).
Rima 33 (XXIV)
Noche y sueo, por Evelyn de Morgan.
Dos rojas lenguas de fuego
que a un mismo tronco enlazadas
se aproximan, y al besarse
forman una sola llama.
Dos notas que del lad 5
a un tiempo la mano arranca,
y en el espacio se encuentran
y armoniosas se abrazan.
Dos olas que vienen juntas
a morir sobre una playa 10
y que al romper se coronan
con un penacho de plata.
Dos jirones
1
de vapor
que del lago se levantan,
y al reunirse en el cielo 15
forman una nube blanca.
Dos ideas que al par brotan,
dos besos que a un tiempo estallan,
dos ecos que se confunden,
eso son nuestras dos almas. 20
Gustavo Adolfo BCQUER
Rimas, Castalia
1
jirn: parte o porcin pequea de un todo.
0B1LLLA.19 8/4/08 10:51 Pgina 329
REFUERZO
1. Un fragmento de la primera
parte de Fortunata y Jacinta
359 20. La narrativa realista
M
A
T
E
R
I
A
L

F
O
T
O
C
O
P
I
A
B
L
E

/


O
x
f
o
r
d

U
n
i
v
e
r
s
i
t
y

P
r
e
s
s

E
s
p
a

a
,
S
.
A
.
ACTI VI DADES
20
Si la hubieras visto! Fortunata tena los ojos como
dos estrellas, muy semejantes a los de la Virgen del Carmen que
antes estaba en Santo Toms, y ahora en San Gins. Pregnta-
selo a Estupi; pregntaselo si lo dudas, a ver Fortunata
tena las manos bastas de tanto trabajar; el corazn lleno de
inocencia Fortunata no tena educacin; aquella boca tan
linda se coma muchas letras y otras las equivocaba. Deca indi-
lugencias, golver, asn. Pas su niez cuidando el ganado.
Sabes lo que es el ganado? Las gallinas. Despus criaba los
palomos a sus pechos. Como los palomos no comen sino del
pico de la madre, Fortunata se los meta en el seno. Y si vieras
t qu seno tan bonito! Solo que tena muchos rasguos que le
hacan los palomos con los garfios de sus patas. Despus coga
en la boca un buche de agua y algunos granos de al garroba,
y metindose el pico en la boca les daba de comerEra la
paloma madre de los tiernos pichoncitos Luego les daba su
calor natural, los arrullaba, les haca rorroo, les cantaba
canciones de nodriza
Pobre Fortunata, pobre Pitusa! Te he dicho que la llama-
ban la Pitusa? No? Pues te lo digo ahora. Que conste Yo la
perd, s, que conste tambin; es preciso que cada cual car-
gue con su responsabilidad Yo la perd; la enga, le dije mil
mentiras, le hice creer que me iba a casar con ella. Has visto?
Si ser pilln! Djame que me ra un poco S, todas las
papas
1
que yo le deca se las tragaba El pueblo es muy ino-
cente, es tonto de remate; todo se lo cree con tal que se lo digan
con palabras finas
La enga, le garfi
2
su honor, y tan tranquilo. Los hom-
bres, digo, los seoritos, somos unos miserables; creemos que
el honor de las hijas del pueblo es cosa de juego No pongas
esa cara, vida ma. Comprendo que tienes razn; soy un infa-
me, merezco tu desprecio. Porque lo que t dirs: una mujer
es siempre una criatura de Dios, verdad? Y yo, despus que me
divert con ella, la dej abandonada en medio de las calles;
justo, su destino es el destino de las perras Di que s. []
Me idolatraba. Crea que yo no era como los dems, que era
la caballerosidad, la hidalgua, la decencia, la nobleza en perso-
na, el acabose de los hombres Nobleza! Qu sarcasmo! No-
bleza en la mentira; digo que no puede ser, y que no, y que
no Qu humanidad tan farsante! El pobre siempre debajo; el
rico hace lo que le da la gana. Yo soy rico Di que soy incons-
tante La ilusin de lo pintoresco se iba pasando. La grosera
con gracia seduce algn tiempo, despus marea Cada da me
pesaba ms la carga que me haba echado encima. El picor del
ajo me repugnaba. Dese, puedes creerlo, que la Pitusa fuera
mala para darle una puntera Pero, qui
3
; ni por esas
Mala ella? A buena parte Si le mando echarse al fuego por
m, al fuego de cabeza! []
El hasto era ya irresistible. La misma Pitusa me era odiosa,
como las palabras inmundas Un da dije vuelvo, y no volv
ms Lo que deca Villalonga: cortar por lo sano Yo tena
algo en mi conciencia, un hilito que me tiraba hacia all Lo
cort Fortunata me persigui: tuve que jugar al escondite.
Ella por aqu, yo por all Yo me escurra como una anguila.
No me coga, no. El ltimo a quien vi fue a Izquierdo; le encon-
tr un da subiendo la escalera de mi casa. Me amenaz; djome
que la Pitusa estaba cambr
4
de cinco meses Cambr de cin-
co meses!Alc los hombros Dos palabras l, dos palabras
yo; alargu este brazo, y plaf Izquierdo baj de golpe un
tramo entero Otro estirn, y plaf, de un brinco el segundo
tramo y con la cabeza para abajo
Benito PREZ GALDS
Fortunata y Jacinta, Hernando
1
papa: aqu mentira.
2
garfiar: (en germana) hurtar, robar.
3
qui: (en germana) interjeccin utilizada para expresar
desconocimiento o sorpresa.
4
cambr: embarazada.
Qu caractersticas del personaje
de Fortunata destaca Juan Santa Cruz
en este texto? Con qu la identifica?
Explica qu juicios formula acerca
de su relacin con ella. Por qu termina?
De qu se entera al final?
Qu visin tiene el seorito del pueblo?
Por qu afirma que les atrae a los burgueses
como l?
Seala la tcnica narrativa utilizada
por Galds en este fragmento.
Comenta el lenguaje empleado por el
personaje en relacin con su situacin
y condicin social.
Relaciona el comentario anterior
con las caractersticas de la novela realista
en general y de la obra de Galds en
particular.
3
2
1
6
5
4
Fortunata y Jacinta es una de las obras ms importante de la narrativa realista. En este fragmento,
que pertenece a la primera parte de la novela, Juan Santa Cruz, que est borracho, habla con su
mujer sobre Fortunata, durante el viaje de novios.
0B1LLLP(2008).20+s 3/6/08 18:51 Pgina 359
REFUERZO
2. Misericordia,
de Benito Prez Galds
360 20. La narrativa realista
M
A
T
E
R
I
A
L

F
O
T
O
C
O
P
I
A
B
L
E

/


O
x
f
o
r
d

U
n
i
v
e
r
s
i
t
y

P
r
e
s
s

E
s
p
a

a
,
S
.
A
.
ACTI VI DADES
20
Los ambientes retratados por la novela realista abarcan toda la diversidad social de
la poca: desde las clases ms acomodadas hasta las ms humildes, e incluso los ms
desamparados. En este pasaje, Benina, protagonista de Misericordia, luego de mostrar su
gran piedad alimentando a un nutrido grupo de indigentes, encuentra a un mendigo
anciano quien le da noticias sobre su amigo Almudena.
Djole despus el pobre viejo que se mora de hambre; que no haba entrado en su boca, en tres das, ms
que un pedazo de bacalao crudo que le dieron en una tienda, y algunos corruscos
1
de pan, que mojaba en la
fuente para reblandecerlos, porque ya no tena hueso en la boca. Desde el da de San Jos que quitaron
la sopa en el Sagrado Corazn, no haba ya remedio para l; en parte alguna encontraba amparo; el cielo
no le quera, la tierra tampoco. Con ochenta y dos aos cumplidos el 3 de febrero, San Blas bendito, un da
despus de la Candelaria, para qu quera vivir ms ni qu se le haba perdido por ac? Un hombre que
sirvi al Rey doce aos; que durante cuarenta y cinco haba picado miles de miles de toneladas de piedra en
esas carreteras de Dios, y que siempre fue bien mirado y puntoso, nada tena que hacer ya, ms que enco-
mendarse al sepulturero para que le pusiera mucha tierra, mucha tierra encima, y apisonara bien. En cuanto
que colocara a las dos criaturas, se acostara para no levantarse hasta el da del Juicio por la tardey se
levantara el ltimo! Traspasada de pena Benina al or la referencia de tanto infortunio, cuya sinceridad no
poda poner en duda, dijo al anciano que la llevara a donde estaba la nia enferma, y pronto fue conducida a
un cuarto lbrego, en la planta baja de la casa grande de corredor, donde juntos vivan, por el pago de tres
pesetas al mes, media docena de pordioseros con sus respectivas proles. La mayor parte de estos hallbanse
a la sazn en Madrid, buscando la santa perra
2
. Solo vio Benina una vieja petiseca
3
y dormilona, que pareca
alcoholizada, y una mujer panzuda, tumefacta, de piel vinosa y tirante, como la de un corambre
4
repleto, con
la cara erisipelada
5
, mal envuelta en trapos de distintos colores.
En el suelo, sobre un colchn flaco, cubierto de pedazos de bayeta amarilla y de jirones de mantas more-
llanas, yaca la nia enferma, como de seis aos, el rostro lvido, los puos cerrados en la boca. Lo que tiene
esta criatura es hambre dijo Benina, que habindola tocado en la frente y manos, la encontr fra como
el mrmol.
Puede que as sea, porque cosa caliente no ha entrado en nuestros cuerpos desde ayer.
No necesit ms la bondadosa anciana para que se le desbordase la piedad, que caudalosa inundaba su
alma; y llevando a la realidad sus intenciones con la presteza que era en ella caracterstica, fue al instante a
la tienda de comestibles, que en el ngulo de aquel edificio existe, y compr lo necesario para poner un
puchero inmediatamente, tomando adems huevos, carbn, bacalao pues ella no haca nunca las cosas
a medias. A la hora, ya estaban remediados aquellos infelices, y otros que se agregaron, inducidos por el
olor que por toda la parte baja de la colmena prontamente se difundi. Y el Seor hubo de recompensar su
caridad, deparndole, entre los mendigos que al festn acudieron, un lisiado sin piernas que andaba con los
brazos, el cual le dio por fin noticias verdicas del extraviado Almudena.
Benito PREZ GALDS
Misericordia , Ctedra
En el texto se retrata el mundo
de los mendigos; seala qu casos
presenta y las caractersticas del espacio
en el que se desarrolla la escena.
Cmo reacciona Benina? Qu consigue
la anciana con su actitud?
Comenta las tcnicas narrativas utilizadas
en este texto.
Indica las caractersticas de la novela realista
que aparecen en este pasaje de Misericordia,
de Benito Prez Galds, correspondiente
al ciclo de novelas espiritualistas del autor.
4
3
2
1
1
corrusco: parte del pan ms tostada
que corresponde a los extremos o al borde.
2
santa perra: dinero.
3
petiseca: raqutica, rugosa.
4
corambre: conjunto de cueros
o pellejos.
5
erisipelado: que padece erisipela,
infeccin microbiana de la piel.
0B1LLLP(2008).20+s 3/6/08 18:51 Pgina 360
AMPLIACIN
3. El sombrero de tres picos,
de Pedro Antonio de Alarcn
361 20. La narrativa realista
M
A
T
E
R
I
A
L

F
O
T
O
C
O
P
I
A
B
L
E

/


O
x
f
o
r
d

U
n
i
v
e
r
s
i
t
y

P
r
e
s
s

E
s
p
a

a
,
S
.
A
.
ACTI VI DADES
20
Pedro Antonio de Alarcn (1833-1891) escribi cuentos, novelas y artculos de crtica literaria.
Su primera novela, El final de Norma (1850), est dentro de la tendencia romntica. Toda su obra
posterior se caracteriza por la combinacin de rasgos del romanticismo y del realismo. Entre
sus ms importantes creaciones pueden citarse El escndalo (1875) y El nio de la bola (1880).
Se trata de novelas de tesis que evidencian un gran conservadurismo poltico y literario; la
preocupacin religiosa y por el orden social establecido son dos constantes de su literatura.
En La moral en el arte (1877), discurso que pronunci a propsito de su ingreso en la Real
Academia Espaola, expuso sus ideas acerca de la finalidad didctica y el sentido moral y reli-
gioso que, a su juicio, deba tener la novela.
Su obra ms importante es El sombrero de tres picos (1874), una narracin breve de enredo, de
carcter cmico, que transcurre a principios del siglo XIX. El relato contiene animados cuadros
de costumbres y desarrolla la historia, sacada de un romance tradicional, de un corregidor
que intenta conseguir los favores de una molinera casada.
El molino
Por varias y diversas razones, haca ya algn tiempo que aquel molino era el predilecto punto de llegada y descanso de los pa se-
antes ms caracterizados de la mencionada ciudad Primeramente, conduca a l un camino carretero, menos intransitable que
los restantes de aquellos contornos. En segundo lugar, delante del molino, haba una plazoletilla, empedrada, cubierta por un parral
enorme, debajo del cual se tomaba muy bien el fresco en verano y el sol en invierno, merced a la alternada ida y venida de los pm-
panos
1
En tercer lugar, el molinero era un hombre muy respetuoso, muy discreto, muy fino, que tena lo que se llama don de gen-
tes, y que obsequiaba a los seores que solan honrarlo con su tertulia vespertina ofrecindoles lo que daba el tiempo, ora habas
verdes, ora cerezas y guindas, ora lechugas en rama y sin sazonar (que estn muy buenas cuando se las acompaa de macarros
2
de
pan y aceite; macarros que se encargaban de enviar por delante sus seoras), ora melones, ora uvas de aquella misma parra que les
serva de dosel
3
, ora rosetas de maz si era invierno, y castaas asadas, y almendras y nueces, y de vez en cuando, en las tardes muy
fras, un trago de vino de pulso (dentro ya de la casa y al amor de la lumbre), a lo que por Pascuas se sola aadir algn pestio,
algn mantecado, algn rosco o alguna lonja de jamn alpujarreo.
Tan rico era el molinero, o tan imprudentes sus tertulianos exclamaris interrumpindome.
Ni lo uno ni lo otro. El molinero solo tena un pasar, y aquellos caballeros eran la delicadeza y el orgullo personificados. Pero en
unos tiempos en que se pagaban cincuenta y tantas contribuciones a la Iglesia y el Estado, poco arriesgaba un rstico de tan claras
luces como aquel de tenerse ganada la voluntad de regidores, cannigos, frailes, escribanos y dems personas de campanillas. As
es que no faltaba quien dijese que el to Lucas (tal era el nombre del molinero) se ahorraba un dineral al ao a fuerza de agasajar a
todo el mundo.
Vuestra merced me va a dar una puertecilla vieja de la casa que ha derribado decale a uno. Vuestra seora
decale a otro va a mandar que me rebajen el subsidio, o la alcabala
4
, o la contribucin de frutos civiles. Vuestra reverencia me
va a dejar coger en la huerta del convento una poca hoja para mis gusanos de seda. Vuestra ilustrsima me va a dar permiso para
traer una poca lea del monte X.
Pedro Antonio DE ALARCN
El sombrero de tres picos, Edelvives
La accin de El sombrero de tres picos se
desarrolla en Andaluca; comenta algunos
rasgos costumbristas.
Analiza el narrador, las perspectivas
y las tcnicas narrativas utilizadas.
Indica las intervenciones del narrador
y las apelaciones al receptor que observes.
Seala qu rasgos de la narrativa realista
estn presentes en este pasaje de El sombrero
de tres picos.
Cules son las caractersticas del protagonista
del relato de Pedro Antonio de Alarcn?
Seala qu justificacin de su conducta ofrece
el narrador.
6
5
4
3
2
1
1
pmpano: nombre que reciben las hojas de la parra.
2
macarro: panecillo de forma alargada y una libra de peso.
3
dosel: mueble que a cierta altura cubre o resguarda un altar, sitial, lecho, etc., adelantndose en pabelln horizontal y cayendo por detrs
a modo de colgadura.
4
alcabala: tributo del tanto por ciento del precio que pagaba al fisco el vendedor en el contrato de compraventa y ambos contratantes
en el de permuta.
0B1LLLP(2008).20+s 3/6/08 18:51 Pgina 361
REFUERZO
4. La procesin de Viernes Santo
en Vetusta
362 20. La narrativa realista
M
A
T
E
R
I
A
L

F
O
T
O
C
O
P
I
A
B
L
E

/


O
x
f
o
r
d

U
n
i
v
e
r
s
i
t
y

P
r
e
s
s

E
s
p
a

a
,
S
.
A
.
ACTI VI DADES
20
En el siguiente pasaje de La Regenta abundan las imgenes que representan percepciones sen-
soriales. Ana ha decidido participar en la procesin del Viernes Santo, descalza, vestida de Naza-
reno, como prueba de adhesin y fidelidad espiritual al Magistral, pero cuando llega el
momento se arrepiente. Los ciudadanos de Vetusta no se pierden el espectculo del cortejo,
admirados y escandalizados.
El va crucis de Ana Ozores
Los tambores vibraban fnebres, tristes, empeados en resucitar un dolor muerto haca dieci-
nueve siglos; a don Vctor s le sonaba aquello a himno de muerte; se le figuraba ya que llevaban a
su mujer al patbulo
1
.
El redoble del parche se destacaba en un silencio igual y montono.
En la calle estrecha, de casas oscuras, se anticipaba el crepsculo; las largas filas de hachas
encendidas se perdan a lo lejos, hacia arriba, mostrando la luz amarillenta de los pabilos
2
, como un
rosario de cuentas doradas, roto a trechos. En los cristales de las tiendas cerradas y de algunos bal-
cones se reflejaban las llamas movibles; suban y bajaban en contorsiones fantsticas, como som-
bras lucientes, en confusin de aquelarre. Aquella multitud silenciosa, aquellos pasos sin ruido,
aquellos rostros sin expresin de los colegiales de blancas albas que alumbraban con cera la calle
triste daban al conjunto apariencia de ensueo. No parecan seres vivos aquellos seminaristas
cubiertos de blanco y negro, plidos unos, con cercos morados en los ojos, otros morenos, casi
negros, de pelo en matorral, casi todos cejijuntos, preocupados con la idea fija del aburrimiento,
mquinas de hacer religin, reclutas de una leva
3
forzosa del hambre y de la holgazanera. Iban a
enterrar a Cristo, como a cualquier cristiano, sin pensar en l; a cumplir con el oficio. []
Tambin Ana pareca de madera pintada; su palidez era como un barniz. Sus ojos no vean.
A cada paso crea caer sin sentido. Senta en los pies, que pisaban las piedras y el lodo, un calor doloro-
so; cuidaba de que no asomasen debajo de la tnica morada; pero a veces se vean. Aquellos pies
desnudos eran para ella la desnudez de todo el cuerpo y de toda el alma. Ella era una loca que
haba cado en una especie de prostitucin singular!; no saba por qu, pero pensaba que despus
de aquel paseo a la vergenza ya no haba honor en su casa. All iba la tonta, la literata, Jorge San-
dio
4
la mstica, la fatua
5
, la loca, la loca sin vergenza. Ni un solo pensamiento de piedad vino en su
ayuda en todo el camino. El pensamiento no le daba ms que vinagre en aquel calvario de su reca-
to. Hasta recordaba textos de fray Luis de Len en La perfecta casada, que, segn ella, condenaban
lo que estaba haciendo. Me ceg la vanidad, no la piedad, pensaba. Yo tambin soy cmica, soy
lo que mi marido. Si alguna vez se atreva a mirar hacia atrs, a la Virgen, senta hielo en el alma.
La Madre de Jess no la miraba, no haca caso de ella; pensaba en su dolor cierto; ella, Mara, iba
all porque delante llevaba a su Hijo muerto, pero Ana, a qu iba?
Leopoldo ALAS, CLARN
La Regenta, Castalia
Explica brevemente el argumento
de este pasaje de La Regenta.
Relaciona los pensamientos y sentimientos
de Ana Ozores con lo que conoces de la obra.
Analiza el narrador y su perspectiva
en este texto del realismo.
Reconoce los casos de discurso valorativo
del narrador.
Qu tcnicas narrativas se usan para
expresar la interioridad del personaje?
El texto comienza con la descripcin de la
procesin; comenta las imgenes que
refieren percepciones acsticas y visuales.
Qu alusiones literarias se incorporan
en el fragmento?
Analiza los recursos expresivos del texto. 8
7
6
5
4
3
2
1
1
patbulo: tablado o lugar en que se ejecuta la pena
de muerte.
2
pabilo: mecha que esta en el centro de la vela.
3
leva: partida de las embarcaciones. Recluta de gente.
4
Jorge Sandio: castellanizacin de George Sand, seudnimo
de Amandine Aurore Lucille Dupin, escritora francesa.
5
fatuo: falto de razn o entendimiento. Lleno de presuncin
o vanidad infundada o ridcula.
0B1LLLP(2008).20+s 3/6/08 18:51 Pgina 362
Evaluacin 20
347 20. La narrativa realista
1. Cmo reaccionan los habitantes de Vetusta ante la
muerte de don Vctor? Cules son sus actitudes y sus
sentimientos hacia Ana? Qu crticas se observan?
2. Relaciona el pasaje con el contenido general de la novela.
3. Analiza el narrador, su perspectiva y las tcnicas narra -
tivas empleadas en el texto.
4. Seala las caractersticas de la protagonista y sus rela-
ciones con Vetusta. Alude a los otros personajes.
5. Explica los aspectos ms relevantes de esta novela:
tema, estructura, tcnicas narrativas.
6. Seala rasgos generales de la novela realista y reconoce
los que estn presentes en el texto.
7. Cita las etapas en la novelstica de Galds, explica sus
caractersticas y menciona sus principales obras.
8. Qu conflictos se desarrollan en Fortunata y Jacinta?
Analiza el espacio y el tiempo de la accin y relacinalos
con las intenciones y novelas del autor.
9. Cita otros escritores y obras del realismo.
10. Escribe una descripcin realista de una escena de tu
centro escolar: puede ser la entrada a clase o el recreo.
Servanda iba y vena como una estatua en movimiento
y los dems vetustenses no entraban en el casern de los Ozores
despus de la muerte de don Vctor.
No entraban. Vetusta la noble estaba escandalizada, horro-
rizada. Unos a otros, con cara de hipcrita compuncin, se
ocultaban los buenos vetustenses el ntimo placer que les cau-
saba aquel gran escndalo que era como una novela, algo
que interrumpa la monotona eterna de la ciudad triste. Pero
ostensiblemente pocos se alegraban de lo ocurrido. Era un
escndalo! Un adulterio descubierto! Un duelo! Un marido, un
ex regente de Audiencia, muerto de un pistoletazo en la vejiga!
En Vetusta, ni aun en los das de la revolucin haba habido
tiros. No haba costado a nadie un cartucho la conquista de los
derechos inalienables del hombre. Aquel tiro de Mesa, del que
tena la culpa la Regenta, rompa la tradicin pacfica del
crimen silencioso, morigerado y precavido. Ya se saba que
muchas damas principales de la Encimada y de la Colonia
engaaban o haban engaado o estaban a punto de engaar
a sus respectivos esposos, pero no a tiros! La envidia, que hasta se
haba disfrazado de admiracin, sali a la calle con toda la
amarillez de sus carnes.
Y result que envidiaban en secreto la hermosura y la fama
de virtuosa de la Regenta, no solo Visitacin Olas de Cuervo
y Obdulia Fandio y la baronesa de la Deuda Flotante, sino
tambin la gobernadora, y la de Pez y la seora de Carraspique
y la de Rianzares, o sea, el Gran Constantino, y las criadas de la
marquesa y toda la aristocracia, y toda la clase media y hasta las
mujeres del pueblo y quin lo dijera! la marquesa misma,
aquella doa Rufina tan liberal que con tanta magnanimidad
se absolva a s misma de las ligerezas de la juventud
y otras!
Hablaban mal de Ana Ozores todas las mujeres de Vetusta
y hasta la envidiaban y despellejaban muchos hombres con
alma como la de aquellas mujeres.
Leopoldo ALAS, CLARN
La Regenta, Alianza
La cada de Ana Ozores
Fotograma de La Regenta, serie televisiva dirigida por Fernndo Mndez-Leite. En la imagen los actores Carmelo Gmez como Fermn de Pas y Aitana Snchez-Gijn como Ana Ozores.
0B1LLLA.20 8/4/08 16:13 Pgina 347

S-ar putea să vă placă și